F R O M T H E E X P E R T S I N E N D O C R I N O L O G Y
2 0 2 2 E N D O C R I N E C A S E M A N A G E M E N T:
MEET THE PROFESSOR
FROM THE EXPERTS IN ENDOCRINOLOGY
ENDO 2022
MEET THE PROFESSOR
ENDOCRINE
CASE MANAGEMENT
2055 L Street, NW, Suite 600
Washington, DC 20036
www.endocrine.org
Other Publications:
endocrine.org/publications
The Endocrine Society is the world’s largest, oldest, and most active organization working to advance the clinical
practice of endocrinology and hormone research. Founded in 1916, the Society now has more than 18,000 global
members across a range of disciplines.
The Society has earned an international reputation for excellence in the quality of its peer-reviewed journals,
educational resources, meetings, and programs that improve public health through the practice and science of
endocrinology.
Clinical Practice Chair, ENDO 2022
Bulent O. Yildiz, MD, PhD
The statements and opinions expressed in this publication are those of the individual authors and do not
necessarily reflect the views of the Endocrine Society. The Endocrine Society is not responsible or liable in any
way for the currency of the information, for any errors, omissions or inaccuracies, or for any consequences
arising therefrom. With respect to any drugs mentioned, the reader is advised to refer to the appropriate medical
literature and the product information currently provided by the manufacturer to verify appropriate dosage,
method and duration of administration, and other relevant information. In all instances, it is the responsibility of
the treating physician or other health care professional, relying on independent experience and expertise, as well
as knowledge of the patient, to determine the best treatment for the patient.
Copyright © 2022 by the Endocrine Society, 2055 L Street, NW, Suite 600, Washington, DC 20036. All rights
reserved. No part of this publication may be reproduced, stored in a retrieval system, posted on the internet, or
transmitted in any form, by any means, electronic, mechanical, photocopying, recording, or otherwise, without
the written permission of the publisher.
Requests for permissions for reproduction should be directed to Copyright Clearance Center (CCC) at
www.copyright.com. For more information or to purchase copies, please contact Society Services by
telephone at 202-971-3646, fax at 202-736-9704, or email at [email protected] or visit the online store:
www.endocrine.org/store.
TRANSLATIONS AND LICENSING: Rights to translate and reproduce Endocrine Society publications
internationally are extended through a licensing agreement on full or partial editions. To request rights for a local
edition, please visit endocrine.org/publications or contact [email protected].
eISBN: 978-1-936704-11-8
ENDO 2022
CONTENTS
ADIPOSE TISSUE, APPETITE, AND OBESITY
Hypothalamic Obesity. . . . . . . . . . . . . . . . . . . . . . . . . . . . . . . . . . . . . . . . . . . . . . . . . . . . . . . . . . . . .2
Ashley H. Shoemaker, MD, MSCI
Electronic Tools to Help Patients During Their Weight-Loss Journey. . . . . . . . . . . . . . . . . . . . .8
Manpreet S. Mundi, MD
Update on the Medical Management of Obesity.. . . . . . . . . . . . . . . . . . . . . . . . . . . . . . . . . . . . .15
David A. D’Alessio, MD, and Jonathan E. Campbell, PhD
Long-Term Follow-Up of Bariatric Surgery in Adolescents . . . . . . . . . . . . . . . . . . . . . . . . . . . .22
Ilene Fennoy, MD, MPH
ADRENAL
Glucocorticoid-Induced Adrenal Insufficiency.. . . . . . . . . . . . . . . . . . . . . . . . . . . . . . . . . . . . . . .30
Alessandro Prete, MD, and Wiebke Arlt, MD, DSc
Advances in the Treatment of Congenital Adrenal Hyperplasia. . . . . . . . . . . . . . . . . . . . . . . .38
Adina F. Turcu, MD, MS
Hypercortisolism: A Challenging Disease to Diagnose and Manage . . . . . . . . . . . . . . . . . . . .46
Ricardo R. Correa, MD, EdD, and Katherine A. Araque, MD, MSCR
Pheochromocytoma and Paraganglioma: Diagnosis and Perisurgical Management . . . . . .54
Annika M. A. Berends, MD, and Michiel N. Kerstens, MD, PhD
Below the Tip of the Iceberg: Just How Much Aldosterone Is Out There?.. . . . . . . . . . . . . . . .68
Martin Reincke, MD
BONE AND MINERAL METABOLISM
Denosumab Discontinuation. . . . . . . . . . . . . . . . . . . . . . . . . . . . . . . . . . . . . . . . . . . . . . . . . . . . . .78
Bente L. Langdahl, MD, PhD, DMSc
Using the Best Available Evidence to Personalize Osteoporosis Treatment . . . . . . . . . . . . . .85
E. Michael Lewiecki, MD, and Micol S. Rothman, MD
ENDO 2022 • Contents iii
CARDIOVASCULAR ENDOCRINOLOGY
Choosing Cholesterol-Lowering Medications in Patients With Liver Disease. . . . . . . . . . . . .92
Savitha Subramanian, MD
Update on Genetic Causes of Hypercholesterolemia: What’s New in the Evaluation
and Treatment of These Hard-to-Treat Patients . . . . . . . . . . . . . . . . . . . . . . . . . . . . . . . . . . . . .98
Marc-Andre Cornier, MD
PCSK9 Inhibitors: Basic Biology to Clinical Outcomes . . . . . . . . . . . . . . . . . . . . . . . . . . . . . . 105
Connie B. Newman, MD, MACP
DIABETES MELLITUS AND GLUCOSE METABOLISM
Disparities in Diabetes Care and Evidence-Based Programs to Address Them .. . . . . . . . 116
Rocio I. Pereira, MD
Time in Range: The New Hemoglobin A1c?. . . . . . . . . . . . . . . . . . . . . . . . . . . . . . . . . . . . . . . . . 123
Jane E. B. Reusch, MD
Closed-Loop Insulin Delivery. . . . . . . . . . . . . . . . . . . . . . . . . . . . . . . . . . . . . . . . . . . . . . . . . . . . 131
Sue A. Brown, MD
NEUROENDOCRINOLOGY AND PITUITARY
Perioperative Management of Pituitary Adenomas . . . . . . . . . . . . . . . . . . . . . . . . . . . . . . . . 140
Ismat Shafiq, MD
Shifting Fluids: Syndrome of Inappropriate Antidiuretic Hormone Secretion
and Diabetes Insipidus. . . . . . . . . . . . . . . . . . . . . . . . . . . . . . . . . . . . . . . . . . . . . . . . . . . . . . . . . 148
Julia Kharlip, MD
Multiple Endocrine Neoplasia Type 1 Across the Lifespan . . . . . . . . . . . . . . . . . . . . . . . . . . . 158
Maria Luisa Brandi, MD, PhD
Aggressive Pituitary Tumors . . . . . . . . . . . . . . . . . . . . . . . . . . . . . . . . . . . . . . . . . . . . . . . . . . . . 164
Nienke Biermasz, MD
Controversies in the Field of Acromegaly.. . . . . . . . . . . . . . . . . . . . . . . . . . . . . . . . . . . . . . . . . 169
Elena V. Varlamov, MD, and Maria Fleseriu, MD
Management of Nonsecretory Pituitary Tumors When Surgery Fails . . . . . . . . . . . . . . . . . 180
Mark Gurnell, MD, PhD
iv ENDO 2022 • Endocrine Case Management
PEDIATRIC ENDOCRINOLOGY
Precocious Puberty: Evidence-Based Management. . . . . . . . . . . . . . . . . . . . . . . . . . . . . . . . . 188
Maria G. Vogiatzi, MD
Dilemmas in Diabetes Mellitus in Youth. . . . . . . . . . . . . . . . . . . . . . . . . . . . . . . . . . . . . . . . . . . 195
Philip S. Zeitler, MD, PhD
Novel Therapies in the Treatment of Congenital Hyperinsulinism. . . . . . . . . . . . . . . . . . . . . 205
Diva D. De Leon-Crutchlow, MD, MSCE
Approach to Pediatric Lipid Disorders. . . . . . . . . . . . . . . . . . . . . . . . . . . . . . . . . . . . . . . . . . . . 213
Ambika P. Ashraf, MD
Approach to Fractures in Children . . . . . . . . . . . . . . . . . . . . . . . . . . . . . . . . . . . . . . . . . . . . . . . 223
Rachel I. Gafni, MD
REPRODUCTIVE ENDOCRINOLOGY
Polycystic Ovary Syndrome Across the Lifespan .. . . . . . . . . . . . . . . . . . . . . . . . . . . . . . . . . . 232
Melanie Cree Green, MD, PhD, and Kathleen M. Hoeger, MD, MPH
Beginner’s Guide to Gender-Affirming Hormone Therapy
for Transgender and Gender-Diverse Adults. . . . . . . . . . . . . . . . . . . . . . . . . . . . . . . . . . . . . . . 241
Caroline J. Davidge-Pitts, MB, BCh, and Sean J. Iwamoto, MD
Diagnosis and Treatment of Male Hypogonadism. . . . . . . . . . . . . . . . . . . . . . . . . . . . . . . . . . 250
Channa N. Jayasena, PhD
Management Strategies for Primary Ovarian Insufficiency
in Adolescence: Protecting Long-Term Health . . . . . . . . . . . . . . . . . . . . . . . . . . . . . . . . . . . . . 257
Lawrence C. Layman, MD
THYROID
Molecular Diagnosis of Indeterminate Thyroid Nodules. . . . . . . . . . . . . . . . . . . . . . . . . . . . . 264
Bryan R. Haugen, MD, and Sarah E. Mayson, MD
What’s Wrong With This Picture? Challenging Cases in Thyroid Ultrasonography . . . . . 272
Jennifer A. Sipos, MD
Unmet Needs in Hypothyroidism. . . . . . . . . . . . . . . . . . . . . . . . . . . . . . . . . . . . . . . . . . . . . . . . . 277
Elizabeth A. McAninch, MD
Management of Hereditary Medullary Thyroid Cancer. . . . . . . . . . . . . . . . . . . . . . . . . . . . . . 285
Uriel Clemente-Gutierrez, MD, Bernice L. Huang, MD, Danica M. Vodopivec, MD, and Nancy D. Perrier, MD
What’s New in the Treatment of Differentiated Thyroid Cancer?. . . . . . . . . . . . . . . . . . . . . 294
Benjamin J. Gigliotti, MD
ENDO 2022 • Contents v
MISCELLANEOUS
Doping With Androgens: Abuse of Androgenic Performance-Enhancing Drugs. . . . . . . . 306
Bradley D. Anawalt, MD
E-Consults: An Evolving Area of Endocrine Care . . . . . . . . . . . . . . . . . . . . . . . . . . . . . . . . . . . 313
David Saxon, MD, and Varsha Vimalananda, MD
vi ENDO 2022 • Endocrine Case Management
ENDO 2022
FACULTY
2022 Endocrine Case Management: Meet the Professor Faculty
Bradley Anawalt, MD Diva Del Carmen De Leon- Lawrence Clarke Layman, BS, MD
University of Washington Crutchlow, MD, MSCE Medical College of Georgia
Children’s Hospital of Philadelphia at Augusta University
Wiebke Arlt, MD, DSc,
FRCP, FMedSci Ilene Fennoy, MD, MPH Mike Lewiecki, MD
University of Birmingham Columbia University New Mexico Clinical Research
& Osteoporosis Center
Ambika Ashraf, MD Maria Fleseriu, MD
Children’s Hospital/University Oregon Health & Science University Sarah Mayson, MD
of Alabama at Birmingham University of Colorado
Rachel Gafni, MD School of Medicine
Annika M. A. Berends, MD NIH
University Medical Center Groningen Elizabeth McAninch, MD
Benjamin Gigliotti, MD Stanford University
Nienke Biermasz, MD University of Rochester School of Medicine
Leiden University Medical Center School of Medicine & Dentistry
Manpreet Mundi, MD
Maria Brandi, MD, PhD Mark Gurnell, MD, PhD Mayo Clinic
FIRMO Foundation University of Cambridge, Wellcome
Trust-MRC Institute of Metabolic Connie Baum Newman, MD, MACP
Sue Brown, MD Science & School of Clinical Medicine New York University
University of Virginia School of Medicine
Bryan Haugen, MD
Jonathan Campbell, PhD University of Colorado Rocio Pereira, MD
Duke University Denver Medical Campus Denver Health Medical Center
Uriel Clemente-Gutierrez, MD Kathleen Hoeger, MD, MPH Nancy Perrier, MD
University of Texas University of Rochester University of Texas
MD Anderson Cancer Center MD Anderson Cancer Center
Bernice L. Huang, MD
Marc-Andre Cornier, MD University of Texas Alessandro Prete, MD
Medical University of South Carolina MD Anderson Cancer Center University of Birmingham
Ricardo Correa, MD, EdD Sean Iwamoto, MD Martin Reincke, MD
University of Arizona College University of Colorado Medizinische Klinik und Poliklinik IV
of Medicine Phoenix School of Medicine & Rocky Mountain
Regional VA Medical Center Jane Reusch, MD
Melanie Cree-Green, MD, PhD Rocky Mountain Regional
University of Colorado and Channa Jayasena, MD VA Medical Center
Children’s Hospital Colorado Imperial College London
Micol Rothman, MD
David D’Alessio, MD Michiel Kerstens, MD, PhD University of Colorado
Duke University School of Medicine University Medical Center Groningen School of Medicine
Caroline Davidge-Pitts, MD, MBBCH Julia Kharlip, MD David Saxon, MD
Mayo Clinic University of Pennsylvania University of Colorado &
Rocky Mountain Regional
Bente Langdahl, MD, PhD, DMSc VA Medical Center
Aarhus University Hospital
ENDO 2022 • Faculty vii
Ismat Shafiq, MD Savitha Subramanian, MD Danica M. Vodopivec, MD
University of Rochester University of Washington University of Texas
MD Anderson Cancer Center
Ashley Shoemaker, MD Adina Turcu, MD, MS
Vanderbilt University Medical Center University of Michigan Maria Vogiatzi, MD
Children’s Hospital of Philadelphia
Jennifer Sipos, MD Varsha Vimalananda, MD, MPH
Ohio State University Center for Healthcare Organization Philip Zeitler, MD, PhD
and Implementation Research Children’s Hospital Colorado/
University of Colorado
Annual Meeting Steering Committee (AMSC)
Stephen Hammes, PhD, Bulent O. Yildiz, MD, PhD – Lauren Fishbein, MD, PhD
MD – AMSC Chair Clinical Practice Chair – Clinical Science Chair
University of Rochester Hacettepe University University of Colorado
School of Medicine School of Medicine
Annual Meeting Steering Committee Clinical Peer Reviewers
Andrew Bauer, MD Stephanie Fish, MD Robin Peeters, MD, PhD
Ernesto Bernal-Mizrachi, MD Lauren Fishbein, MD, PhD Margareta Pisarska, MD
Antonio Bianco, MD, PhD Matthew Freeby, MD Philipp Scherer, PhD
Kristien Boelaert, MD, PhD Adda Grimberg, MD Jennifer Sherr, MD, PhD
Massimiliano Caprio, MD, PhD Niki Karavitaki, FRCP, PhD Robert Wermers, MD
Bart Clarke, MD Marta Korbonits, MD, PhD Selma Witchel, MD
Dawn Davis, MD, PhD Maria Veronica Mericq, MD Bulent O. Yildiz, MD
Daniel Dumesic, MD Gabrielle Page-Wilson, MD Maria-Christina Zennaro, MD, PhD
viii ENDO 2022 • Endocrine Case Management
ENDO 2022
XXXXXXXXXXXX
OVERVIEW
OVERVIEW TARGET AUDIENCE
Endocrine Case Management: Meet the Professor is designed Endocrine Case Management: Meet the Professor provides
to provide physicians with a concise and high-quality case-based education to clinicians interested in
review of more than 35 common and rare endocrine improving patient care.
disorders to help you keep your practice current. It
consists of case-based clinical vignettes and rationales by
STATEMENT OF INDEPENDENCE
experts in all areas of endocrinology, diabetes, and
As a provider of CME accredited by the Accreditation
metabolism.
Council for Continuing Medical Education, the
Endocrine Society has a policy of ensuring that the
ACCREDITATION STATEMENT content and quality of this educational activity are
The Endocrine Society is accredited by balanced, independent, objective, and scientifically
the Accreditation Council for rigorous. The scientific content of this activity was
Continuing Medical Education to developed under the supervision of the Endocrine
provide continuing medical education Society’s Annual Meeting Steering Committee.
for physicians. The Endocrine Society has received
Accreditation with Commendation.
DISCLOSURE POLICY
The Endocrine Society designates this enduring material The faculty, committee members, and staff who are in
for a maximum of 30.0 AMA PRA Category 1 Credits™. position to control the content of this activity are
Physicians should claim only the credit commensurate required to disclose to the Endocrine Society and to
with the extent of their participation in the activity. learners any relevant financial relationship(s) of the
individual or spouse/partner that have occurred within
the last 12 months with any commercial interest(s)
LEARNING OBJECTIVES whose products or services are related to the content.
Endocrine Case Management: Meet the Professor will allow Financial relationships are defined by remuneration in
learners to assess their knowledge of all aspects of any amount from the commercial interest(s) in the form
endocrinology, diabetes, and metabolism. of grants; research support; consulting fees; salary;
ownership interest (e.g., stocks, stock options, or
Upon completion of this educational activity, learners ownership interest excluding diversified mutual funds);
will be able to: honoraria or other payments for participation in
speakers’ bureaus, advisory boards, or boards of
• Recognize clinical manifestations of endocrine and directors; or other financial benefits. The intent of this
metabolic disorders and select among current disclosure is not to prevent planners with relevant
options for diagnosis, management, and therapy. financial relationships from planning or delivery of
• Identify risk factors for endocrine and metabolic content, but rather to provide learners with information
disorders and develop strategies for prevention. that allows them to make their own judgments of
whether these financial relationships may have
• Evaluate endocrine and metabolic manifestations of influenced the educational activity with regard to
systemic disorders. exposition or conclusion. The Endocrine Society has
reviewed all disclosures and resolved or managed all
• Use existing resources pertaining to clinical
identified conflicts of interest, as applicable.
guidelines and treatment recommendations for
endocrine and related metabolic disorders to guide
diagnosis and treatment.
ENDO 2022 • Overview ix
The Endocrine Society has reviewed these relationships Maria Brandi, MD, PhD; Uriel Clemente-Gutierrez,
to determine which are relevant to the content of this MD; Ricardo Correa, MD, EdD; Melanie Cree-
activity and resolved any identified conflicts of interest Green, MD, PhD; Caroline Davidge-Pitts, MD,
for these individuals. MBBCH; Ilene Fennoy, MD, MPH; Rachel Gafni,
MD; Benjamin Gigliotti, MD; Mark Gurnell, MD,
The faculty reported the following relevant financial PhD; Bryan Haugen, MD; Kathleen Hoeger, MD,
relationship(s) during the content development process for this MPH; Bernice L. Huang, MD; Sean Iwamoto, MD;
activity: Sue Brown, MD, Grant Recipient: Dexcom, Michiel Kerstens, MD, PhD; Julia Kharlip, MD;
Tandem Diabetes Care, Insulet Corporation. Jonathan Lawrence Clarke Layman, BS, MD; Sarah Mayson,
Campbell, PhD, Advisory Board Member: ShouTi; MD; Manpreet Mundi, MD; Connie Baum
Grant Recipient: Eli Lilly & Company, Novo Nordisk, Newman, MD, MACP; Rocio Pereira, MD; Nancy
Proteostasis; Speaker: Eli Lilly & Company. Marc- Perrier, MD; Alessandro Prete, MD; Martin
Andre Cornier, MD, Advisory Board Member: Reincke, MD; Jane Reusch, MD; Micol Rothman,
Regeneron Pharmaceuticals; Grant Recipient: Novartis MD; David Saxon, MD; Ismat Shafiq, MD; Jennifer
Pharmaceuticals. David D’Alessio, MD, Advisory Sipos, MD; Varsha Vimalananda, MD, MPH; Danica
Board Member: Eli Lilly & Company; Grant Recipient: M. Vodopivec, MD; and Maria Vogiatzi, MD
Merck & Co., Applied Therapeutics, Inc., Boehringer
Ingelheim, Eli Lilly & Company, Genentech, Inc., The following AMSC peer reviewers reported relevant
Hanmi, Janssen Research & Development Company, financial relationships: Andrew Bauer, MD, Speaker:
Metacrine, Novartis Pharmaceuticals, Novo Nordisk, Hexal, AG. Antonio Bianco, MD, PhD, Consulting
Oramed, Pfizer, Inc., REMD Biotherapeutics Inc., Fee: Allergan, BLA Technology, IBSA Foundation,
Sanofi, VTV Therapeutics. Research Investigator. Diva Synthonics. Kristien Boelaert, MD, PhD, Advisory
Del Carmen De Leon-Crutchlow, MD, MSCE, Board: Pfizer, EISAI. Massimiliano Caprio, MD, PhD,
Consulting Fee: Ultragenyx, Zealand Pharma, Hanmi Grant Recipient: Bayer AG. Bart Clarke, MD, Advisory
Pharmaceuticals, Heptares Therapeutics, Eiger Pharma, Board: Bristol-Myers Squibb; Consulting Fee: Shire/
Crinetics Pharmaceuticals; Grant Recipient: Twist Takeda, Inc., Calcilytix, Inc., Amolyt, Inc. Dawn Davis,
Pharma, Crinetics Pharmaceuticals; Stock Owner: MD, PhD, Employee: Department of Veterans Affairs;
Merck & Co. Maria Fleseriu, MD, Consulting Fee: Grant Recipient: Department of Veterans Affairs, NIH.
Ionis Pharmaceuticals Inc., Ipsen, Pfizer Global R&D, Daniel Dumesic, MD, Advisory Board: Spruce
Crinetics, Amryt, Recordati; Grant Recipient: Ionis BioSciences Inc.; Grant Recipient: NIH. Lauren
Pharmaceuticals Inc., Crinetics, Recordati. Channa Fishbein, MD, PhD, Advisory Board: PheoPara
Jayasena, MD, Grant Recipient: Logixx Pharma Ltd. Alliance; Consulting Fee: Lantheus/Azedra. Matthew
Bente Langdahl, MD, PhD, DMSc, Advisory Board Freeby, MD, Grant Recipient: Novo Nordisk, Abbott
Member: Amgen, UCB, Gedeon-Richter; Grant Diabetes. Adda Grimberg, MD, Advisory Board:
Recipient: Novo Nordisk, Amgen; Speaker: Amgen, Pfizer; Consulting Fee: Sandoz; Grant Recipient: Eunice
UCB, Gedeon-Richter, Astra-Zenica, Astellas. Mike Kennedy Shriver National Institute of Child Health and
Lewiecki, MD, Consulting Fee: Amgen Inc; Speaker: Human Development. Niki Karavitaki, FRCP, PhD,
Amgen Inc. Elizabeth McAninch, MD, Owner/ Advisory Board: Recordati Rare Diseases, Pfizer, Ipsen;
Co-Owner: Equilibrate Therapeutics. Ashley Speaker: HRA Pharma, Pfizer, Ipsen. Marta Korbonits,
Shoemaker, MD, Advisory Board Member: Radius MD, PhD, Consultant Fee: ONO, Novo Nordisk,
Health Inc, Saniona, Rhythm Pharmaceuticals. Savitha Corcept; Speaker: Pfizer, Ipsen. Maria Veronica
Subramanian, MD, Advisory Board Member: Abbott Mericq, MD, Advisory Board: Pfizer, Sandoz, Merck,
Laboratories, Akcea Therapeutics, Amarin. Adina Novo Nordisk; Consultant Fee: Novartis/Sandoz; Grant
Turcu, MD, MS, Advisory Board Member: Novartis; Recipient: Merck. Gabrielle Page-Wilson, MD,
Consulting Fee: CinCor, PhaseBio. Philip Zeitler, MD, Advisory Board: Strongbridge Biopharma, Recordati
PhD, Consulting Fee: Boehringer Ingelheim, Daiichi Rare Diseases, Inc. Robin Peeters, MD, PhD, Advisory
Sankyo, Eli Lilly & Company, Janssen Research & Board: Sanofi Genzyme, Bayer; Speaker: Berlin-Chemie,
Development Company, Merck. Goodlife Fertility BV, Institut Biochimique SA (IBSA),
Sanofi Genzyme, Bayer, EISAI. Jennifer Sherr, MD,
The following faculty reported no relevant financial PhD, Advisory Board: Bigfoot Biomedical, Lilly, Insulet,
relationships: Bradley Anawalt, MD; Wiebke Arlt, Cecelia Health; Consultant Fee: Medtronic Diabetes,
MD, DSc, FRCP, FMedSci; Ambika Ashraf, MD; Sanofi; Grant Recipient: JDRF
Annika M. A. Berends, MD; Nienke Biermasz, MD;
x ENDO 2022 • Endocrine Case Management
The following AMSC peer reviewers reported no relevant PRIVACY AND CONFIDENTIALITY
financial relationships: Ernesto Bernal-Mizrachi, MD; STATEMENT
Stephanie Fish, MD; Margareta Pisarska, MD; The Endocrine Society will record learner’s personal
Philipp Scherer, PhD; Robert Wermers, MD; Selma information as provided on CME evaluations to allow
Witchel, MD; Bulent O. Yildiz, MD, PhD; and for issuance and tracking of CME certificates. The
Maria-Christina Zennaro, MD, PhD Endocrine Society may also track aggregate responses to
questions in activities and evaluations and use these data
The Endocrine Society staff associated with the to inform the ongoing evaluation and improvement of
development of content for this activity reported no its CME program. No individual performance data or
relevant financial relationships. any other personal information collected from
evaluations will be shared with third parties.
DISCLAIMERS
The information presented in this activity represents ACKNOWLEDGMENT OF
the opinion of the faculty and is not necessarily the COMMERCIAL SUPPORT
official position of the Endocrine Society. This activity is not supported by educational grant(s) or
other funds from any commercial supporter.
USE OF PROFESSIONAL JUDGMENT:
The educational content in this enduring activity relates
to basic principles of diagnosis and therapy and does not AMA PRA CATEGORY 1 CREDIT
substitute for individual patient assessment based on the (CME) INFORMATION
health care provider’s examination of the patient and To receive a maximum of 30.0 AMA PRA Category 1
consideration of laboratory data and other factors Credits™, participants must complete the online
unique to the patient. Standards in medicine change as interactive module and activity evaluation located at
new data become available. https://education.endocrine.org/MTP2022.
Participants must achieve a minimum score of 70% to
DRUGS AND DOSAGES: claim CME credit. After initially completing the module,
When prescribing medications, the physician is advised if participants do not achieve a minimum score of 70%,
to check the product information sheet accompanying they have the option to change their answers and make
each drug to verify conditions of use and to identify any additional attempts to achieve a passing score. Learners
changes in drug dosage schedule or contraindications. also have the option to clear all answers and start over.
POLICY ON UNLABELED/OFF-LABEL USE METHOD OF PARTICIPATION
The Endocrine Society has determined that disclosure of This enduring material is presented online and in print
unlabeled/off-label or investigational use of commercial format. The estimated time to complete this activity,
product(s) is informative for audiences and therefore including review of material, is 30 hours. Participants
requires this information to be disclosed to the learners must achieve a minimum score of 70% to claim CME
at the beginning of the presentation. Uses of specific credit. After initially completing the module, if
therapeutic agents, devices, and other products discussed participants do not achieve a minimum score of 70%, they
in this educational activity may not be the same as those have the option to change their answers and make
indicated in product labeling approved by the Food and additional attempts to achieve a passing score. Participants
Drug Administration (FDA). The Endocrine Society also have the option to clear all answers and start over.
requires that any discussions of such “off-label” use be
based on scientific research that conforms to generally
SYSTEM REQUIREMENTS
accepted standards of experimental design, data
To complete this activity, participants must have access
collection, and data analysis. Before recommending or
to a computer or mobile device with an Internet
prescribing any therapeutic agent or device, learners
connection and use an up to date version of any major
should review the complete prescribing information,
Web browser, such as Internet Explorer 10+, Firefox
including indications, contraindications, warnings,
32+, Safari, or Google Chrome 37+. In addition, cookies
precautions, and adverse events.
and Javascript must be enabled in the browser’s options.
LAST REVIEW DATE: April 2022
ENDO 2022 • Overview xi
ACTIVITY RELEASE DATE: June 9, 2022 For questions about content or obtaining CME credit,
please contact the Endocrine Society at
ACTIVITY EXPIRATION DATE: http://education.endocrine.org/contact.
June 30, 2023 (date after which this enduring material is
no longer certified for AMA PRA Category 1 Credits™)
COMMON ABBREVIATIONS
ACTH = corticotropin FSH = follicle-stimulating hormone NPH insulin = neutral protamine
ACE inhibitor = angiotensin-converting GH = growth hormone Hagedorn insulin
enzyme inhibitor GHRH = growth hormone–releasing PCSK9 inhibitor = proprotein
ALT = alanine aminotransferase hormone convertase subtilisin/kexin 9 inhibitor
AST = aspartate aminotransferase GLP-1 receptor agonist = glucagonlike PET = positron emission tomography
BMI = body mass index peptide 1 receptor agonist PSA = prostate-specific antigen
GnRH = gonadotropin-releasing PTH = parathyroid hormone
CNS = central nervous system
hormone PTHrP = parathyroid hormone–related
CT = computed tomography
hCG = human chorionic gonadotropin protein
DHEA = dehydroepiandrosterone
HDL = high-density lipoprotein SGLT-2 inhibitor = sodium-glucose
DHEA-S = dehydroepiandrosterone cotransporter 2 inhibitor
sulfate HIV = human immunodeficiency virus
SHBG = sex hormone–binding globulin
DNA = deoxyribonucleic acid HMG-CoA reductase inhibitor =
3-hydroxy-3-methylglutaryl coenzyme A T3 = triiodothyronine
DPP-4 inhibitor = dipeptidyl-peptidase
reductase inhibitor T4 = thyroxine
4 inhibitor
IGF-1 = insulinlike growth factor 1 TPO antibodies = thyroperoxidase
DXA = dual-energy x-ray
LDL = low-density lipoprotein antibodies
absorptiometry
LH = luteinizing hormone TRH = thyrotropin-releasing hormone
FDA = Food and Drug Administration
MCV = mean corpuscular volume TRAb = thyrotropin-receptor antibodies
FGF-23 = fibroblast growth factor 23
MIBG = meta-iodobenzylguanidine TSH = thyrotropin
FNA = fine-needle aspiration
MRI = magnetic resonance imaging VLDL = very low-density lipoprotein
xii ENDO 2022 • Endocrine Case Management
ADIPOSE TISSUE,
APPETITE, AND OBESITY
Hypothalamic Obesity
Ashley H. Shoemaker, MD, MSCI. Department of Pediatrics, Endocrinology, and Diabetes,
Vanderbilt University Medical Center, Nashville, TN; E-mail:
[email protected]Learning Objectives Significance of the
As a result of participating in this session, learners Clinical Problem
should be able to: Hypothalamic obesity occurs in up to 60%
• Identify signs and symptoms of hypothalamic of patients with tumors in the hypothalamic
obesity. region, most commonly craniopharyngiomas.
Hypothalamic dysfunction can be due to
• Explain the risks and benefits of available tumor infiltration or be a consequence of
treatment options for hypothalamic obesity. surgery or radiation therapy. Survival rates for
craniopharyngiomas are excellent with an overall
5-year survival rate of 80%, but survivors still
face a 5-times greater overall mortality rate and
Main Conclusions a 3-times greater cardiovascular mortality rate
Hypothalamic obesity is a common complication compared with rates in the general population.1
of tumors in the hypothalamic region. It is Those who develop hypothalamic obesity have
characterized by rapid weight gain and is often even greater morbidity and mortality than normal-
accompanied by hypopituitarism. Risk factors weight survivors.2 Prevention and treatment of
for development of hypothalamic obesity obesity in this population is vital to decrease the
include large tumor size, damage to the posterior morbidity and mortality from diabetes, stroke, and
hypothalamus, hypopituitarism, pretreatment myocardial infarction.
obesity, and younger age. Hypothalamic obesity
is refractory to treatment and is highlighted by
Barriers to Optimal Practice
variable hyperphagia, reduced energy expenditure,
and hyperinsulinemia. Lifestyle modifications Multiple abnormalities in energy balance
including a reduced calorie, low-carbohydrate diet, lead to hypothalamic obesity. Patients with
and regular physical activity are recommended craniopharyngioma who have obesity may have
but are unlikely to lead to significant weight loss. decreased resting energy expenditure compared
Medications such as GLP-1 receptor agonists with control patients who have obesity, most likely
have shown potential to mitigate weight gain due to decreased sympathetic tone,3 although this
in patients with hypothalamic obesity, although has not been a consistent finding. Resting energy
weight loss is still difficult. Bariatric surgery can expenditure represents approximately two-thirds
be a safe and effective option for long-term weight of daily energy expenditure, with physical activity
loss and weight-loss maintenance. and other nonexercise activities accounting for the
remaining one-third. Patients with hypothalamic
obesity typically have a decreased daily activity
level. Patients with craniopharyngioma have
decreased movement counts as measured by
accelerometry and rate their physical capabilities
lower than those of healthy control participants.2
2 ENDO 2022 • Endocrine Case Management
We used accelerometry to measure physical YY, and to our knowledge there are no studies
activity over a 1-year period in 31 children and evaluating glucagon, adiponectin, or GLP-1 levels
young adults with hypothalamic obesity enrolled compared with those of control participants.
in a clinical trial.4 Overall, patients spent the most There are conflicting data on hyperleptinemia in
of the time in sedentary activity and low-intensity patients with hypothalamic obesity.5,7 One concern
activity with an average of only 24 minutes per is that 2 of the studies that found hyperleptinemia
day of moderate or vigorous activity—well below in patients with hypothalamic obesity correlated
the recommendation of 60 minutes per day. Other leptin levels with BMI, not fat mass.7 We suspect
studies have also shown decreased quality of life a role of hyperleptinemia due to both the lack
scores in the areas of physical mobility, fatigue, of hypothalamic feedback and the stimulation
and energy. of leptin production from adipocytes in chronic
Along with the reduction in daily energy hyperinsulinemia.
expenditure, there is a failure to reduce food
intake. While some patients with hypothalamic
obesity have significant hyperphagia, most patients
Strategies for Diagnosis,
do not report extreme hunger or food-seeking Therapy, and/or Management
behaviors.3,5 Hyperphagia questionnaire scores are Patients may present with hypothalamic obesity
similar to those of patients with common obesity.5 at the time of brain tumor diagnosis. If a patient
Patients with hypothalamic obesity struggle with has obesity at diagnosis, they are at high risk of
accurate food recall, underreporting caloric intake hypothalamic obesity. While obesity is common
by an average of 26%.5 It is possible that the long- in the United States population, the presence of
term cognitive effects of brain tumors and their obesity at diagnosis may also signal abnormal
treatment make food recall particularly difficult weight gain over the preceding months. Most
for patients with hypothalamic obesity, although patients develop hypothalamic obesity as a result
underreporting is common in patients with all of treatment, typically surgery or radiation.2
forms of obesity. Surgical effects are seen immediately while
Hypothalamic obesity is associated with radiation effects can take months to present.
increased risk of metabolic syndrome.6 Even Presence of hypopituitarism is another risk factor
children with hypothalamic obesity have been for hypothalamic obesity, most likely due to the
found to have impaired fasting glucose and most correlation with extensive hypothalamic damage.
children with hypothalamic obesity meet criteria Posterior hypothalamic damage can be a marker
for metabolic syndrome. These patients often have for hypothalamic obesity risk. While not yet
insulin resistance and decreased insulin sensitivity. routinely used in clinical care, there is a published
A consistent finding is increased insulin secretion, MRI scoring system that may help identify patients
particularly an exaggerated first-phase response. at high risk of developing hypothalamic obesity.
In animal models, lesions of the ventromedial The hallmark of hypothalamic obesity is rapid
hypothalamus cause obesity and hyperinsulinemia weight gain leading to a notable change in weight
that is ameliorated by vagotomy. The increased trajectory. The most rapid weight gain is typically
vagal efferent activity in hypothalamic obesity seen in the first 6 months, highlighting the need for
may directly stimulate insulin secretion from the early diagnosis and intervention, as prevention of
pancreas. With the exception of bariatric surgery, weight gain can be more successful than weight loss.
currently available medical treatments have not Medical treatments have not been very
been shown to improve glucose intolerance or successful in treating hypothalamic obesity.8
metabolic syndrome in hypothalamic obesity.4 Octreotide, a somatostatin analogue that decreases
Changes in other gut hormones are not clear; insulin secretion, was one of the first medications
there are conflicting studies on ghrelin and peptide tried for hypothalamic obesity, but its use in a
ENDO 2022 • Adipose Tissue, Appetite, and Obesity 3
randomized controlled trial failed to cause weight dexamphetamine in patients with hypothalamic
loss. Additional studies have used metformin alone obesity. Another stimulant, phentermine, is approved
or in combination but did not result in significant for short-term treatment of obesity in adults and a
weight loss, although metformin may help slow phentermine/topiramate combination is approved
the rate of weight gain. Perhaps due to its safety for long-term treatment of obesity. The mechanism
profile and the prevalence of metabolic syndrome/ of action is not well understood, but most likely
prediabetes, metformin is commonly prescribed involves the hypothalamus. Since the degree of
to patients with hypothalamic obesity; 31% of hypothalamic damage is variable in hypothalamic
patients (27 of 87) in the International Registry obesity, it is possible that some patients will respond
of Hypothalamic Obesity Disorders reported use to phentermine/topiramate, but studies are needed.
of metformin for weight loss.8 Low-carbohydrate Oxytocin is synthesized in the hypothalamus
diets are often recommended in clinical practice and secreted through the posterior pituitary. Unlike
despite a lack of robust supporting evidence.9 other pituitary hormones, oxytocin is not routinely
GLP-1 is an incretin that enhances release of replaced. This neuropeptide has been shown to
insulin in response to hyperglycemia, decreases reduce appetite and increase energy expenditure.
the rate of gastric emptying, and increases satiety. Oxytocin is not approved for treatment of obesity
At pharmacologic dosages, GLP-1 crosses the or hypopituitarism. It is typically given intranasally,
blood brain barrier and has direct effects on and an effective dosage regimen is not yet defined.
GLP-1 receptors in the brain. GLP-1 receptors In Prader-Willi syndrome, a genetic obesity
are expressed in numerous brain regions outside disorder with hypothalamic dysfunction, oxytocin
of the hypothalamus, particularly the brainstem. has had variable effects on hyperphagia. An
GLP-1 receptor agonists are increasingly used oxytocin analogue, carbetocin, was evaluated by
for treatment of obesity; the US FDA has granted the US FDA in 2021 for treatment of hyperphagia
approval for liraglutide (2020) and semaglutide but was not approved due to concerns about
(2021) for chronic weight management. Due to efficacy. It is recommended that oxytocin not be
the extrahypothalamic actions of GLP-1 receptor used outside of clinical trials.
agonists, they are a potential pharmacotherapy for Bariatric surgery may be successful in
hypothalamic obesity. A randomized controlled patients with hypothalamic obesity.8,10 A recent
clinical trial in children and young adults showed retrospective, case-control study from the
evidence that GLP-1 receptor agonists may slow Netherlands showed a mean weight loss of 22%
the rate of weight gain and decrease fat mass in at 5 years in patients with hypothalamic obesity.10
patients with hypothalamic obesity.4 Unfortunately, While this was significantly less than weight loss
patients treated with GLP-1 receptor agonists also in control patients, it is still clinically significant
showed a significant reduction in total daily energy weight loss. Sleeve gastrectomy performed better
expenditure despite simply slowing their rate of than Roux-en-Y gastric bypass.10 The International
weight gain and not having significant weight loss. Registry of Hypothalamic Obesity Disorders
Central nervous system stimulants may be found that only 8% of patients underwent
helpful in treating patients with hypothalamic bariatric surgery but it was most effective with a
obesity.9 Dexamphetamine and methylphenidate median 8.2 kg/m2 decrease in BMI.8 Patients with
are most commonly used in attention-deficit hypothalamic obesity who underwent gastric
disorder and can have a side effect of decreased bypass also reported decreased hunger and cravings.
appetite. Many pediatric patients with hypothalamic
obesity also have attention and/or executive
function deficits, warranting consideration of a
stimulant. There are reports of weight stabilization
and increased daytime wakefulness from
4 ENDO 2022 • Endocrine Case Management
Clinical Case Vignettes Which of the following is the most
likely reason for this patient’s
Case 1 postoperative weight gain?
A 9.5-year-old girl presents to her pediatrician with A. Fluid retention due to diabetes insipidus
headaches that have been worsening for 10 days. B. Hypothalamic obesity due to tissue damage
She has decreased height velocity with steady weight from tumor extension
gain along the 50th percentile. Head imaging shows
C. Increased appetite due to postoperative
a complex mixed cystic and solid suprasellar mass
dexamethasone
(3 × 3 × 3.4 cm) that extends superiorly, effacing
the third ventricle and resulting in obstructive D. Postoperative hypothalamic obesity
hydrocephalus. She is admitted to the hospital, Answer: D) Postoperative hypothalamic obesity
and the tumor is resected surgically. Pathology
findings are consistent with a craniopharyngioma. This patient demonstrates the classic weight gain
She is discharged on hormone replacement for pattern of hypothalamic obesity due to tumor
panhypopituitarism, including diabetes insipidus, treatment with a sharp inflection point in the rate
and a 2-week course of dexamethasone for of weight gain from prediagnosis to posttreatment.
postoperative swelling. One month later, she Complete resection of a large tumor may increase
returns to the endocrine clinic for follow-up. Her the risk of postoperative hypothalamic obesity
parents report that she has experienced increased (Answer D). Other risk factors for hypothalamic
swelling and weight gain since coming home from obesity include her young age and evidence
the hospital. Her clothing size has increased from of endocrine dysfunction at diagnosis (poor
7 to 12. She has increased hunger and is craving of growth rate). She did not have the risk factor of
sweet foods/junk foods that she never ate before. preoperative obesity.
She snacks frequently throughout the day and is Dexamethasone can increase appetite and
hungry again immediately after mealtimes. Her weight gain, but the effects do not continue after
growth chart is shown (see Figure). the medication is discontinued (thus, Answer C
is incorrect).
Hypothalamic obesity due to tumor extension
is often more gradual in onset and the increased
weight gain is evident pretreatment. In this
patient, the rate of weight gain was steady until
the time of diagnosis and intervention (thus,
Answer B is incorrect).
While a small amount of weight gain caused
by fluid retention is possible due to overtreatment
of diabetes insipidus, a weight gain of 10 kg in
6 months cannot be explained by hypervolemia
(thus Answer A is incorrect).
Early recognition of hypothalamic
obesity allows for environmental and dietary
interventions to decrease caloric intake and slow
the rate of weight gain. A visit with a registered
dietitian can help families implement daily caloric
maximums and parent-guided portion sizes. This
patient shows symptoms of hyperphagia such as
increased interest in food and decreased satiety.
ENDO 2022 • Adipose Tissue, Appetite, and Obesity 5
Hyperphagic symptoms should be systematically C. Lifestyle modifications such as a reduced
evaluated at each clinic visit to assess efficacy of calorie diet and increased exercise
interventions. Similar to patients with Prader- D. Reduction in his hydrocortisone dosage to
Willi syndrome, patients with hypothalamic 5 mg/m2 daily
obesity and hyperphagia may benefit from
scheduled meals/snacks. A regular meal/snack Answer: A) Antidiabetes medication with weight-
schedule can help with anxiety around food and loss effects such as a GLP-1 receptor agonist
minimize food-seeking behaviors. For patients
This patient demonstrates evidence of
with food-seeking behaviors such as sneaking food
hypothalamic obesity due to radiation adverse
and nighttime eating, environmental modification
effects. Radiation can be used as adjunctive or
such as locking the pantry/refrigerator and
primary therapy for craniopharyngiomas. Proton
not leaving food and drinks on counters can
radiotherapy provides precise delivery of radiation
be effective. Since hyperinsulinism is common
due to a lower entrance dose and elimination
in hypothalamic obesity, a lower carbohydrate
of the exit dose compared with photon-beam
diet can be beneficial. While this has not been
radiotherapy. While proton radiotherapy is more
evaluated in a well-controlled study, several case
targeted, treatment of suprasellar tumors still
reports demonstrate benefit, typically starting at
confers a risk of postradiation hypopituitarism
less than 50 g of carbohydrates per day.
and hypothalamic obesity. Children are more
sensitive to radiation adverse effects, with about
Case 2 50% of children demonstrating at least 1 pituitary
A 52-year-old man is diagnosed with a hormone deficiency 4 years after treatment.
craniopharyngioma after presenting to his Hypothalamic obesity is most commonly seen
primary care physician with concerns of decreased in patients who receive 51 Gy or more to the
peripheral vision. He is treated with proton hypothalamic region.
radiotherapy. Six months after radiation therapy, Hypothalamic obesity is refractory to
he develops central hypothyroidism and adrenal treatment and aggressive therapy is recommended
insufficiency, managed with levothyroxine and to prevent further weight gain. Lifestyle
hydrocortisone, 8 mg/m2 daily. He does not report modifications (Answer C) are important, but
symptoms of hyperphagia but does describe a 50-lb success is more likely when paired with an
(22.6-kg) weight gain over the past 6 months. His antiobesity medication. Since this patient has
BMI has increased from 26 kg/m2 pretreatment evidence of hyperglycemia, medications such as
to 34 kg/m2 posttreatment. A close diet history GLP-1 receptor agonists (Answer A) and SGLT-2
reveals some increased snacking and decreased inhibitors have the potential to lower his
ability to feel full; for example, he often finishes hemoglobin A1c and support weight loss. While
his children’s leftovers when cleaning up the no medications are approved for treatment of
kitchen. Screening labs are notable for mild mixed hypothalamic obesity, GLP-1 receptor agonists
hyperlipidemia and a hemoglobin A1c value of 6.7% have shown some benefit in a phase 2 clinical
(50 mmol/mol). trial, and they are approved for treatment of
type 2 diabetes and general obesity. SGLT-2
Which of the following would be inhibitors have not been trialed in patients with
the best management strategy for hypothalamic obesity, but their mechanism of
this patient’s weight gain? action, increased glucosuria, is preserved in
A. Antidiabetes medication with weight-loss hypothalamic obesity. These medication classes
effects such as a GLP-1 receptor agonist have extrahypothalamic mechanisms of action and
therefore potential for efficacy in patients with
B. Bariatric surgery
hypothalamic obesity. Antidiabetes medications
6 ENDO 2022 • Endocrine Case Management
that increase weight gain, such as insulin and weight management is recommended as first-
sulfonylureas, should be avoided. line therapy. A tertiary medical center can be
There is no evidence that appropriate necessary for patients with panhypopituitarism
glucocorticoid replacement causes or and hypothalamic obesity due to the increased
worsens hypothalamic obesity, so reducing operative and postoperative complexity of adrenal
his hydrocortisone dosage (Answer D) is not insufficiency and diabetes insipidus. We also
indicated. encourage patients to follow clinicaltrials.gov, as
Bariatric surgery (Answer D) can be effective there are several investigational drugs targeting
in treating hypothalamic obesity, but medical hypothalamic obesity.
References
1. Bulow B, Attewell R, Hagmar L, Malmstrom P, Nordstrom CH, Erfurth EM. 6. Srinivasan S, Ogle GD, Garnett SP, Briody JN, Lee JW, Cowell CT. Features
Postoperative prognosis in craniopharyngioma with respect to cardiovascular of the metabolic syndrome after childhood craniopharyngioma. J Clin
mortality, survival, and tumor recurrence. J Clin Endocrinol Metab. Endocrinol Metab. 2004;89(1):81-86. PMID: 14715831
1998;83(11):3897-3904. PMID: 9814465 7. Roth C, Wilken B, Hanefeld F, Schroter W, Leonhardt U. Hyperphagia in
2. Muller HL, Bueb K, Bartels U, et al. Obesity after childhood children with craniopharyngioma is associated with hyperleptinaemia and a
craniopharyngioma--German multicenter study on pre-operative risk factors failure in the downregulation of appetite. Eur J Endocrinol. 1998;138(1):89-91.
and quality of life. Klin Padiatr. 2001;213(4):244-249. PMID: 11528558 PMID: 9461323
3. Holmer H, Pozarek G, Wirfalt E, et al. Reduced energy expenditure and 8. Rose SR, Horne VE, Bingham N, Jenkins T, Black J, Inge T. Hypothalamic
impaired feeding-related signals but not high energy intake reinforces obesity: 4 years of the International Registry of Hypothalamic Obesity
hypothalamic obesity in adults with childhood onset craniopharyngioma. J Disorders. Obesity (Silver Spring). 2018;26(11):1727-1732. PMID: 30296362
Clin Endocrinol Metab. 2010;95(12):5395-5402. PMID: 20826582 9. Abuzzahab MJ, Roth CL, Shoemaker AH. Hypothalamic obesity: prologue
4. Roth CL, Perez FA, Whitlock KB, et al. A phase 3 randomized clinical and promise. Horm Res Paediatr. 2019;91(2):128-136. PMID: 30884490
trial using a once-weekly glucagon-like peptide-1 receptor agonist in 10. van Santen SS, Wolf P, Kremenevski N, et al. Bariatric surgery
adolescents and young adults with hypothalamic obesity. Diabetes Obes Metab. for hypothalamic obesity in craniopharyngioma patients: a
2021;23(2):363-373. PMID: 33026160 retrospective, matched case-control study. J Clin Endocrinol Metab.
5. Shoemaker AH, Silver HJ, Buchowski M, et al. Energy balance in 2021;106(11):e4734-e4745. PMID: 34265053
hypothalamic obesity in response to treatment with a once-weekly GLP-1
receptor agonist. Int J Obes (Lond). 2022;46(3):623-629. PMID: 34975146
ENDO 2022 • Adipose Tissue, Appetite, and Obesity 7
Electronic Tools to Help
Patients During Their
Weight-Loss Journey
Manpreet S. Mundi, MD. Division of Endocrinology, Diabetes, Metabolism, and Nutrition,
Mayo Clinic, Rochester, MN; E-mail:
[email protected]Learning Objectives where feasible and beneficial. These aids, such as
smartphone applications, can be used to improve
As a result of participating in this session, learners
capture of dietary intake, which is sometimes
should be able to:
significantly underreported through typical dietary
• Discuss the use of electronic tools to assist in recall conducted in the clinical setting. Similarly,
improving dietary capture. patients often overreport their physical activity
levels, and accuracy is improved significantly with
• Review the benefits of devices such as
the use of devices with built-in accelerometers.
smartwatches in terms of physical activity.
Technological aids such as virtual visits,
smartphone applications, and web-based programs
can be used to augment standard cognitive
behavioral therapy in an effort to improve results
Main Conclusions while reducing the time allocated by providers per
The prevalence of obesity continues to rise, with patient.
some estimates predicting that 1 in 2 adults in
the United States will have obesity by 2030. The
association of obesity with higher prevalence
of medical conditions such as diabetes mellitus,
Significance of the
hyperlipidemia, hypertension, obstructive Clinical Problem
sleep apnea, and cancer will have devastating Despite significant efforts to raise awareness and
implications for health care resources. Although develop novel treatment options, the prevalence
we have a plethora of treatment options for of obesity continues to rise. Without significant
managing obesity, including state-of-the art changes in these trends, it is estimated that by
lifestyle modification programs, medications, 2030, close to 1 in 2 adult Americans will have
endoscopic procedures, and bariatric surgery, obesity or have a BMI of 30 kg/m2 or greater
implementation of these strategies can be with the prevalence being higher than 50% in
quite resource-intensive. Additionally, patient 29 states.1 More alarming is the projection that
adherence can be low, leading to suboptimal close to 1 in 4 adults will have severe obesity or a
results, especially for programs that include BMI of 35 kg/m2 or greater. This has devastating
intensive lifestyle modifications. To manage the implications for health care, as obesity is associated
high volume of patients in a cost- and resource- with more than 60 comorbid medical conditions
effective manner, we must dramatically change and increases the risk of at least 12 different types
the way in which we deliver care to patients of cancer.2 Fortunately, we have a plethora of
with obesity with the use of technological aids treatment options available, including several
8 ENDO 2022 • Endocrine Case Management
medications approved by the US FDA with programs. Unfortunately, studies have noted
indications for weight loss, endoscopic procedures, that self-reported questionnaires are largely
and bariatric surgeries. Despite the availability dependent on an individual’s recall and perception
of these modalities, we would all agree that their of their quantity and intensity of physical activity
success is dependent on a strong foundation of and are thus prone to errors from recall bias,
lifestyle modification. Several large randomized, misinterpretation, and social desirability, which
prospective trials have demonstrated that a can be worse in patients with a higher BMI.
combination of caloric restriction, increased Slootmaker et al noted that 301 adults reported a
activity, energy expenditure, and intensive median of 340 minutes of moderate- and vigorous-
cognitive behavior therapy can result in 5% to 7% intensity activity per week while the accelerometer
weight loss, with some individuals achieving much only captured 144 minutes, an overreporting
better results. of 58%.4 Other studies with similar design have
Unfortunately, even though the curriculum also noted overreporting, especially of vigorous-
from these large trials, such as the Diabetes intensity activity, in many cases by as much as
Prevention Program and the Look AHEAD trial, is double of what was actually performed.5
widely available and free to use, implementation in Cognitive behavioral therapy and structured
clinical practice has proven difficult because of lifestyle interventions (which often use frequent
several factors. The biggest limiting factors are the group or individual visits that provide nutrition
resources needed to effectively implement and physical activity education) that rely on
intensive lifestyle modification programs, which behavior change techniques, motivational
often require touch points with a frequency of interviewing, self-monitoring, and accountability,
close to once weekly for the initial weight-loss have been noted to increase physical activity,
period of 3 to 6 months, followed by less frequent reduce sedentary behavior, and improve adherence
visits in weight maintenance periods.3 to dietary interventions, resulting in significant
Additionally, successful reduction in caloric intake weight loss. As an example, the Look AHEAD trial
requires accurate capture of dietary intake and used weekly group and individual counseling
accurate assessment of the caloric content and sessions targeting a caloric goal of 1200 to
portion size of meals. Often, capture of dietary 1800 kcal daily, along with 175 minutes of
intake is conducted through self-reported recall at moderate-intensity activity per week for 6 months
the time of office visits, which depends on the followed by less frequent visits for the next
patient’s ability to remember and willingness to 6 months to achieve an average weight loss of
report. When compared with more accurate 8.6% at 1 year. Unfortunately, these programs
markers such as doubly labeled water, self- can be quite labor- and resource-intensive with
reported dietary intake can be underreported by limited reimbursement, making them difficult to
as much as 37%. Improving the accuracy of dietary implement for most clinical practices.
recall may require ongoing interviews with trained
personnel, patient education, and more accurate
assessment of macronutrient composition and
Barriers to Optimal Practice
volume. These strategies can be quite time
• Accurate dietary and activity capture.
consuming and cost prohibitive for most clinical
practices and may not overcome social and • Patient adherence to lifestyle modification.
psychological determinants of underreporting. • Cost- and resource-prohibitive intensity
Similarly, difficulty with assessing intensity of therapy needed to achieve significant
of activity level can make it difficult for patients weight loss.
to achieve the 150 minutes of moderate- to • Reimbursement for weight-loss programs.
vigorous-intensity activity recommended by most
ENDO 2022 • Adipose Tissue, Appetite, and Obesity 9
Strategies for Diagnosis, wide-angle camera around their neck capturing
an image approximately every 20 seconds.7
Therapy, and/or Management
The use of ongoing image capture reduced
As the prevalence of obesity continues to increase, underreporting of dietary intake to 9% in men
we must innovate and dramatically change the way and 7% in women. Most of this improvement
in which we deliver care for patients with obesity, occurred through capture of unreported food
including implementation of lifestyle modifications intake and misreporting errors. These previous
and intensive cognitive behavioral therapy. This techniques have involved the use of a trained
change initially starts with data capture, especially member of the team to analyze the captured
of dietary intake, activity level, and monitoring images to assess portion sizes and percentage
body weight. Transitioning away from self- consumed, again requiring significant resources
reported dietary intake to point-of-consumption to implement outside of clinical trials. Advances
dietary capture can improve accuracy. Tooze et al in artificial intelligence are allowing for the
evaluated the accuracy of dietary recall with food- development of platforms that can detect the
frequency questionnaires and 24-hour dietary amount of food consumed with similar accuracy
entry compared with doubly labeled water and to that of a dietitian analyzing images and is good
noted that food-frequency questionnaires were to excellent when compared with the weight of
associated with average underreporting of 34% in the tray. Currently, these applications are being
women and 30% in men.6 Use of 24-hour entry tested in the hospital setting where artificial
decreased the underreporting to 17% for women intelligence is used to analyze before and after
and 11% for men. The authors also found that images to capture consumption, and they rely on
social desirability, fear of a negative evaluation, knowledge of the caloric content of the meal.8 As
BMI, eating frequency, and variability of meals per artificial intelligence continues to evolve, we see
day were factors associated with underreporting. advancements being made towards the application
This and other similar trials have demonstrated of this technology to assess dietary intake for food
that although point-of-consumption dietary entry whose caloric content has not been prespecified.
can improve accuracy, it is still dependent on the With physical activity, advancements
patient measuring portion sizes accurately and in wearable device technology combining
entering all of their food intake, which can be accelerometers with sensors continue to add the
biased significantly by a number of factors ranging ability to monitor steps taken, distances traveled,
from education level and nutritional knowledge to heart rate, and energy expenditure. Additional
social desirability, especially the concept that their sensors, including those that measure oxygen
current weight is associated with their intake. As saturation, blood glucose, and the presence
such, we must continue to evaluate technological of arrhythmias, have the potential to increase
improvements such as imaging technology that safe activity for those with significant medical
can improve accuracy of dietary capture. comorbidities. A recent trial that enrolled 40
Imaging technology has been used in clinical patients with known cardiovascular disease
trials in some capacity since the 1980s to better assessed the accuracy of a consumer-grade
define portion sizes and caloric content of meals. smartwatch in measuring heart rate and energy
Many of these studies have used a handheld device expenditure.9 They noted that across variable
to manually capture a patient’s dietary intake. activity levels, the standard deviation of difference
More recently, use of wearable camera-assisted for energy expenditure was within the acceptable
dietary assessment has sought to eliminate the range for clinical practice at 17.5 kcal and the
error associated with manual image capture smartwatch tended to measure a higher value for
and further improve accuracy. In one trial, 40 energy expenditure by approximately 30.47 kcal.
ambulatory participants were asked to wear a Heart rate accuracy was much better. In addition
10 ENDO 2022 • Endocrine Case Management
to more accurate capture, what is more intriguing participants lost weight to some degree, while
is the ability of these devices to decrease sedentary 22.7% experienced more that 10% weight loss.12
behavior and increase activity. In a recent These investigators reported that frequency of
prospective trial, all participants were provided input of body weight, exercise, and dinner along
an activity tracker, with some participants being with baseline BMI were all positively correlated
blinded while others were able to monitor with the amount of weight reduction.
their step count.10 They also randomly assigned
individuals to receive brief feedback sessions to
increase activity. The combination of being able
Clinical Case Vignettes
to monitor their activity combined with brief Case 1
feedback sessions resulted in more than 10% A 44-year-old woman with depression and
increase in activity level. Another trial randomly obesity complicated by type 2 diabetes presents for
assigning individuals to both blinding and assistance with weight loss after recent diagnosis
unblinding of activity measured in a smartphone of diabetes.
app, as well as feedback through text messages, On physical examination, her height is 68.5 in
documented similar results. Patients who were (174 cm), and weight is 202 lb (92 kg) (BMI =
unblinded to their activity level increased their 30.4 kg/m2). She has central adiposity.
number of steps per day by 1024 compared with In her 20s, she reports being able to maintain
those who were blinded, while those who were her weight in the range of 120 to 140 lb (54-64 kg),
unblinded and received feedback through text although she was an avid runner at the time. In her
messages increased their steps by 2534. 30s, her activity level declined significantly because
Similar technological advancements are of knee injuries, rearing young children, and work,
being made in the implementation of cognitive and her weight stabilized in the range of 160 to
behavioral therapy to address many factors 180 lb (73-82 kg). In her 40s, she transitioned to
that lead to patient attrition. The use of virtual a consulting position that required significantly
meetings, which reduces the need to travel for more travel. Her weight gradually increased,
weekly meetings, has been greatly accelerated and depression and diabetes were diagnosed.
during the COVID-19 pandemic and shows Bupropion was prescribed for depression, and she
similar results to in-person sessions. Virtual reports that her mood has been stable for the last
meetings, however, do not reduce provider 3 years. Her diabetes is managed with metformin,
resources used, as equivalent time is needed for 500 mg twice daily, and her last hemoglobin A1c
in-person vs virtual sessions. Thus, there has measurement was 6.2% (44 mmol/mol).
been a movement to replicate many of the key She reports that although she does not capture
components of behavioral therapy and provide her dietary intake, she targets between 1200 and
them in an enhanced manner through initially 1500 calories per day. She describes being active at
web-based interface and now smartphone apps. work, as well as at home with household activities
In a clinical trial, the use of a mobile application such as gardening. She does not engage in any
that allows patients to enter dietary intake and formal exercise.
activity level combined with standard cognitive
behavioral therapy resulted in 3.1% more weight
loss than that achieved by standard therapy alone.11
Weight loss was even more significant in those
who used the application more, which is key
to efficacy of these approaches. A retrospective
analysis of more than 35,000 users of a popular
weight-loss smartphone app noted that 77.9% of
ENDO 2022 • Adipose Tissue, Appetite, and Obesity 11
Which of the following is the again keeping in mind that most individuals tend
most appropriate next step in the to underreport caloric intake.
management of this patient’s obesity? Similarly, lifestyle modifications should be
A. Initiation of second-line therapy for diabetes attempted before adding a second-line agent for
B. Intensive lifestyle program with ongoing diabetes (Answer A) given her current hemoglobin
monitoring of dietary intake and activity level A1c level of 6.2% (44 mmol/mol). There are many
medications now approved for management of
C. Referral to psychiatry for transition to another
diabetes that are either weight neutral or have
therapy for depression
weight-loss properties. Some GLP-1 receptor
D. Very low-calorie diet targeting 600 to agonists also have separate indications for
800 calories per day weight loss and can be quite effective in reducing
Answer: B) Intensive lifestyle program with ongoing caloric intake.
monitoring of dietary intake and activity level
Case 2
In this case, the initial treatment option should
focus on lifestyle modification (Answer B). A 37-year-old woman with a history of bipolar
Patients often underreport caloric intake disorder, posttraumatic stress disorder, tobacco
and overreport activity level. Starting with use (1 pack per day), and obesity complicated by
ongoing capture of her dietary intake through a type 2 diabetes, hypertension, hyperlipidemia,
smartphone application would be ideal, not only to and obstructive sleep apnea seeks help with
ascertain her intake, but also to educate her about weight loss. She began to gain weight in high
the caloric content of the food she is consuming. school and weighed 210 lb (95.3 kg) by the time
Additionally, use of an activity tracking device she graduated. Soon after graduating, she became
can assist with not only more accurate capture but pregnant with her son and reports that she
also facilitate a gradual increase in activity. These gained 60 lb (27.2 kg) during her pregnancy, with
efforts should be combined with an intensive minimal weight loss afterwards. She estimates
lifestyle program that initially meets weekly gaining 20 to 30 lb (9.1 to 13.6 kg) with each of
or biweekly for 3 to 6 months followed by less her 2 subsequent pregnancies and weighed close
frequent meetings. to 300 lb (136 kg) by age 30 years. She tried to lose
Referral to psychiatry for transition to another weight through “countless” programs, including
therapy (Answer C) would not be best next step, as a commercial calorie-counting program, but
her depression is well controlled. Bupropion when her most successful effort was meal replacement
combined with naltrexone has been shown to be whereby she lost 50 lb (22.7 kg) over 6 months in
an effective weight-loss agent.13 her early 30s.
A caloric target of 600 to 800 calories per day On physical examination, her height is 66.1 in
(Answer D) may be too restrictive, especially in (168 cm) and weight is 317 lb (143.8 kg) (BMI =
the setting of underreporting of intake. Based on 51 kg/m2). She has increased neck circumference,
predictive equations such as the Harris-Benedict acanthosis nigricans, and central adiposity.
equation, her caloric needs at her current weight Her diabetes is currently managed
would be approximately 2300 kcal. Thus, it with metformin, 1000 mg twice daily, and
would be appropriate for her to target between liraglutide, 1.8 mg daily. Her last hemoglobin A1c
1600 to 1800 calories per day combined with measurement was 7.9% (63 mmol/mol). She is
increased activity for weight loss. If necessary, not able to tolerate continuous positive airway
further reduction in calories can be attempted if pressure because she feels claustrophobic. She has
inadequate weight loss is achieved with that target, longstanding posttraumatic stress disorder due to
12 ENDO 2022 • Endocrine Case Management
childhood trauma and feels that bipolar disorder is history of childhood trauma often report that
a misdiagnosis. weight issues began at an early age and are more
She currently eats most of her meals outside of likely to have weight-related comorbidities even
the home. She works 2 jobs. On the way to her after controlling for BMI. Victims of childhood
first job, she typically stops for breakfast at a trauma also report lower self-esteem, are more
fast-food restaurant. She then skips lunch, but likely to feel judged by their health care provider,
usually stops for a hot dog or pretzel at a and are less likely to feel they are treated with
convenience store on the way to her second job. respect. Thus, history of childhood trauma, whether
Her evening meal is usually picked up on the way it is physical or sexual abuse, can dramatically
home from her second job. She also reports affect how individuals interact with their health
snacking throughout the day. Her beverage of care providers, including seeking care and being
choice is soda. She does not engage in any formal forthcoming regarding their abuse. Weight can
exercise, but she reports being quite active also be protective, leading to increased failed
at work. attempts at weight loss. Therefore, in addressing
weight loss, it is important to create a
Which of the following is the best next nonjudgmental environment that allows the
step in this patient’s management? patient to openly discuss contributing factors to
A. Add an SGLT-2 inhibitor to improve weight gain. Providers should also solicit a history
glycemic control of trauma, especially in patients with weight gain
B. Initiate dietary restriction targeting 800 kcal at a young age, such as this patient. Patients should
per day be referred to a provider trained in the management
of trauma. Often, management will require
C. Refer her for bariatric surgery (Roux-en-Y
ongoing therapy that may start with addressing
gastric bypass)
mood disorders and trauma, followed by
D. Refer her to psychiatry for evaluation of introduction of concepts focused on weight loss.
posttraumatic stress disorder/bipolar disorder Dietary restriction (Answer B) or addition of a
followed by virtual cognitive behavioral therapy second-line agent for diabetes (Answer A) would
Answer: D) Refer her to psychiatry for evaluation not be the ideal next step in the setting of an
of posttraumatic stress disorder/bipolar disorder unaddressed mood disorder and trauma history.
followed by virtual cognitive behavioral therapy Bariatric surgery (Answer C) can be very
beneficial for patients with class III obesity,
This patient’s weight-loss journey should start by producing significant weight loss, as well as
addressing her childhood trauma (Answer D), as remission of many obesity-related comorbidities
that is most likely a major contributor to weight such as diabetes. However, it is essential to ensure
gain. A recent survey of overweight and obese that patients are medically optimized and that
individuals noted that 36.8% of participants with mood disorders are stable and well managed before
class III obesity (BMI ≥40 kg/m2) reported being a proceeding with surgery.
victim of childhood trauma.14 Persons with a
References
1. Ward ZJ, Bleich SN, Cradock AL, et al. Projected U.S. state-level prevalence 3. Jensen MD, Ryan DH, Apovian CM, et al. 2013 AHA/ACC/TOS guideline
of adult obesity and severe obesity. N Engl J Med. 2019;381(25):2440-2450. for the management of overweight and obesity in adults: a report of the
PMID: 31851800 American College of Cardiology/American Heart Association Task Force on
2. Hurt RT, Edakkanambeth Varayil J, Mundi MS, Martindale RG, Ebbert JO. practice guidelines and The Obesity Society. J Am Coll Cardiol. 2014;63(25 Pt
Designation of obesity as a disease: lessons learned from alcohol and tobacco. B):2985-3023. PMID: 24239920
Curr Gastroenterol Rep. 2014;16(11):415. PMID: 25277042
ENDO 2022 • Adipose Tissue, Appetite, and Obesity 13
4. Slootmaker SM, Schuit AJ, Chinapaw MJ, Seidell JC, van Mechelen W. 10. Nanda S, Hurt RT, Croghan IT, et al. Improving physical activity and body
Disagreement in physical activity assessed by accelerometer and self-report in composition in a medical workplace using brief goal setting. Mayo Clin Proc
subgroups of age, gender, education and weight status. Int J Behav Nutr Phys Innov Qual Outcomes. 2019;3(4):495-505. PMID: 31993569
Act. 2009;6:17. PMID: 19320985 11. Spring B, Duncan JM, Janke E, et al. Integrating technology into standard
5. Quinlan C, Rattray B, Pryor D, et al. The accuracy of self-reported physical weight loss treatment: a randomized controlled trial. JAMA Intern Med.
activity questionnaires varies with sex and body mass index. PLoS ONE. 2013;173(2):105-111. PMID: 23229890
2021;16(8):e0256008. PMID: 34379676 12. Chin SO, Keum C, Woo J, et al. Successful weight reduction and maintenance
6. Tooze JA, Subar AF, Thompson FE, Troiano R, Schatzkin A, Kipnis V. by using a smartphone application in those with overweight and obesity. Sci
Psychosocial predictors of energy underreporting in a large doubly labeled Rep. 2016;6:34563. PMID: 27819345
water study. Am J Clin Nutr. 2004;79(5):795-804. PMID: 15113717 13. Khera R, Murad MH, Chandar AK, et al. Association of pharmacological
7. Gemming L, Rush E, Maddison R, et al. Wearable cameras can reduce dietary treatments for obesity with weight loss and adverse events: a systematic
under-reporting: doubly labelled water validation of a camera-assisted 24 h review and meta-analysis. JAMA. 2016;315(22):2424-2434. PMID: 27299618
recall. Br J Nutr. 2015;113(2):284-291. PMID: 25430667 14. Mundi MS, Hurt RT, Phelan SM, et al. Associations between experience
8. Van Wymelbeke-Delannoy V, Juhel C, Bole H, et al. A cross-sectional of early childhood trauma and impact on obesity status, health, as well as
reproducibility study of a standard camera sensor using artificial intelligence perceptions of obesity-related health care. Mayo Clin Proc. 2021;96(2):408-
to assess food items: The FoodIntech Project. Nutrients. 2022;14(1):221. 419. PMID: 33549259
PMID: 35011096
9. Falter M, Budts W, Goetschalckx K, Cornelissen V, Buys R. Accuracy of
Apple Watch measurements for heart rate and energy expenditure in patients
with cardiovascular disease: cross-sectional study. JMIR MHealth UHealth.
2019;7(3):e11889. PMID: 30888332
14 ENDO 2022 • Endocrine Case Management
Update on the Medical
Management of Obesity
David A. D’Alessio, MD. Department of Medicine, Division of Endocrinology, Duke
University Medical Center, Durham, NC; E-mail: david.d’
[email protected]Jonathan E. Campbell, PhD. Department of Medicine, Division of Endocrinology, Duke
University Medical Center, Durham, NC; E-mail:
[email protected]Learning Objectives Significance of the
As a result of participating in this session, learners Clinical Problem
should be able to: Obesity is a worldwide health problem that is
• Identify the individual characteristics that linked to reduced life expectancy and multiple
determine the appropriate weight-loss comorbidities. While the causes of the obesity
intervention. epidemic are multifactorial, they involve some
combination of genetic predisposition with
• Develop a better understanding of the current environmental forces, which are immutable risk
and emerging pharmacological strategies for factors for most people. However, a desire to
weight loss. lose weight is one of the most common patient
concerns in clinical medicine. Excessive adiposity
leads to an increased risk of cardiometabolic
complications, including dyslipidemia,
Main Conclusions hypertension, nonalcoholic steatosis, insulin
resistance, and type 2 diabetes mellitus (T2DM).
• A range of drugs that can cause weight loss is The World Health Organization has reported
now available. a 3-fold increase in the prevalence of obesity in
• Most agents are only modestly effective adults in the last 50 years. Even more concerning
and meet the standard of clinical efficacy is that the rate of obesity in children has
(>5% placebo-corrected weight loss) in half increased nearly 5-fold during the same period,
of patients. foreshadowing how if left unchecked, the current
• Recently developed injectable GLP-1 receptor rates of obesity will continue to accelerate.
agonists show more promise, with average
placebo-corrected weight loss of 10% to 15%. Barriers to Optimal Practice
• Incretin-based drugs in development are likely
to be even more potent. • Lifestyle interventions, primarily diet and
exercise, can lead to clinically meaningful
weight loss in the short term, but success over
periods longer than 1 year is limited.
• Bariatric surgery is currently the most effective
mechanism to produce significant, long-term
weight loss and to reverse obesity-related
ENDO 2022 • Adipose Tissue, Appetite, and Obesity 15
comorbidities. However, surgery is limited by (40 mg/dL [0.45 mmol/L]), as well as increases
access, scalability, and expense. in HDL cholesterol (5 mg/dL [0.13 mmol/L]).3
• Until recently, available drugs for weight The improvement in these risks factors for
loss had only modest efficacy (~5% placebo- cardiovascular disease increases in proportion to
corrected weight loss). the degree of weight loss, demonstrating a linear
relationship between reductions in adiposity
and improvements in cardiometabolic health.
Moreover, weight reductions of approximately
Strategies for Diagnosis,
8% lower the risk of all obesity-related cancers
Therapy, and/or Management by 16%.4 Thus, even modest reductions in body
The Threat of Obesity weight of 5% (~10 to 15 lb [4.5 to 6.8 kg] in the
to Global Health majority of individuals with obesity) can have a
significant effect on clinical outcomes. However,
Obesity, defined as a BMI greater than 30 kg/m2, given the linear relationship between weight loss
now affects more than half the population of and improvements in health, a common target
the United States. Excess weight places an for weight loss proposed by various associations
enormous mechanical and psychological stress is 10%, with even greater losses in body weight
on individuals, leading to a number of daily associated with improved metabolic outcomes.
burdens, including osteoarthritis, joint and
musculoskeletal pain, impaired body image, low
self-esteem, and depression. These alone greatly Lifestyle Interventions
decrease the quality of life for individuals with Initial actions to combat obesity often start with
obesity. Also of great concern are the numerous lifestyle modifications, including reducing caloric
comorbidities associated with obesity that lead intake through diet modifications and enhancing
to an increased risk of death. It is generally energy expenditure through physical activity.
estimated that 75% to 86% of persons with Diet-alone interventions have the greatest
T2DM have obesity, with the increased adiposity effect on weight loss, with physical activity-
proposed to increase inflammation, cause insulin alone interventions often failing to produce a
resistance, and impair metabolic homeostasis. meaningful decrease in body weight in individuals
The combination of obesity and T2DM produces with obesity.5 The combination of diet and
dyslipidemia, hypertension, coagulation defects, exercise comprises most nonpharmacological
and atherosclerosis, greatly increasing the risk or nonsurgical interventions for the treatment
of cardiovascular disease.1 Finally, obesity has of obesity. Intense lifestyle modifications in the
been estimated to account for 10% to 20% of all setting of clinical trials, where participants often
cases of cancer.2 When considered as a whole, the receive weekly treatment sessions designed to
economic costs of chronic diseases driven by the modify both eating and activity habits, have
risk factor of obesity in the United States has been proven to be effective for weight loss. In this
estimated to be $1.72 trillion, or approximately 9% setting, patients often consume fewer calories
of the gross domestic product. Thus, addressing (1200 to 2000 kcal daily, low-fat diet) and engage
the rising prevalence of obesity is a pressing need. in 150 minutes per week of physical activity.6
Even modest reductions in body weight can These lifestyle interventions lead to 7% reductions
have significant impacts on both health and quality in body weight, sufficient to produce positive
of life. Compared with weight-stable individuals, effects on cardiometabolic outcomes. However,
patients who lose 5% to 10% of their body weight it is important to emphasize that these changes
show reductions in hemoglobin A1c (0.5%), are achieved with organized lifestyle modification
fasting glycemia (20 mg/dL [1.11 mmol/L]), programs that are biased by high attrition
blood pressure (5 mm Hg), and triglycerides
16 ENDO 2022 • Endocrine Case Management
rates and fail to set generalizable outcomes for effect on the obesity problem in the United States.
individuals attempting lifestyle modifications Approximately 250,000 metabolic surgeries are
on their own. This is highlighted by the limited performed annually in the United States, with
efficacy of the organized lifestyle modification about half being Roux-en-Y gastric bypass and
programs when examining the long-term effects. half being vertical sleeve gastrectomy. Given that
Individuals who lose 7% of their body weight more than 30 million American adults have a
while in the program have a high likelihood BMI of 40 kg/m2 or greater, performing bariatric
of rebounding when managing their lifestyle surgery in this population is currently not feasible.
independently, with most study participants This is without considering the population of
returning to their pretrial body weight within 1 the additional 100 million Americans with a BMI
to 2 years. Based on the results of clinical trials, it greater that 30 kg/m2 who qualify as candidates for
seems unlikely that most individuals with obesity metabolic surgery. Moreover, the surgery requires
can maintain a reduced body weight solely with a multidisciplinary team and comes with additional
lifestyle modifications. risks associated with an invasive procedure.
Therefore, while surgical intervention for obesity
Surgical Interventions is tremendously effective with proven durability,
it fails to be an option for most individuals with
Bariatric surgery is an intervention that provides obesity who require weight loss.
effective and sustainable weight loss. Longstanding
eligibility criteria for bariatric surgery are a BMI
greater than 40 kg/m2 or a BMI greater than Pharmacological Interventions
35 kg/m2 in the presence of comorbidities. These Pharmacotherapy for obesity provides a mechanism
criteria have been challenged in recent years with to augment weight loss for individuals who do not
the demonstration that patients with lower BMI reach clinical goals (>5% weight loss) with lifestyle
also benefit from surgery. For example, patients modification alone. Early medications that provided
with T2DM and a BMI of 30 to 35 kg/m2 have weight loss were also found to increase the risk
substantial improvements in glycemic control. of cardiovascular disease, ultimately conferring
Expected weight loss following bariatric surgery is more harm than good for overweight individuals.
between 20% to 30%, depending on starting BMI This led to stricter regulatory oversight, requiring
and the surgical procedure performed. A 10-year current medications to demonstrate significant
follow-up study of 573 patients undergoing efficacy and pass cardiovascular safety concerns.
Roux-en-Y gastric bypass averaged 21% weight Most antiobesity medications target appetite
loss 10 years following the surgery,7 and only 3% through neuronal networks that control food
of the participants regained to within 5% of their intake, which has proven to be an effective way to
preoperative weight 10 years after surgery. In a promote weight loss. A brief overview of current
randomized controlled trial of bariatric surgery obesity drugs is presented below.
compared with intensive lifestyle management,
patients with T2DM who had gastric bypass or Phentermine Plus Topiramate
sleeve gastrectomy lost approximately 25% of Approved in 2012, the combination of phentermine
starting body weight and had 2% reduction of plus topiramate is available for persons with a
hemoglobin A1c, results that were 5-fold greater BMI greater than 30 kg/m2 or a BMI greater than
than in the diet and exercise group.8 Whether 27 kg/m2 with at least 1 weight-related comorbidity.
these effects of bariatric surgery to improve The mechanism for phentermine involves
diabetes are due to mechanisms beyond weight increased norepinephrine activity in the CNS that
loss alone has been debated.9 However, bariatric drives a reduction in food intake. It may also act
surgery is not a realistic means to have a major as a sympathomimetic in the periphery to increase
energy expenditure. Topiramate decreases food
ENDO 2022 • Adipose Tissue, Appetite, and Obesity 17
intake through induction of taste aversion. Maximal liraglutide (1.2/1.8 mg). It is a GLP-1 receptor
dosage (15 mg/92 mg daily) efficacy ranges between agonist that activates satiety centers in the CNS
8.7% and 9.3% placebo-subtracted weight loss in to reduce food intake. In a 56-week clinical trial,
clinical trials, with metabolic improvements in liraglutide 3.0 mg produced between 4% and 5.2%
hypertension, lipids, and fasting glycemia/insulin reductions in placebo-subtracted weight loss.
levels. Common adverse effects include paresthesias, Additional improvements in waist circumference,
dizziness, constipation, insomnia, and anxiety, lipids, hemoglobin A1c, and blood pressure were
which are possibly mitigated with dosage titration. also demonstrated in clinical trials. Liraglutide
3.0 mg has not undergone a cardiovascular
Bupropion and Naltrexone outcomes trial, but liraglutide at the lower dosages
Approved is 2014, the combination of bupropion (1.2/1.8 mg) has shown a 22% reduction in
and naltrexone is available for persons with a cardiovascular death and a 15% reduction in all-
BMI greater than 30 kg/m2 or a BMI greater than cause mortality and is approved for cardiovascular
27 kg/m2 with at least 1 weight-related comorbidity. prevention in adults with T2DM. In clinical
Bupropion inhibits the reuptake of dopamine and trials, liraglutide 3.0 mg produced meaningful
norepinephrine to increase activity in the CNS decreases in blood pressure, LDL cholesterol,
and promote satiety. Naltrexone is an antagonist and triglycerides, along with increases in HDL
of the opioid receptor that inhibits the feedback cholesterol, suggesting a similar effect on
loop on anorexigenic neurons that are activated cardiovascular outcomes. Adverse effects include
by bupropion, enhancing the efficacy. Maximal nausea, vomiting, and gastrointestinal distress that
dosage (360 mg/32 mg) efficacy was 4.8% placebo- can be potentially mitigated with dosage-titration.
subtracted weight loss in clinical trials, with significant
improvements in waist circumference, adiposity, Semaglutide 2.4 mg
and lipids. Common adverse effects include nausea, Approved in 2021, semaglutide 2.4 mg is available
constipation, headache, dizziness, and insomnia. for persons with a BMI greater than 30 kg/m2
or a BMI greater than 27 kg/m2 with at least
Orlistat 1 weight-related comorbidity. Semaglutide is a
Approved in 1999, orlistat is available for persons long-acting GLP-1 receptor agonist that is taken
with a BMI greater than 30 kg/m2 or a BMI greater weekly (liraglutide is taken daily). The mechanism
than 27 kg/m2 with at least 1 weight-related is similar to that of liraglutide 3.0 mg, where
comorbidity. It inhibits gastrointestinal lipase GLP-1 receptor agonism in the CNS promotes
activity, decreasing the absorption of dietary fat by reduction in food intake. Clinical trials showed
up to 30%. During a 2-year randomized, placebo- that semaglutide 2.4 mg produced a 10% to 15%
controlled trial, orlistat (120 mg, 3 times daily) reduction in placebo-subtracted weight loss when
produced 3.1% placebo-subtracted weight combined with behavioral therapy over a 68-week
loss and led to modest improvements in waist period.10 Semaglutide 2.4 mg produced significant
circumference and lipids. Significant adverse improvements in waist circumference, blood
effects include gastrointestinal events and pressure, lipids, and hemoglobin A1c. Cardiovascular
deficiency in fat-soluble vitamins. outcomes trials are ongoing for semaglutide
2.4 mg (expected completion in 2023); however,
Liraglutide 3.0 mg the diabetes drug semaglutide 1.0 mg produced
Approved in 2014, liraglutide 3.0 mg is available significant improvements in major adverse cardiac
for persons with a BMI greater than 30 kg/m2 events. Common adverse effects include nausea
or a BMI greater than 27 kg/m2 with at least and gastrointestinal distress that can be potentially
1 weight-related comorbidity. Liraglutide 3.0 mg mitigated with dosage-titration.
is the high-dosage option of the antidiabetes drug
18 ENDO 2022 • Endocrine Case Management
The Future of Metabolic Cagrilintide Plus Semaglutide, Novo Nordisk
Pharmacology Cargrilintide is a long-acting amylin analogue,
given in combination with semaglutide 2.4 mg.
Tirzepatide, Eli Lilly Both are once-weekly subcutaneous injectables.
Tirzepatide is a multireceptor agonist for both In a phase 1b clinical trial, the combination of
the GLP-1 receptor and the glucose-dependent cargrilintide 2.4 mg and semaglutide 2.4 mg
insulinotropic polypeptide receptor (GIPR). The given for 20 weeks to overweight individuals
metabolic outcomes of tirzepatide were compared (BMI >27 kg/m2) produced a 17.1% reduction in
with those of semaglutide 1.0 mg in a 40-week, body weight, compared with a 9.8% reduction
phase-3 clinical trial in persons with T2DM.11 for semaglutide 2.4 mg alone.12 Native amylin
Semaglutide 1.0 mg produced 6.7% reductions in is cosecreted with insulin and has been shown
body weight, while tirzepatide produced 8.5%, to reduce gastric emptying, inhibit postprandial
11%, and 13.1% weight loss with 5-, 10-, and glucagon secretion, and suppress food intake
15-mg doses, respectively. This resulted in greater in the CNS. Amylin has also been proposed to
reductions in hemoglobin A1c, fasting glycemia, regulate food preferences in the CNS. Given
triglycerides, and VLDL cholesterol when alone to individuals with obesity, high dosages
comparing all doses of tirzepatide to semaglutide. of cargrilintide (4.5 mg) reduced body weight by
All dosages of tirzepatide also produced greater 10.8% over a 26-week period. The mechanism
reductions in waist circumference and BMI. A that drives the additive effects on weight loss
direct comparison to semaglutide 2.4 mg (the with the combination of cargrilintide and
dosage of semaglutide used for obesity alone) has semaglutide remains unknown, but likely stems
not been performed. Cardiovascular outcome from enhanced activation of food suppression in
trials are currently underway and are expected the CNS. There were increased reports of nausea
to be completed in 2024. The mechanism that and vomiting in the combination therapy group
drives weight loss in response to tirzepatide vs the semaglutide alone group, supporting the
remains unresolved. There is a reduction in food notion of additive effects on CNS activity. Overall,
intake, which has been largely attributed to GLP- adverse effects were modestly increased relative
1 receptor agonism. In preclinical models, GIPR to semaglutide 2.4, supporting further pursuit of
agonism has been proposed to engage separate this intervention.
CNS networks that limit the nausea in response
to GLP-1 receptor agonism. From this, it has Glucagon Agonism
been proposed that the combination of GIPR and Currently, the most effective pharmacological
GLP-1 receptor agonism may produce enhanced agents for obesity primarily target the CNS to
efficacy on food intake, but this has yet to be reduce food intake. The combination of reduced
tested in humans. Furthermore, GIPR agonism in food intake along with interventions that enhance
adipose tissue has been shown to enhance insulin energy expenditure may provide additional
sensitivity in a weight-independent manner, efficacy for weight loss. Multiple interests have
potentially increasing the efficacy of tirzepatide pursued glucagon receptor (GCGR) agonism to
on glycemia. Adverse effects are similar to those enhance energy expenditure. Glucagon drives
of GLP-1 receptor agonists, with nausea and numerous catabolic effects in hepatocytes,
gastrointestinal distress being predominant. including enhanced metabolism of carbohydrates,
Dosage escalation over a 12-week period has lipids, and amino acids. There is some evidence to
been shown to be effective in producing a more suggest that glucagon agonism may also enhance
favorable adverse effect profile. thermogenesis in adipose tissue. The notion
that GCGR agonism also enhances endogenous
glucose production is a concern for individuals
ENDO 2022 • Adipose Tissue, Appetite, and Obesity 19
with obesity who are often hyperglycemic or complications. His current weight is 273 lb
present with T2DM. Thus, GCGR agonism has (BMI = 38 kg/m2). Since his wife died 2 years ago,
often been paired with GLP-1 receptor agonism he has been preparing his own meals but eats at
to mitigate the hyperglycemic effects, and in some restaurants half the time. To help him qualify for
cases, triple agonisms for the GCGR, the GLP-1 surgery quickly, you choose medical management.
receptor, and the GIPR are being investigated.
Glucagon/GLP-1 coagonism is often derived Which of the following options
from either engineered peptides with activity at is the best choice?
both receptors or derivatives of oxyntomodulin, A. Dulaglutide
a naturally occurring agonist for both receptors. B. Liraglutide
There are multiple ongoing clinical trials for
C. Semaglutide, injectable
the glucagon/GLP-1 coagonists, with only a
few published results. Many early combinations D. Semaglutide, oral
were discontinued due to unacceptable safety E. Whatever his health insurance will cover
profiles or lack of efficacy. However, some of
Answer: C) Semaglutide, injectable
these compounds have provided impressive
effects on weight loss. Analogue 17 (Altimmune) Injectable semaglutide (Answer C) is the most
is a balanced glucagon/GLP-1 compound that potent medical agent available for weight loss,
produced 10.3% reduction in body weight especially the 2.4 mg dosage. This man needs
over a 12-week period. Cotadutide, a separate aggressive treatment to help him meet his
balance glucagon/GLP-1 agonist, produced 5% surgeon’s criteria for knee replacement. Oral
reduction in body weight over a 52-week period, semaglutide (Answer D), dulaglutide (Answer
and LY3305677, an oxyntomodulin derivative, A), or liraglutide (Answer B) is unlikely to result
produced 6% reduction in body weight over in the 25-lb (11.3-kg) weight loss that it will take
a 12-week period. Thus, individual therapies to lower his BMI to 35 kg/m2. However, with
have varied outcome efficacy. Combinations injectable semaglutide, this degree of weight loss is
with GIPR agonism are also being pursued, with realistic. Unfortunately, GLP-1 receptor agonists
multiple companies investigating GLP-1/GIP/ are very expensive and for long-term treatment,
glucagon triagonists. This combination therapy cost concerns are an important consideration in
has yielded impressive effects on body weight in choosing drugs. However, for the 4 to 6 months
preclinical models, with phase 1 and 2 clinical that this man will require treatment, efficacy is
trials currently ongoing. Overall, the combination probably the deciding factor (thus, Answer E is
of GCGR agonism with GLP-1/GIP activity shows incorrect).
early promising effects on body weight and the
associated metabolic outcomes.
Case 2
A 55-year-old woman is referred for help with
Clinical Case Vignettes weight loss. Her BMI has been greater than
Case 1 30 kg/m2 since the birth of her third child 24 years
ago, and her weight has steadily increased. Her
A 67-year-old man presents for help with weight
current weight is 232 lb (105.2 kg). She has treated
loss. He has degenerative joint disease in both
hypertension and stage 3 chronic kidney disease
knees with limited ability to ambulate and pain
with an estimated glomerular filtration rate of 50
that disturbs his sleep. The orthopedic surgeon
to 60 mL/min per 1.73 m2, but no other major
who is caring for him wants him to achieve a
morbidities. She has tried multiple diets over the
BMI less than 35 kg/m2 because evidence suggests
years with varied success, but these have ultimately
this will decrease his risk for postoperative
20 ENDO 2022 • Endocrine Case Management
not changed her long-term weight trajectory. The most common adverse effects of GLP-1
You decide to treat her with a GLP-1 receptor receptor agonists are gastrointestinal: nausea,
agonist, and the preferred choice on her insurance vomiting, and diarrhea. Patients taking liraglutide
formulary is liraglutide. and semaglutide have also been reported to have
more cholelithiasis and episodes of clinical gall
Which of the following adverse effects bladder disease. GLP-1 receptor agonists are
is LEAST LIKELY to occur with a typically associated with an increase in heart
course of liraglutide, 3.0 mg daily? rate of 3 to 5 beats/min, but greater increases
A. Cholelithiasis have been reported. GLP-1 receptor agonists can
B. Diarrhea cause dehydration, mostly with initiation and
gastrointestinal adverse effects. This can be of a
C. Increased heart rate
sufficient magnitude to cause acute kidney injury,
D. Nephrolithiasis but increased rates of nephrolithiasis have not
E. Vomiting been reported (thus, Answer D is correct).
Answer: D) Nephrolithiasis
References
1. Van Gaal LF, Mertens IL, De Block CE. Mechanisms linking obesity with 7. Maciejewski ML, Arterburn DE, Van Scoyoc L, et al. Bariatric surgery and
cardiovascular disease. Nature. 2006;444(7121):875-880. PMID: 17167476 long-term durability of weight loss. JAMA Surg. 2016;151(11):1046-1055.
2. Lauby-Secretan B, Scoccianti C, Loomis D, et al; International Agency for PMID: 27579793
Research on Cancer Handbook Working Group. Body fatness and cancer-- 8. Schauer PR, Bhatt DL, Kirwan JP, et al; STAMPEDE Investigators. Bariatric
viewpoint of the IARC Working Group. N Engl J Med. 2016;375(8):794-798. surgery versus intensive medical therapy for diabetes - 5-year outcomes. N
PMID: 27557308 Engl J Med. 2017;376(7):641-651. PMID: 28199805
3. Wing RR, Lang W, Wadden TA, et al; Look AHEAD Research Group. 9. Yoshino M, Kayser BD, Yoshino J, et al. Effects of diet versus gastric bypass
Benefits of modest weight loss in improving cardiovascular risk factors on metabolic function in diabetes. N Engl J Med. 2020;383(8):721-732. PMID:
in overweight and obese individuals with type 2 diabetes. Diabetes Care. 32813948
2011;34(7):1481-1486. PMID: 21593294 10. Wilding JPH, Batterham RL, Calanna S, et al; STEP 1 Study Group. Once-
4. Look AHEAD Research Group, Yeh H-C, Bantle JP, et al. Intensive weight weekly semaglutide in adults with overweight or obesity. N Engl J Med.
loss intervention and cancer risk in adults with type 2 diabetes: analysis 2021;384(11):989. PMID: 33567185
of the Look AHEAD Randomized Clinical Trial. Obesity (Silver Spring). 11. Frias JP, Davies MJ, Rosenstock J, et al; SURPASS-2 Investigators.
2020;28(9):1678-1686. PMID: 32841523 Tirzepatide versus semaglutide once weekly in patients with type 2 diabetes.
5. Stephens SK, Cobiac LJ, Veerman JL. Improving diet and physical activity N Engl J Med. 2021;385(6):503-515. PMID: 34170647
to reduce population prevalence of overweight and obesity: an overview of 12. Enebo LB, Berthelsen KK, Kankam M, et al. Safety, tolerability,
current evidence. Prev Med. 2014;62:167-178. PMID: 24534460 pharmacokinetics, and pharmacodynamics of concomitant administration
6. Knowler WC, Barrett-Connor E, Fowler SE, et al; Diabetes Prevention of multiple doses of cagrilintide with semaglutide 2.4 mg for weight
Program Research Group. Reduction in the incidence of type 2 diabetes management: a randomised, controlled, phase 1b trial. Lancet.
with lifestyle intervention or metformin. N Engl J Med. 2002;346(6):393-403. 2021;397(10286):1736-1748. PMID: 33894838
PMID: 11832527
ENDO 2022 • Adipose Tissue, Appetite, and Obesity 21
Long-Term Follow-Up
of Bariatric Surgery
in Adolescents
Ilene Fennoy, MD, MPH. Department of Pediatrics, Division of Pediatric Endocrinology,
Diabetes, and Metabolism, New York, NY; E-mail:
[email protected]Learning Objectives improvement in hepatic steatosis, and another has
demonstrated improvement in kidney function
As a result of participating in this session, learners
for patients with abnormalities at baseline.
should be able to:
Both pseudotumor cerebri and obstructive sleep
• Identify clinical implications for long-term apnea improve with weight loss after bariatric
health in adolescents with severe obesity. procedures in adolescents. Although health-
related quality of life (HRQoL) improvements are
• Identify the magnitude of BMI change
well documented, psychiatric disorders are more
expected from bariatric surgical procedures.
variable in their response and frequently require
• Identify the metabolic parameters shown to ongoing postsurgical treatment.
improve postbariatric interventions, as well as Nutritional deficiencies do occur, with low values
how psychopathology problems change. for vitamin B12, iron, and vitamin D frequently
• Identify key nutritional deficiencies known to identified, particularly after Roux-en-Y gastric
occur after bariatric surgery. bypass (RYGB).
Significance of the
Main Conclusions Clinical Problem
Severe obesity in adolescents is associated with
Obesity, defined as a BMI ≥95th percentile, and
comorbidities that increase in prevalence as the
severe obesity, defined as a BMI ≥120% of the 95th
severity of the obesity increases. Bariatric surgery
percentile, has continued to increase among children
results in variable changes in BMI dependent on
aged 2 to 19 years, with recent data demonstrating
the specific surgical procedure performed, but it
an increase from 13.9% between 1999 and 2000 to
leads to an overall average decrease of –13.5 kg/m2
19.3% between 2017 and 2018. The prevalence of
(95% CI, –15.1 to -11.9 kg/m2) 1 year after the
severe obesity increased from 3.6% to 6.1% during
procedure. Weight regain often occurs after
this same period.1 As of 2015/2016, obesity was
bariatric surgery, but patients remain, on average,
present in 20.6% of 12- to 19-year-olds, and severe
significantly below their presurgical weight.
obesity was present in 7.7% of this age group.2
Inconsistency exists in the reporting of
Bariatric surgery in adolescence has become an
comorbidities. Improvements have been shown
expanding intervention to address this rise in
across studies evaluating prediabetes, diabetes,
severe obesity, particularly as recent studies now
dyslipidemia, and hypertension. Additionally,
demonstrate that patients with severe obesity are
at least 1 report has documented biopsy-proven
not always successful with lifestyle interventions.3
22 ENDO 2022 • Endocrine Case Management
Guidelines from the American Society of baseline BMIs of 52.4, 49.6, and 49.6 kg/m2 for
Metabolic and Bariatric Surgery4 and the American patients undergoing RYGB, adjustable gastric
Academy of Pediatrics5,6 support bariatric surgery banding (AGB), and SG, respectively.12 Four of
as appropriate for adolescents based on the the 23 articles had follow-up of 24 to 36 months,
definition of severe obesity as ≥120% of the 95th but the remaining studies had follow-up limited
percentile in a patient with comorbidities or to approximately 12 months. Mean BMI change
≥140% of the 95th percentile in a patient without at 12 months for all procedures was –13.5 kg/m2
comorbidities. These comorbidities include type (95% CI, –15.1 to –11.9 kg/m2). Mean BMI
2 diabetes, obstructive sleep apnea, nonalcoholic change was –17.2 kg/m2 (95% CI, –20.1 to
fatty liver disease, idiopathic intracranial –14.3 kg/m2) for RYGB; –14.1 kg/m2 (95% CI,
hypertension, orthopedic disease (particularly –17.3 to –11.7 kg/m2) for SG; and –10.5 kg/m2
slipped capital femoral epiphysis and Blount (95% CI, –11.8 to –9.1 kg/m2) for AGB.12 This
disease), gastroesophageal reflux, and hypertension. early meta-analysis described inconsistency
Prevalence of these comorbidities has been shown in reporting of comorbidity definitions and
to increase as the severity of obesity increases.7 resolution in the various studies, and only 1
Adolescents with severe obesity also report low study reported on significant numbers of patients
HRQoL; mental health disturbances; and disordered with comorbidities in which dyslipidemia was
eating encompassing loss-of-control eating, binge moderately improved.
eating, bulimia, and night eating syndrome.4,5 More recently, Pedroso et al provided a
In the adolescent population, frequency of systematic literature review and meta-analysis
surgical procedures, however, has slowly increased through 2016 of weight-loss outcomes in
from 0.8 per 100,000 in 2000 to 2.4 per 100,000 in adolescents and adults younger than 21 years
2009,8 and recently more rapidly to 22.7 per 100,000 from 24 studies spanning follow-up periods of
in 2017.9 Sleeve gastrectomy (SG) has become the 6, 12, 24, and 36 months postoperatively.13 This
most common bariatric procedure in the United study reviewed 1928 patients, of whom 1010
States10 and in adolescents in New York state.11 had AGB, 139 had SG, and 779 had RYGB. The
Outcomes have been described both in terms of observed mean change in BMI at 6 months was
weight loss and resolution of comorbidities over –5.4 kg/m2 (95% CI, –3.0 to –7.8 kg/m2) for
varied periods. AGB, –11.5 kg/m2 (95% CI, –8.8 to –14.2 kg/m2)
for SG, and –18.8 kg/m2 (95% CI, –10.9 to
–26.6 kg/m2) for RYGB. At 24 months, nadir
Barriers to Optimal Practice BMI was –11.69 kg/m2 (95% CI, –10.33 to
–13.04 kg/m2) for AGB and –27.63 kg/m2
• Limited availability of comprehensive
(95% CI, –21.56 to –33.70 kg/m2) for RYGB,
adolescent bariatric surgery programs.
with patients in both groups showing weight
• Poor reimbursement for obesity care. regain by 36 months such that BMI change
• Incomplete long-term follow-up data. from baseline was –10.34 kg/m2 (95% CI, –6.95
to –13.94 kg/m2) for AGB and –15.00 kg/m2
(95% CI, –13.50 to –16.50 kg/m2) for RYGB.
Strategies for Diagnosis, No data were reported for SG at 24 months, but
12-month and 36-month data were similar, with
Therapy, and/or Management
mean BMI change of –13.05 kg/m2 (95% CI,
Outcomes –9.68 to –16.42 kg/m2) for SG at 12 months and
Black et al conducted a meta-analysis in 2013 –13.00 kg/m2 (95% CI, –11.00 to –15.00 kg/m2) at
that evaluated 23 studies including a total of 36 months. Comorbidities were not addressed in
637 patients, ages 12 to 19 years, with reported this review.
ENDO 2022 • Adipose Tissue, Appetite, and Obesity 23
One author has published 1-year biopsy Inge et al reported 3-year outcomes of RYGB
outcome data on the effect of bariatric surgery in and SG in 242 adolescents from 5 centers in
nonalcoholic steatohepatitis and hepatic fibrosis. the United States.20 Mean BMI was 53 kg/m2
In this study, 20 participants had SG with baseline with a mean decrease of –15 kg/m2 (95% CI,
liver biopsy and repeated biopsy at 12 months.14 –16 to –13 kg/m2) overall at 3 years. The mean
Using histological criteria for grading steatosis, BMI decrease was –15 kg/m2 (95% CI, –17 to
inflammation and portal inflammation, ballooning, –14 kg/m2) in the RYGB group and –13 kg/m2
and fibrosis, the authors were able to demonstrate (95% CI, –15 to –11 kg/m2) in the SG group. At
improvement in all markers of nonalcoholic liver 3 years, only 26% of the patients still had obesity.
disease in association with the SG procedure However, 2% of those undergoing RYGB and
compared with a similar number of patients 4% of those undergoing SG had regained above
undergoing treatment with intragastric weight-loss baseline weight at 3 years. Comorbidities also
devices and 53 nonsurgical weight-loss patients. demonstrated improvement. Of the 96 patients
Only patients who underwent SG experienced with hypertension, 74% had resolution by
significant weight loss, with a decline in weight and 3 years and only 4 incident cases occurred among
BMI of 21.5% and 20.6%, respectively, at 12 months. the 98 patients with normal blood pressure at
Several authors have evaluated pediatric baseline. Of 171 patients with dyslipidemia,
patients with pseudotumor cerebri who 66% had resolution by 3 years without lipid-
underwent bariatric surgery. Follow-up was of 6 lowering medications and only 3 incident cases
to 18 months’ duration with a very small number occurred among the 39 patients without lipid
of patients reported as case reports. Sugerman abnormalities at baseline. Nineteen of 20 patients
et al described 3 adolescents who underwent with type 2 diabetes demonstrated remission
gastroplasty or gastric bypass with resolution at 3 years with a mean hemoglobin A1c value of
of pseudotumor cerebri 1 year after surgery.15 5.3% (34 mmol/mol) and fasting blood glucose
Chandra et al described 1 patient with resolution of concentration of 88 mg/dL (4.9 mmol/L).
6 months after RYGB associated with 43% excess Prediabetes, defined as a hemoglobin A1c value
weight loss16 and Hoang et al described 3 patients, of 5.7% or greater (≥39 mmol/mol) and less than
2 of whom had resolution 18 months after SG.17 6.5% (<48 mmol/mol) or fasting blood glucose
Bariatric surgery has also been reported to concentration of 100 mg/dL or greater (≥5.6 mmol/L)
relieve obstructive sleep apnea in adolescents. Kaar and less than 126 mg/dL (<7.0 mmol/L) occurred
et al identified 81 patients (mean age 16.9 years), in 19 patients at baseline. Prediabetes resolved in
of whom 54% had obstructive sleep apnea based 76% of these patients at 3 years with 1 incident
on an apnea hypopnea index of 5 or greater.18 Only case occurring. Thirty-six of 212 patients (17%)
23 were available for follow-up, of whom 66% had abnormal kidney function at baseline, with 19
had remission 5 months after surgery. Resolution of 22 patients demonstrating resolution at 3 years.
of sleep apnea was not associated with amount Ferritin concentrations were low at baseline in 11
of weight loss during the first year after surgery, of 225 patients (5%) and were low at 3 years in 98
but it was associated with lower presurgical of 171 patients (57%), with a significant increase
weight. Similar results were found by Kalra et al in low concentrations in patients who underwent
in reviewing data of adolescents who underwent either RYGB or SG. Vitamin B12 showed a similar
RYGB with a mean decrease in BMI of 19 kg/m2 significant trend, with 1 of 222 patients (<1%)
6 months after surgery.19 All 10 patients evaluated having low values at baseline and 13 of 160 patients
postoperatively demonstrated improvement or (8%) having low values at 3 years. Low albumin,
resolution of obstructive sleep apnea. folate, and vitamin D at baseline demonstrated no
Data have accumulated regarding 3-year significant change at 3 years. Prevalence of high
outcomes for both RYGB and SG. In 2015, transferrin concentrations at baseline significantly
24 ENDO 2022 • Endocrine Case Management
increased in patients undergoing RYGB but not in Nutritional disorders of vitamin B12, iron deficiency,
patients undergoing SG. and vitamin D insufficiency increased in the
In a nationwide Swedish study of adolescents surgical group compared with the control group.
aged 13 to 18 years who underwent bariatric HRQoL was also evaluated in this Swedish
surgery (RYGB), 81 patients were matched study using the SF-36 short form and the Obesity
against a control group receiving conventional Related Problem Scale.21 The physical summary
therapy and against adults aged 35 to 45 years component of the SF-36 showed improvement
undergoing RYGB.21 Patients were evaluated among adolescents in the domains of physical role
5 years after surgery. Control patients entered the functioning, general health-related perceptions
conventional weight-loss program within 1 month and physical functioning. The physical role
of the bariatric surgery date and were evaluated functioning domain was better in the surgical
5 years later. Baseline BMI was greater in both group than in the control group. Surgical
adolescent and adult surgical patients compared patients also had improvement in weight-related
with BMI in the control group. At 5 years, mean psychosocial problems. Further exploration of
BMI was –13.1 kg/m2 (95% CI, –14.5 to –11.8) in mental health issues in this cohort has demonstrated
adolescents, –12.7 kg/m2 (95% CI, –13.7 to –10.9) similar proportions of patients prescribed
in adults, and +3.3 kg/m2 (95% CI, +1.1 to +4.8) in psychiatric drugs before and after surgery
the control group. Seventy-two percent and 76% compared with the control group.22 Although
of adolescent and adult surgical patients, respectively, treatment for mental health and behavioral
reached a BMI less than 35 kg/m2. Only 7% of the disorders did not differ before surgery, the surgical
control group achieved a BMI less than 35 kg/m2. group received more specialized therapy for a
Thirty-seven percent of adolescents and 40% of mental health disorder after surgery compared
adults no longer had obesity, with only 3% of with the control group. Self-esteem and binge-
the control group achieving this status. Most eating improved in surgical patients, whereas
surgical patients achieved 20% or greater weight mood did not. Binge eating was not related to
loss (69% of adolescents and 85% of adults) while degree of weight loss.22 Zeller reported similar
69% of control patients gained weight. Nadir findings in a cohort of 14 adolescents undergoing
weight occurred at 2 years with similar weight RYGB with improvements in HRQoL overall, but
regain by 5 years in both adolescents and adults, with similar proportions remitting or requiring
approximating a 10% rise over nadir BMI. ongoing psychiatric support 6 years after surgery.23
This Swedish study reported improvement in A systematic review of bariatric surgery
multiple metabolic parameters.21 Measurements procedures in adolescents documented consistent
of glucose homeostasis at 5 years in the surgical improvement in HRQoL at 2 years across multiple
group demonstrated remission of type 2 diabetes studies irrespective of surgical procedure.24
at 5 years in all patients who had diabetes at study Physical functioning most consistently improved,
initiation. At baseline, 27% of the surgical groups with less effectiveness demonstrated for emotional
had prediabetes, which resolved in 86% at 5 years, and mental health functioning. Depressive
and 2 new cases developed. The control group symptoms were most improved at 6 to 9 months
demonstrated a 5-year prevalence of prediabetes with some deterioration thereafter.24
of 16% with 1 new case reported. Dyslipidemia Loss-of-control eating is reported to be more
prevalence decreased in surgical patients from common in obese and overweight adolescents,25
69% to 15%, while the control group manifested with eating disorders being prevalent among
a 5-year prevalence of 73%. The prevalence of adolescents before bariatric surgery.26 Higher
elevated C-reactive protein, alanine aminotransferase, measures of loss-of-control eating are associated
and blood pressure decreased in the surgical group with less successful weight loss.27 However,
and was persistent or increased in the control group. a decrease in loss-of-control eating has been
ENDO 2022 • Adipose Tissue, Appetite, and Obesity 25
associated with weight-loss interventions both surgery, additional data are needed for outcomes at
in a systematic review involving 20 of 21 studies longer time points and with consistent reporting
using behavioral, pharmacotherapy, and surgical across comorbidities.
interventions over variable periods,27 as well
as in the Teen-LABS study. In the Teen-LABS
study, prevalence of loss-of-control eating over
Clinical Case Vignettes
6 years after bariatric surgery declined from 32.4% Case 1
presurgically to 7.9% 1 year after surgery, with a A 10-year-old boy with a history of weight problems
rise to 14.5% 4 years after surgery, then a decline since age 5 or 6 years presents for weight-loss
to 11.5% 6 years after surgery.28 management. His height is 59.0 in (149.8 cm), and
In an 8-year follow-up of 58 of 74 patients weight is 163.8 lb (74.3 kg) (BMI = 33.1 kg/m2
undergoing RYGB, Inge et al reported outcomes of [149% of the 95th percentile). Screening for
patients who had surgery at a mean age of 17 years obesity comorbidities documents that he has low
with a mean presurgery BMI of 58.5 kg/m2.29 At HDL cholesterol. Abdominal ultrasonography
follow-up, mean BMI was 41.7 kg/m2. The shows an enlarged, echogenic liver.
prevalence of hypertension decreased from 47%
to 16%, dyslipidemia from 86% to 38%, and type 2 Which of the following interventions
diabetes from 16% to 2%. Mild anemia was present is most likely to successfully help
in 46%, hyperparathyroidism in 45%, and low this patient lose weight?
vitamin B12 in 16%. A. Lifestyle interventions
B. Bariatric surgery
Conclusion
Answer: B) Bariatric surgery
Currently, data regarding long-term follow-up
of bariatric surgery in adolescents beyond 5 years With respect to lifestyle interventions (Answer A),
are limited to patients who underwent RYBG. Barlow et al recently demonstrated minimal weight
The 3-year outcome data available for SG, the loss at 3 months in patients with severe obesity
most common procedure currently performed, (>120% of the 95th percentile) compared with those
are not as robust as the outcome data for bypass with modest obesity (110% of the 95th percentile),
surgery. From the weight-loss point of view, despite followed by significant weight gain at 12 months
a greater than 10 kg/m2 BMI decline, most patients in patients with severe obesity compared with
still have obesity after bariatric surgery, although weight returning to just below baseline in those
metabolic abnormalities and HRQoL substantially with modest obesity.3 This is in stark contrast to
improve. Nutritional deficiencies noted at baseline bariatric surgery interventions (Answer B) that
for vitamin D, albumin, and folate were noted result in significant weight loss at 12 months.
to be unchanged at 3 years by Inge et al, with
vitamin B12 deficiency increasing after both SG
Case 1 (continued)
and RYGB. Deficiencies of iron, vitamin B12, and
vitamin D have been described 5 years after RYGB. The patient participates in lifestyle interventions
In another study, anemia, low vitamin B12, and over the next 3 years, during which time he is
hyperparathyroidism have been reported 8 years noted to gain weight. His BMI is 47.8 kg/m2 at age
after RYGB. Mental health problems remain in 14 years. He is referred to an adolescent bariatric
patients after bariatric surgery and require ongoing surgery program. Dyslipidemia and an enlarged,
intervention. Although outcomes reported echogenic liver persist, but liver enzyme levels
to date are favorable for both weight loss and are normal. The patient now has mild systolic
comorbidity resolution 3 to 5 years after bariatric hypertension (blood pressure = 132/73 mm Hg
26 ENDO 2022 • Endocrine Case Management
[94th percentile; 78th percentile for age and Case 2
height]). He undergoes vertical SG.
An 8.9-year-old old girl with a history of
progressive obesity since age 3 to 4 years
How much change in BMI is expected
by 1 year after surgery? presents for weight management. Her height is
58.6 in (149 cm), and weight is 128.5 lb (58.3 kg)
A. –6 kg/m2 (BMI = 26.9 kg/m2 [123% of the 95th percentile]).
B. –13 kg/m2 She has a history of Chiari 1 malformation status
C. –18 kg/m2 post suboccipital decompression at age 6 years,
D. –23 kg/m2 migraine headaches, pseudotumor cerebri,
obstructive sleep apnea, chronic abdominal
Answer: B) –13 kg/m2 pain of unknown origin, anxiety, and obsessive-
compulsive behavior. Despite ongoing dietary
Average BMI change after SG is approximately
intervention, family therapy, and psychologic
–13 kg/m2 (95% CI, –11 to –17 kg/m2) (Answer
counseling, her weight continues to rise, and
B),11,12 whereas AGB results in a smaller degree of
she undergoes bariatric surgery at age 10 years.
weight loss and RYGB results in a greater degree
Weight loss after surgery is excellent, with her
of weight loss.11,12,14
BMI declining to 16.23 kg/m2 (34th percentile)
1 year after surgery.
Case 1 (continued)
One year after bariatric surgery, his BMI is Which of the following comorbidities
30.5 kg/m2. would be expected to have resolved at
1 year with successful weight loss?
Three to 5 years after bariatric surgery, A. Anxiety and obsessive-compulsive behavior
his BMI would be expected to fall into B. Obstructive sleep apnea
which of the following ranges? C. Pseudotumor cerebri
A. 35-40 kg/m2 D. A and C
B. >40 kg/m2 E. B and C
C. 30-35 kg/m2 F. All of the above
D. <30 kg/m2
Answer: E) B and C
Answer: C) 30-35 kg/m2
Both pseudotumor cerebri (Answer C) and
Patients usually reach their nadir weight 1 to obstructive sleep apnea (Answer B) have been
2 years after bariatric procedures, with some demonstrated to resolve16 or improve in the first
rebound thereafter.12,14 In this patient, baseline year after surgery.15-18 Mental health issues persist
BMI was 47.8 kg/m2 with a BMI decline of in a significant number of patients and may
–17 kg/m2 at 1 year. Given the rebound that require ongoing psychiatric intervention.
usually occurs and the data suggesting BMI at
3 years is near the BMI at 1 year, one might expect
his BMI to be approximately 30 to 35 kg/m2 Case 2 (continued)
(Answer C) at 3 years. Given her BMI of 16.93 kg/m2 (34th
percentile), should one be concerned that
she has developed an eating disorder?
A. Yes
B. No
ENDO 2022 • Adipose Tissue, Appetite, and Obesity 27
Answer: B) No surgery.26 However, their prevalence typically
decreases postsurgically and, when they remain,
Eating disorders are associated with obesity and they are associated with poor weight loss.27
are common in patients presenting for bariatric
References
1. QuickStats: prevalence of obesity* and severe obesity† among persons aged 15. Sugerman HJ, Sugerman EL, DeMaria EJ, et al. Bariatric surgery for severely
2-19 years - National Health and Nutrition Examination Survey, 1999-2000 obese adolescents. J Gastrointest Surg. 2003;7(1):102-108. PMID: 12559191
through 2017-2018. MMWR Morb Mortal Wkly Rep. 2020;69(13):390. PMID: 16. Chandra V, Dutta S, Albanese CT, et al. Clinical resolution of severely
32240130 symptomatic pseudotumor cerebri after gastric bypass in an adolescent. Surg
2. Hales CM, Fryar CD, Carroll MD, Freedman DS, Ogden CL. Trends in Obes Relat Dis. 2007;3(2):198-200. PMID: 17324634
obesity and severe obesity prevalence in US youth and adults by sex and age, 17. Hoang KB, Hooten KG, Muh CR. Shunt freedom and clinical resolution of
2007-2008 to 2015-2016. JAMA. 2018;319(16):1723-1725. PMID: 29570750 idiopathic intracranial hypertension after bariatric surgery in the pediatric
3. Barlow SE, Durand C, Salahuddin M, Pont SJ, Butte NF, Hoelscher DM. population: report of 3 cases. J Neurosurg Pediatr. 2017;20(6):511-516. PMID:
Who benefits from the intervention? Correlates of successful BMI reduction 28960170
in the Texas Childhood Obesity Demonstration Project (TX-CORD). Pediatr 18. Kaar JL, Morelli N, Russell SP, et al. Obstructive sleep apnea and early weight
Obes. 2020;15(5):e12609. PMID: 31944617 loss among adolescents undergoing bariatric surgery. Surg Obes Relat Dis.
4. Pratt JSA, Browne A, Browne NT, et al. ASMBS pediatric metabolic and 2021;17(4):711-717. PMID: 33478907
bariatric surgery guidelines, 2018. Surg Obes Relat Dis. 2018;14(7):882-901. 19. Kalra M, Inge T, Garcia V, et al. Obstructive sleep apnea in extremely
PMID: 30077361 overweight adolescents undergoing bariatric surgery. Obesity Res.
5. Armstrong SC, Bolling CF, Michalsky MP, Reichard KW, Section on 2005;13(7):1175-1179. PMID: 16076986
Obesity, Section on Surgery. Pediatric metabolic and bariatric surgery: 20. Inge TH, Courcoulas AP, Jenkins TM, et al; Teen-LABS Consortium. Weight
evidence, barriers, and best practices. Pediatrics. 2019;144(6):e20193223. loss and health status 3 years after bariatric surgery in adolescents. N Engl J
PMID: 31656225 Med. 2015;374(2):113-123. PMID: 26544725
6. Bolling CF, Armstrong SC, Reichard KW, Michalsky MP. Metabolic 21. Olbers T, Beamish AJ, Gronowitz E, et al. Laparoscopic Roux-en-Y gastric
and bariatric surgery for pediatric patients with severe obesity. Pediatrics. bypass in adolescents with severe obesity (AMOS): a prospective, 5-year,
2019;144(6):e20193224. PMID: 31656226 Swedish nationwide study. Lancet Diabetes Endocrinol. 2017;5(3):174-183.
7. Kelly AS, Barlow SE, Rao G, et al. Severe obesity in children and adolescents: PMID: 28065734
identification, associated health risks, and treatment approaches: a 22. Jarvholm K, Bruze G, Peltonen M, et al. 5-year mental health and eating
scientific statement from the American Heart Association. Circulation. pattern outcomes following bariatric surgery in adolescents: a prospective
2013;128(15):1689-1712. PMID: 24016455 cohort study. Lancet Child Adolesc Health. 2020;4(3):210-219. PMID: 31978372
8. Kelleher DC, Merrill CT, Cottrell LT, Nadler EP, Burd RS. Recent national 23. Zeller MH, Pendery EC, Reiter-Purtill J, et al. From adolescence to young
trends in the use of adolescent inpatient bariatric surgery: 2000 through 2009. adulthood: trajectories of psychosocial health following Roux-en-Y gastric
JAMA Pediatr. 2013;167(2):126-132. PMID: 23247297 bypass. Surg Obes Relat Dis. 2017;13(7):1196-1203. PMID: 28465159
9. Bouchard ME, Tian Y, Linton S, et al. Utilization trends and disparities in 24. White B, Doyle J, Colville S, Nicholls D, Viner RM, Christie D. Systematic
adolescent bariatric surgery in the United States 2009-2017. Child Obes. 2021 review of psychological and social outcomes of adolescents undergoing
[Online ahead of print] PMID: 34647817 bariatric surgery, and predictors of success. Clin Obes. 2015;5(6):312-324.
10. English WJ, DeMaria EJ, Hutter MM, et al. American Society for Metabolic PMID: 26541244
and Bariatric Surgery 2018 estimate of metabolic and bariatric procedures 25. He J, Cai Z, Fan X. Prevalence of binge and loss of control eating among
performed in the United States. Surg Obes Relat Dis. 2020;16(4):457-463. children and adolescents with overweight and obesity: an exploratory meta-
PMID: 32029370 analysis. Int J Eat Disord. 2017;50(2):91-103. PMID: 28039879
11. Humayon S, Altieri MS, Yang J, Nie L, Spaniolas K, Pryor AD. Recent trends 26. Kim RJ, Langer JM, Baker AW, Filter DE, Williams NN, Sarwer DB.
of bariatric surgery in adolescent population in the state of New York. Surg Psychosocial status in adolescents undergoing bariatric surgery. Obes Surg.
Obes Relat Dis. 2019;15(8):1388-1393. PMID: 31262649 2008;18(1):27-33. PMID: 18085345
12. Black JA, White B, Viner RM, Simmons RK. Bariatric surgery for obese 27. Moustafa AF, Quigley KM, Wadden TA, Berkowitz RI, Chao AM. A
children and adolescents: a systematic review and meta-analysis. Obes Rev. systematic review of binge eating, loss of control eating, and weight loss in
2013;14(8):634-644. PMID: 23577666 children and adolescents. Obesity Res. 2021;29(8):1259-1271. PMID: 34227229
13. Pedroso FE, Angriman F, Endo A et al. Weight loss after bariatric surgery in 28. Goldschmidt AB, Khoury J, Jenkins TM, et al. Adolescent loss-of-control
obese adolescents: a systematic review and meta-analysis. Surg Obes Relat Dis. eating and weight loss maintenance after bariatric surgery. Pediatrics.
2018;14(3):413-422. PMID: 29248351 2018;141(1):e20171659. PMID: 29237801
14. Manco M, Mosca A, De Peppo F, et al. The benefit of sleeve gastrectomy 29. Inge TH, Jenkins TM, Xanthakos SA, et al. Long-term outcomes of bariatric
in obese adolescents on nonalcoholic steatohepatitis and hepatic fibrosis. J surgery in adolescents with severe obesity (FABS-5+): a prospective follow-
Pediatr. 2017;180:31-37.e2. PMID: 27697327 up analysis. Lancet Diabetes Endocrinol. 2017;5(3):165-173. PMID: 28065736
28 ENDO 2022 • Endocrine Case Management
ADRENAL
Glucocorticoid-Induced
Adrenal Insufficiency
Alessandro Prete, MD. Institute of Metabolism and Systems Research, University of
Birmingham, Birmingham, United Kingdom; E-mail:
[email protected]Wiebke Arlt, MD, DSc. Institute of Metabolism and Systems Research, University of
Birmingham, Birmingham, United Kingdom; E-mail: [email protected]
Learning Objectives Significance of the
As a result of participating in this session, learners Clinical Problem
should be able to: Glucocorticoid-induced AI is among the most
• Identify patients at higher risk of common causes of cortisol deficiency.1,2 In the
glucocorticoid-induced adrenal insufficiency adult population, 0.5% to 1.8% are long-term
(AI). users of oral glucocorticoids, and the absolute
risk of glucocorticoid-induced AI in this group
• Diagnose and treat glucocorticoid-induced AI. is around 50%.3 Inhaled glucocorticoids are also
an established cause of HPA-axis suppression,
with approximately 8% of patients developing
glucocorticoid-induced AI.3 HPA-axis suppression
Main Conclusions is also common following intraarticular
Synthetic glucocorticoids are widely used for their glucocorticoid administration. Small-scale
antiinflammatory and immunosuppressive actions. studies have shown that up to half of patients
A possible unwanted effect of glucocorticoid can develop some degree of biochemical HPA-
treatment is the suppression of the hypothalamic- axis suppression in the first 2 months following
pituitary-adrenal (HPA) axis, which can lead to injection, but robust data regarding the duration
AI. Factors affecting the risk of glucocorticoid- and clinical consequences of HPA-axis suppression
induced AI include the duration of glucocorticoid in these patients are lacking.3 Glucocorticoid-
therapy, mode of administration, glucocorticoid induced AI is also possible following use of topical
dosage and potency, and concomitant drugs that glucocorticoids, but evidence is mostly anecdotal.
interfere with glucocorticoid metabolism. Signs A mismatch exists between the high
and symptoms of AI can be vague and nonspecific prevalence of “biochemical” glucocorticoid-
and can overlap with those of iatrogenic Cushing induced AI and the reporting of its “clinical”
syndrome and the underlying disorder for which consequences, suggesting potential underdiagnosis
glucocorticoids are prescribed. Glucocorticoid and lack of awareness among clinicians.
therapy should not be completely stopped until Considering the widespread use of glucocorticoids
recovery of adrenal function is verified. When across multiple medical specialties, the number of
glucocorticoid-induced AI is diagnosed or patients potentially exposed to harm is staggering,
suspected, patient education is essential to improve and the lack of a consensus regarding management
clinical outcomes. of glucocorticoid-induced AI contributes to
suboptimal clinical outcomes.
30 ENDO 2022 • Endocrine Case Management
Barriers to Optimal Practice overlap.1 Nonetheless, considering the various
contributing factors can help clinicians establish
• Low specificity of signs and symptoms of the individualized risk for each patient (Figure 1).
glucocorticoid-induced AI. It is important to note, however, that high-quality
evidence is limited.1
• Lack of tools to accurately stratify patients
Systemic glucocorticoids, especially those with
according to their risk of developing
longer half-life and higher potency, are more likely
glucocorticoid-induced AI.
to cause glucocorticoid-induced AI as compared
• Lack of consensus on how to assess with other administration routes because of
adrenal function in patients with suspected the direct negative feedback on the HPA axis.
glucocorticoid-induced AI. Short courses of high-dosage glucocorticoids
(<2 weeks) and pulse therapy (intermittent
intravenous administration over a few days or
Strategies for Diagnosis, weeks) are usually associated with swift recovery
Therapy, and/or Management of adrenal function. The possible exception to
this is the administration of frequently repeated
The risk of developing glucocorticoid-induced AI
short glucocorticoid courses, as regularly used in
is difficult to predict on an individual basis. The
the treatment of asthma and chronic obstructive
glucocorticoid formulation used, dosage, treatment
pulmonary disease and during emetogenic
duration, and time of administration during the
chemotherapy.1
day can affect this risk; there is a great degree of
Figure 1. Likelihood of Clinically Relevant Adrenal Suppression By Systemic and Inhaled Glucocorticoids
Systemic glucocorticoids include oral, intravenous, and intramuscular administration. The risk of developing
glucocorticoid-induced adrenal insufficiency is a continuum, and this chart is a guide only. Clinical judgment is required to
establish the risk on an individual basis.
ENDO 2022 • Adrenal 31
Inhaled glucocorticoids can enter the systemic • Glucocorticoid-induced AI is possible in patients
circulation via the gastrointestinal tract or direct with cortisol values less than 10.0 μg/dL
absorption from the lungs and can exert negative (<275 nmol/L). Glucocorticoids should be
feedback on the HPA axis. Patients using higher continued (possibly switching treatment
dosages for longer periods, as well as those to a replacement dosage of short-acting
exposed to other glucocorticoids (eg, multiple hydrocortisone [eg, 15 mg hydrocortisone])
courses of rescue treatment for chronic obstructive and adrenal function should be reassessed
pulmonary disease exacerbations) carry the highest over time. Repeated tests include early-
risk of glucocorticoid-induced AI.1,4 morning cortisol and dynamic testing when
The data regarding glucocorticoid-induced results are indeterminate. Dynamic tests
AI associated with intraarticular glucocorticoid include the 250-mcg cosyntropin-stimulation
therapy are limited and heterogeneous. Small-scale test, overnight metyrapone-stimulation
studies identified asymptomatic glucocorticoid- test, or insulin tolerance test. Patients
induced AI in up to 60% of patients in the weeks must be educated on stress-dose coverage
following intraarticular glucocorticoid administration; (sick-day rules).
inflammatory arthropathies and repeated
injections with higher dosages increase this risk.1 HPA-axis recovery in patients with glucocorticoid-
Cases of glucocorticoid-induced AI have been induced AI can take months to years, and 2%
reported after exposure to nasal, intradermal, and to 7% of patients may never recover adrenal
transcutaneous glucocorticoid administration, but function.1 All patients at risk of or with established
short-term administration at the recommended glucocorticoid-induced AI should receive adequate
dosages is generally considered safe. Skin education. Special attention should be paid to:
inflammation, impaired barrier function, the • Increasing patient awareness (including over-
use of occlusive dressings, and the application the-counter drugs containing glucocorticoids
to sites with higher absorption (such as mucous and intraarticular glucocorticoid injections).
membranes, eyelids, and scrotum) affect the rate of
• Measures targeted towards adrenal crisis
systemic absorption with topical use.1
prevention.
In patients at high risk of glucocorticoid-induced
AI, glucocorticoids should not be discontinued until • Sick-day rules (ie, the need to increase the
recovery of the HPA axis is documented. When glucocorticoid replacement dose during
treatment is no longer needed for the underlying illness and other causes of major stress such as
disease in patients taking systemic glucocorticoids, trauma, surgery, childbirth).
the dosage should be gradually tapered to a dose
Strategies to improve clinical outcomes include
of 5 mg prednisolone-equivalent. Early-morning
providing clear information on glucocorticoid
serum cortisol should then be measured to guide
therapy, special situations that may require
management1,4:
adjustment of the glucocorticoid replacement
• Glucocorticoid-induced AI is unlikely in dosage, training on the use of injectable
patients with cortisol values greater than glucocorticoid therapy in emergency situations,
12.7 μg/dL (>350 nmol/L) and glucocorticoids and signs and symptoms of overreplacement and
can usually be discontinued. underreplacement, including those of adrenal
crisis. Patients with glucocorticoid-induced AI
• Glucocorticoid-induced AI is unlikely to be
should receive adequate education regardless of
relevant in basal conditions for patients with
the estimated duration of AI/time to recovery.
cortisol values 10.0 to 12.7 μg/dL (275 to
Patients with glucocorticoid-induced AI should
350 nmol/L). Daily glucocorticoids can be
wear medical alert IDs.
stopped, but the patient should be educated on
stress-dose coverage (sick-day rules).
32 ENDO 2022 • Endocrine Case Management
Clinical Case Vignettes receptor (18 times that of dexamethasone), which
explains the high risk of HPA-axis suppression
Important note: Cortisol cutoffs vary according to the
following repeated systemic absorption. Nonetheless,
cortisol assay used and local practices. The cortisol
cases of glucocorticoid-induced AI have been
values reported in the following cases were measured by
associated with all the medications listed above,
immunoassay (Abbott Alinity). For this immunoassay,
highlighting the importance of closely monitoring
the current cutoff used at our institution for a short
all patients treated with inhaled glucocorticoids.
250-mcg cosyntropin-stimulation test is a 30-minute
Treatment with high dosages of inhaled
cortisol value ≥16.3 μg/dL (≥450 nmol/L).
glucocorticoids put patients at a particularly high risk of
glucocorticoid-induced AI, up to 18.5% (Table 1).1 It
Case 1 is recommended that these patients receive appropriate
A 63-year-old woman is referred by her primary education, including being issued a steroid emergency
care physician because of an early-morning card, and undergo periodic clinical assessment for signs
cortisol value of 1.7 μg/dL (47 nmol/L). She has and symptoms of glucocorticoid-induced AI.5
a 4-month history of unintentional weight loss,
muscle mass loss, and tiredness. Asthma was Table 1. Factors Affecting Risk of Developing
Glucocorticoid-Induced Adrenal Insufficiency in
diagnosed at age 50 years, and she has been on Patients Receiving Inhaled Glucocorticoids
regular inhaled glucocorticoids for many years.
Treatment Treatment with high daily doses of inhaled
dosage glucocorticoids (regardless of treatment
Laboratory test results: duration) increases the risk of glucocorticoid-
induced adrenal insufficiency. High doses are:
ACTH = 6.7 pg/mL (10-60 pg/mL) (SI: 1.47 pmol/L
[2.2-13.2 pmol/L]) • Fluticasone propionate: >500 mcg daily
(adults), >200 mcg daily (children)
Serum cortisol 30 minutes after administration of
250 mcg cosyntropin = 4.0 μg/dL (SI: 110 nmol/L) • Beclometasone dipropionate (standard
particle inhalers): >1000 mcg daily
(adults); >400 mcg daily (children)
Recent pituitary MRI shows normal appearance of
• Beclometasone dipropionate (extra-fine
the hypothalamic-pituitary region. particle inhalers): >400 mcg daily (adults);
>200 mcg daily (children)
Which of the following inhaled • Budesonide: >800 mcg daily (adults);
400 mcg daily (children)
glucocorticoids is most commonly
• Ciclesonide: >320 mcg daily (adults);
associated with adrenal suppression? >160 mcg daily (children)
A. Ciclesonide • Fluticasone furoate: >100 mcg daily (adults)
B. Extra-fine particle beclometasone dipropionate • Mometasone furoate: >400 mcg daily (adults)
C. Fluticasone furoate Treatment Treatment for longer than 6 to 12 months
duration (regardless of the dosage used) increases
D. Fluticasone propionate the risk of glucocorticoid-induced adrenal
insufficiency
Answer: D) Fluticasone propionate Glucocorticoid Fluticasone propionate carries the highest
type risk of causing glucocorticoid-induced
This patient had been treated with inhaled adrenal insufficiency
fluticasone propionate (Answer D) for more than Concomitant • Concomitant use of other exogenous
5 years (initially 250 mcg daily for 2 years, then treatments glucocorticoids (including intermittent
use [eg, rescue glucocorticoids in chronic
500 mcg daily for 2 years, then 1000 mcg daily obstructive pulmonary disease])
for 18 months before being tested). Most cases of • CYP3A4 inhibitors (see Table 3)
glucocorticoid-induced AI in patients using inhaled Other factors • Lower BMI
glucocorticoids have been linked to fluticasone (children)
• Higher adherence to treatment
propionate. This drug has a long half-life (14.4 hours)
Partially based on information from manufactures’ summaries of product
and a strong binding affinity for the glucocorticoid characteristics, Global Initiative for Asthma (2018), British National
Formulary, and British National Formulary for Children.
ENDO 2022 • Adrenal 33
Case 2 Which of the following symptoms
can be found in patients with
An 84-year-old woman is admitted to hospital for
glucocorticoid-induced AI?
COVID-19–related pneumonia. She collapsed at
home and described symptoms of feeling light- A. Easy bruising
headed and fainting with no loss of consciousness; B. Hypoglycemia
she has been having shortness of breath and a C. Fatigue
productive cough for a few days. Secondary AI is D. All of the above
diagnosed during evaluation of hyponatremia:
Answer: D) All of the above
Random cortisol = 1.2 μg/dL (SI: 33 nmol/L)
ACTH = 9 pg/mL (10-60 pg/mL) (SI: 1.98 pmol/L All the signs and symptoms listed can be observed in
[2.2-13.2 pmol/L])
patients with glucocorticoid-induced AI (Answer D).
Cortisol 30 minutes after administration of 250 mcg
cosyntropin = 7.9 μg/dL (SI: 218 nmol/L) A high degree of clinical suspicion is paramount
when there is a history of glucocorticoid exposure.
She has a history of chronic obstructive pulmonary In fact, patients with glucocorticoid-induced
disease treated with beclomethasone dipropionate AI may be asymptomatic, present with varying
inhalers (800 mcg/daily) and steroid injections degrees of symptoms of cortisol deficiency and
into the knee joint every 6 months for the last iatrogenic Cushing syndrome, or present with life-
2 years to treat osteoarthritis. threatening adrenal crisis (Table 2). Importantly,
signs and symptoms of cortisol deficiency and
Cushing syndrome can coexist, depending on
when glucocorticoids are discontinued.
Another element of interest in this case is the
possible cumulative effect of intraarticular and
Table 2. Signs and Symptoms Associated With Glucocorticoid-Induced Adrenal Insufficiency and Adrenal Crisis
Iatrogenic adrenal insufficiency Iatrogenic Cushing syndrome Adrenal crisis
• General malaise • Proximal muscle weakness At least 2 of the following:
• Fatigue • Weight gain and central obesity • Hypotension or hypovolemic
shock
• Weakness • Disproportionate supraclavicular and
dorsocervical fat pads • Nausea or vomiting
• Dizziness (including postural dizziness)
• Facial and upper neck plethora • Severe fatigue
• Gastrointestinal symptoms (nausea, vomiting,
diarrhea, cramps, loss of appetite) • Facial rounding • Fever
• Weight loss • Skin atrophy, easy bruising, red stretch • Impaired consciousness
marks (including lethargy, confusion,
• Hypotension (including postural hypotension)
somnolence, collapse, delirium,
• Acne
• Headaches (usually in the morning) coma, seizures)
• Poor wound healing
• Arthralgias (especially in hand joints)
• Insomnia
• Myalgias
• Increased appetite
• Recurrent respiratory infections with slow recovery
• Irritability, impaired memory, depression
• Alabaster-like, pale skin
• Hypertension
• Poor linear growth (children)
• Abnormal glucose metabolism
• Poor weight gain (children)
• Menstrual irregularities (women)
• Hypoglycemia (more frequent in children)
• Poor linear growth (children)
• Hyponatremia
• Lymphocytosis
• Eosinophilia
34 ENDO 2022 • Endocrine Case Management
inhaled glucocorticoids in causing glucocorticoid- Answer: A) Bolus of intravenous hydrocortisone,
induced AI, emphasizing the potential risks associated 50 to 100 mg, followed by 200 mg hydrocortisone
with their systemic absorption. Patients with infusion/24 h + intravenous fluids
glucocorticoid-induced AI were found to carry a
higher risk of adrenal crisis than patients with other Many cases of adrenal crisis and hospitalization
etiologies of adrenal insufficiency and the most for glucocorticoid-induced AI have been described,
common trigger was infections, as in this case.6,7 A including several deaths.1 In this case, the patient’s
lack of awareness of glucocorticoid-induced AI and random cortisol measurement was less than
the presence of nonspecific signs and symptoms 1 μg/dL (<28 nmol/L) and her ACTH concentration
(which can be mistakenly attributed to other was 6 pg/mL (1.32 pmol/L), suggesting secondary
causes or to the disease for which glucocorticoids AI. This was later confirmed with a short 250-mcg
were prescribed) are likely contributors to the cosyntropin test (30-minute cortisol = 11.9 μg/dL
increased risk of life-threatening adrenal crisis in [329 nmol/L]). It is imperative to remember,
this patient population. however, that diagnostic measures should never
delay treatment (thus, Answer C is incorrect):
a patient with a suspected adrenal crisis must
Case 3 be promptly treated with lifesaving injectable
A 74-year-old woman is admitted to the hydrocortisone and fluid resuscitation.5
emergency department with collapse, vomiting, Continuous intravenous hydrocortisone
diarrhea, and hypotension. infusion should be favored over intermittent bolus
administration in the prevention and treatment of
Laboratory test results: adrenal crisis during major stress (Answer A).8,9
Sodium = 129 mEq/L (136-142 mEq/L) In fact, continuous intravenous hydrocortisone
(SI: 129 mmol/L [136-142 mmol/L]) has been shown to be the only delivery mode that
Plasma osmolality = 272 mOsm/kg (275-295 mOsm/kg) steadily maintains circulating cortisol in the range
(SI: 272 mmol/kg [275-295 mmol/kg]) observed during major stress, while intermittent
Urinary sodium = 107 mEq/L (SI: 107 mmol/L)
bolus administration of hydrocortisone results
She has been receiving steroid injections into the in frequent troughs with lower concentrations,
knee joint every 6 months for the past 2 years thereby potentially exposing patients with adrenal
(methylprednisolone acetate, 40 mg), and she uses insufficiency to periods of underreplacement
a topical steroid (betamethasone valerate, 0.1% (Figure 2) (thus, Answer B is incorrect).9
as required) for psoriasis with psoriatic arthritis.
Adrenal crisis is suspected, and measurement of Case 4
random ACTH and cortisol is requested.
A 63-year-old woman taking oral prednisolone
Which of the following is the for temporal giant-cell arteritis has been tapering
best treatment now? the dosage over a 3-year period, with the aim of
discontinuing treatment. While taking
A. Bolus of intravenous hydrocortisone, 50 to
prednisolone, 4 mg daily, early-morning cortisol is
100 mg, followed by 200 mg hydrocortisone
measured 24 hours after the last dose and is
infusion/24 h + intravenous fluids
documented to be 8.1 μg/dL (223 nmol/L). The
B. Bolus of intravenous hydrocortisone, 100 mg, patient’s regimen is switched from prednisolone to
followed by oral hydrocortisone, 20 mg 3 times twice-daily hydrocortisone (10 mg upon
daily, if the vomiting improves awakening + 5 mg at lunchtime), and a short
C. Intravenous fluids to control the hypovolemia 250-mcg cosyntropin-stimulation test is arranged.
while awaiting ACTH and cortisol results Cortisol 30 minutes after cosyntropin
D. None of the above administration is 14.9 μg/dL (411 nmol/L).
ENDO 2022 • Adrenal 35
Figure 2. Serum Cortisol Following Stress- The borderline result of the cosyntropin-
Dose Hydrocortisone Administration stimulation test suggests that the patient has
and Pharmacokinetic Modeling
sufficient adrenal cortisol reserve in baseline
conditions and is unlikely to require regular daily
glucocorticoid replacement (thus, Answers A and
B are incorrect).10 The patient should be educated
about hydrocortisone administration during major
stress (thus, Answer D is correct and Answer C
is incorrect).
Case 5
A 27-year-old woman has suspected
glucocorticoid-induced AI following multiple
cycles of high-dosage oral dexamethasone
for relapse of Philadelphia-negative acute
lymphoblastic leukemia (fatigue, hypotension,
random cortisol = 4.7 μg/dL [129 nmol/L];
ACTH = <5 pg/mL [<1.10 pmol/L]). She was
discharged from the hospital on twice-daily
hydrocortisone (10 mg + 5 mg) 3 months ago.
The patient also has a history of premature ovarian
insufficiency, and her primary care physician has
recently commenced oral hormone replacement
therapy (estradiol, 2 mg + dydrogesterone,
10 mg). The hematology consultant requested an
Serum cortisol (nmol/L) after hydrocortisone administered orally (Panel
A), intramuscularly (Panel B), as intravenous boluses (Panel C), or as
early-morning cortisol measurement 24 hours
continuous intravenous infusion (Panel D). Linear pharmacokinetic after the last dose of hydrocortisone (13.8 μg/dL
modeling was used to predict the serum cortisol response to initial 50
mg (Panel E) and 100 mg (Panel F) intravenous bolus injections followed [381 nmol/L]) and has referred the patient to the
by continuous intravenous infusion of 200 mg hydrocortisone over 24
hours. Adapted from Prete A et al. J Clin Endocrinol Metab, 2020; 105(7)
endocrinology department for further evaluation.
© Endocrine Society.
Which of the following is the
most appropriate next step?
Which of the following is the best next
step in this patient’s management? A. Reassess adrenal function
A. Continue hydrocortisone and measure early- B. Stop hydrocortisone and discharge
morning cortisol in 3 months C. Stop twice-daily hydrocortisone and advise
B. Restart prednisolone, 4 mg daily, and measure stress dosing only
early-morning cortisol in 3 months D. None of the above
C. Stop twice-daily hydrocortisone and discharge Answer: A) Reassess adrenal function
D. Stop twice-daily hydrocortisone and advise
stress dosing only The patient needs to stop oral hormone
replacement therapy and have her adrenal function
Answer: D) Stop twice-daily hydrocortisone reassessed (Answer A). The estrogen component
and advise stress dosing only increases corticosteroid-binding globulin levels
and consequently total serum cortisol. After
a washout of 6 weeks from oral estradiol, the
36 ENDO 2022 • Endocrine Case Management
result of the short 250-mcg cosyntropin test was, strong CYP3A4 inducers and inhibitors (Table
in fact, abnormal: 3). CYP3A4 is, in fact, the main pathway for the
inactivation of most prescribed glucocorticoids.
Baseline cortisol = 4.1 μg/dL (SI: 113 nmol/L)
30-Minute cortisol = 13.0 μg/dL (SI: 358 nmol/L) Strong CYP3A4 inhibitors (such as antifungal
drugs, clarithromycin, and antiretrovirals) can
The cortisol cutoffs discussed thus far do not increase the systemic exposure to glucocorticoids,
apply to patients treated with high-dosage oral leading to high risk of glucocorticoid-induced
estrogens as part of hormonal contraception AI. Conversely, strong CYP3A4 inducers (such
(oral, intramuscular, or implant), which must be as carbamazepine, phenytoin, rifampicin,
discontinued for at least 6 weeks to allow for an enzalutamide, and mitotane) increase the
unambiguous assessment. Interactions between metabolism of glucocorticoids and can trigger
exogenous glucocorticoids and other drugs are symptoms of cortisol deficiency, including adrenal
potentially relevant for patients at risk of or with crisis, in patients with glucocorticoid-induced AI.
established glucocorticoid-induced AI, chiefly
Table 3. Drug Interactions That Can Affect Patients at Risk of Glucocorticoid-Induced Adrenal Insufficiency
Interaction mechanism with exogenous Possible consequences for patients treated with
Drugs glucocorticoids exogenous glucocorticoids
Oral estrogens Increased corticosteroid-binding globulin levels Increased total serum cortisol levels, which affect the
interpretation of early-morning cortisol and dynamic
testing
CYP3A4 inducers Reduced systemic exposure to synthetic Increased risk of developing signs and symptoms
glucocorticoids of cortisol deficiency (including adrenal crisis) in
patients with underlying glucocorticoid-induced
adrenal insufficiency
CYP3A4 inhibitors Increased systemic exposure to synthetic Increased risk of developing glucocorticoid-induced
glucocorticoids adrenal insufficiency
Megestrol acetate Glucocorticoid activity (binds to the Increased risk of developing glucocorticoid-induced
glucocorticoid receptor) adrenal insufficiency
Medroxyprogesterone acetate Glucocorticoid activity (binds to the Increased risk of developing glucocorticoid-induced
(high dosages) glucocorticoid receptor) adrenal insufficiency
References
1. Prete A, Bancos I. Glucocorticoid induced adrenal insufficiency. BMJ. 7. Hahner S, Spinnler C, Fassnacht M, Burger-Stritt S, Lang K, Milovanovic
2021;374:n1380. PMID: 34253540 D,et al. High incidence of adrenal crisis in educated patients with chronic
2. Bancos I, Hahner S, Tomlinson J, Arlt W. Diagnosis and management of adrenal adrenal insufficiency: a prospective study. J Clin Endocrinol Metab.
insufficiency. Lancet Diabetes Endocrinol. 2015;3(3):216-226. PMID: 25098712 2015;100(2):407-416. PMID: 25419882
3. Broersen LH, Pereira AM, Jorgensen JO, Dekkers OM. Adrenal insufficiency 8. Bornstein SR, Allolio B, Arlt W, et al. Diagnosis and treatment of primary
in corticosteroids use: systematic review and meta-analysis. J Clin Endocrinol adrenal insufficiency: an Endocrine Society clinical practice guideline. J Clin
Metab. 2015;100(6):2171-2180. PMID: 25844620 Endocrinol Metab. 2016;101(2):364-389. PMID: 26760044
4. Woods CP, Argese N, Chapman M, et al. Adrenal suppression in patients taking 9. Prete A, Taylor AE, Bancos I, et al. Prevention of adrenal crisis: cortisol
inhaled glucocorticoids is highly prevalent and management can be guided by responses to major stress compared to stress dose hydrocortisone delivery. J
morning cortisol. Eur J Endocrinol. 2015;173(5):633-642. PMID: 26294794 Clin Endocrinol Metab. 2020;105(7):2262-2274. PMID: 32170323
5. Simpson H, Tomlinson J, Wass J, Dean J, Arlt W. Guidance for the 10. Agha A, Tomlinson JW, Clark PM, Holder G, Stewart PM. The long-term
prevention and emergency management of adult patients with adrenal predictive accuracy of the short synacthen (corticotropin) stimulation test for
insufficiency. Clin Med (Lond). 2020;20(4):371-378. PMID: 32675141 assessment of the hypothalamic-pituitary-adrenal axis. J Clin Endocrinol Metab.
6. Li D, Genere N, Behnken E, et al. Determinants of self-reported health 2006;91(1):43-47. PMID: 16249286
outcomes in adrenal insufficiency: a multisite survey study. J Clin Endocrinol
Metab. 2021;106(3):e1408-e1419. PMID: 32995875
ENDO 2022 • Adrenal 37
Advances in the
Treatment of Congenital
Adrenal Hyperplasia
Adina F. Turcu, MD, MS. Division of Metabolism, Endocrinology, and Diabetes, University of
Michigan, Ann Arbor, MI; E-mail:
[email protected]Learning Objectives and continuous subcutaneous hydrocortisone
pumps; targeting ACTH excess via CRH receptor
As a result of participating in this session, learners
antagonists, ACTH antibodies, or ACTH receptor
should be able to:
antagonists; and inhibiting enzymes required for
• Describe established and upcoming therapies androgen synthesis.
for patients with classic and nonclassic In parallel with novel therapeutic developments,
congenital adrenal hyperplasia (CAH). growing evidence supports the use of
11-oxyandrogens as indicators of an adrenal vs
• Implement use of traditional and novel
gonadal source, with advantages over traditional
biomarkers for monitoring disease control and
androgens in guiding the management of patients
guiding therapy for patients with classic and
with CAH.
nonclassic CAH.
Significance of the
Main Conclusions Clinical Problem
Treatment of CAH focuses on 2 main aspects: (1) CAH is due to autosomal recessive pathogenic
hormonal replacement for adrenal insufficiency variants in genes encoding enzymes required for
in classic CAH and (2) suppression of adrenal cortisol synthesis. Defects in the gene encoding
androgen excess whenever clinically relevant. steroid 21-hydroxylase (CYP21A2) account for
Since they became available in the 1950s, most CAH cases.1 Severe forms of 21-hydroxylase
glucocorticoids have been the primary tool to deficiency (21-OHD) result in clinically overt
address both CAH treatment goals, with only a adrenal insufficiency and are termed “classic.”
few available preparations. While sufficient for the Milder enzymatic defects, in which both cortisol
treatment of adrenal insufficiency, glucocorticoid and aldosterone synthesis are compensated, are
dosages and timing of administration required termed “nonclassic.” While classic 21-OHD affects
for adrenal androgen suppression are linked to approximately 1 in 15,000 newborns,2 nonclassic
numerous off-target effects. Recent developments 21-OHD is relatively common, occurring in
to advance treatment of patients with CAH have approximately 1 in 1000 White individuals and in
focused on 2 fronts: (1) glucocorticoid delivery up to 1 in 30 individuals of certain ethnicities (eg,
that closely mimics the physiologic circadian Ashkenazi Jewish, Hispanic, and Mediterranean).3
rhythm and (2) nonglucocorticoid strategies to In all cases of 21-OHD, obstructed cortisol
overcome adrenal androgen excess. Examples synthesis prompts ACTH elevations, which,
include modified-release oral glucocorticoids in turn, stimulate adrenal cortical growth and
38 ENDO 2022 • Endocrine Case Management
steroidogenic flux. The combination of ACTH adrenal androgen excess leads to adverse
elevation and 21-OHD favors the overproduction effects typical of iatrogenic Cushing syndrome.
of adrenal androgens. Clinical manifestations of • Lack of reliable biomarkers to guide therapy.
adrenal androgen excess include virilization of
external genitalia in girls with classic 21-OHD,
premature pubarche, rapid somatic growth,
Strategies for Diagnosis,
advanced bone age, hirsutism, acne, irregular
menses, and subfertility.4-7 Sustained ACTH Therapy, and/or Management
elevations also promote the expansion of adrenal Steroid Therapy
rest tissue, such as testicular or ovarian adrenal
In patients with classic 21-OHD, oral
rest tumors.
glucocorticoid regimens aim to distribute
The 2 main treatment goals in 21-OHD are: (1)
diurnal replacement doses, as in any form of
to replace insufficient hormones (glucocorticoids
adrenal insufficiency. For this purpose, short-
and mineralocorticoids, when needed) and (2)
acting hydrocortisone, administered 2 or 3 times
to suppress excessive production of adrenal
daily, offers the lowest risk of glucocorticoid-
androgens. Since their first use in the 1950s, oral
induced adverse effects, and it is preferred
glucocorticoids have been the mainstay of 21-
in children (Table 1). A morning dose of an
OHD therapy. In addition to the typical daytime
intermediate-acting glucocorticoid (prednisone
replacement dosing common to all forms of
or prednisolone), or even a long-acting
adrenal insufficiency, bedtime glucocorticoids are
glucocorticoid (dexamethasone), can be considered
used in patients with 21-OHD to reduce the early-
in patients who prefer to minimize medication
morning rise in ACTH and the consequent adrenal
frequency, or when adherence is challenging. To
androgen excess. Supraphysiologic administration
counteract adrenal androgen excess, a low dose of
of glucocorticoids leads to obesity, dyslipidemia,
intermediate- or long-acting glucocorticoid can be
hyperglycemia, and bone loss.8-13
administered at bedtime.
Another important challenge in managing
21-OHD is the lack of reliable biomarkers to
Table 1. Available Oral Glucocorticoids
guide glucocorticoid dosing. Normalization
of 17α-hydroxyprogesterone can only be Average adrenal Androgen
insufficiency suppression
achieved with excessive glucocorticoid doses Relative replacement bedtime
and consequential iatrogenic Cushing syndrome, Glucocorticoid potency doses doses
which is not a reasonable tradeoff. Moreover, Hydrocortisone 1 15 to 20 mg daily *Not used
(divided in 2 to for this
17α-hydroxyprogesterone, androstenedione, and 3 doses) purpose
testosterone correlate poorly with clinical evidence due to short
half-life
of androgen excess,14,15 in part due to their dual
Prednisolone 4 5 mg in AM 1 to 3 mg
adrenal and gonadal origin. DHEA-S, the major
adrenal androgen precursor, is often paradoxically Prednisone 4 5 mg in AM 1 to 3 mg
low in patients with poorly controlled 21-OHD. Dexamethasone 30 1 mg in AM ≤1 mg
To closely mimic physiologic glucocorticoid
Barriers to Optimal Practice synthesis, and to minimize the adverse effects
of supraphysiologic doses, modified-release
• Limited treatment options beyond glucocorticoid formulations have been developed.
glucocorticoids. Plenadren (Shire Services BVBA, Belgium) is
• Nonphysiologic administration of a dual-release hydrocortisone tablet with an
glucocorticoids to suppress ACTH-driven outer immediate-release coating and an inner
sustained-release core approved in Europe for
ENDO 2022 • Adrenal 39
once-daily administration in patients with adrenal not be used without reliable contraceptive methods
insufficiency, but its use in patients with 21- due to feminization effects on male fetuses.
OHD does not achieve good control of androgen
excess.16 Chronocort (Diurnal Europe B.V., Emerging Nonglucocorticoid
The Netherlands) is a multiparticulate delayed-
release hydrocortisone formulation that provides Therapies
sustained drug delivery and is particularly useful Several therapeutic strategies that target adrenal
for suppressing the early-morning ACTH rise in androgen excess are being developed (Table 2).
patients with 21-OHD.17 Two oral small-molecule inhibitors of the pituitary
Further fine-tuning the delivery of corticotropin-releasing factor type 1 (CRF1)
glucocorticoids could theoretically be achieved receptors are currently in clinical trials:
with continuous intravenous or subcutaneous crinecerfont (Neurocrine Biosciences, Inc, USA)
delivery systems, but data are limited to small and tildacerfont (Spruce Biosciences, USA). By
studies. Like insulin pumps, such systems involve interrupting the signaling communication
relatively complex operational logistics and add between the hypothalamus and pituitary, these
risks, including malfunction and site infections.18,19 agents offer an alternative to bedtime
Patients with nonclassic 21-OHD have glucocorticoids to dampen the morning ACTH
compensated cortisol production and do not need rise. Other strategies to reduce ACTH elevation
replacement therapy. In women with nonclassic include antiACTH monoclonal antibodies21 and
21-OHD who are not interested in conceiving, ACTH receptor antagonists,22 but both are
oral contraceptives help reduce ovarian androgen experimental therapies and have not yet been
production and increase SHBG, which lowers tested in humans.
the fraction of bioavailable and free testosterone. Abiraterone acetate is a potent inhibitor
Spironolactone, primarily a mineralocorticoid of CYP17A1, an enzyme needed for both
receptor antagonist, is also an androgen receptor glucocorticoid and sex-hormone synthesis, and
antagonist at higher dosages, and it can be it is expressed in the adrenal glands and gonads.
added to further treat hirsutism and/or acne. Abiraterone acetate effectively reduces androgen
Spironolactone crosses the placenta, and it should synthesis and has been shown to decrease
mortality in men with castration-resistant prostate
Table 2. Novel Treatment Strategies in CAH
Treatment Administration Advantages Disadvantages
Glucocorticoids Modified-release Twice daily • Oral administration • Potential for iatrogenic
hydrocortisone Cushing syndrome
• Early-morning ACTH
(Plenadren,
suppression
Chronocort)
Subcutaneous Continuous • Close replication of circadian • Continuous device wear
hydrocortisone secretory patterns for ACTH
• Local irritation
infusion and cortisol
• Potential for malfunctioning
• Potential for ultradian pulsatility
replication if a real-time ACTH/ • High fixed costs and
cortisol monitoring is added complexity
Androgen biosynthetic CYP17A1 inhibitors Once daily • Highly effective • Does not lower ACTH
enzyme blockage (abiraterone
• Well-tolerated • Gonadal sex-steroid
acetate)
inhibition
CRH receptor type 1 Crinecerfont, Once daily • Reduce ACTH secretion • Long-term safety data are
antagonists tildacerfont lacking
Reprinted from Turcu AF & Auchus RJ. Curr Opin in Endocrinol Diabetes Obes, 2016; 23(3) © Wolters Kluwer Health, Inc. All rights reserved.
40 ENDO 2022 • Endocrine Case Management
cancer.23-25 In a study of 6 women with poorly More than testosterone, patients with 21-OHD
controlled classic 21-OHD, abiraterone acetate overproduce a set of androgens unique to the
normalized androstenedione when added to adrenal gland, called 11-oxygenated C19 steroids, or
physiologic hydrocortisone and fludrocortisone.26 11-oxyandrogens. Of these, 11β-androstenedione
Because it inhibits the production of gonadal sex is the most abundant, and its downstream
steroids, abiraterone acetate is not a good option metabolite 11-ketotestosterone is a potent
for reproductive-aged patients. Also, by posing androgen, bioequivalent to testosterone.30,31
a second block in the biosynthetic pathway of Higher serum concentrations of 11-oxyandrogens
cortisol, this agent is not sensible in patients with are associated with several clinical indicators of
nonclassic 21-OHD. suboptimal 21-OHD control, including larger
All therapies discussed above are adjunctive to adrenal glands, presence of testicular adrenal rest
glucocorticoid replacement. Future development tumors, and menstrual irregularities.32
of gene-based therapy27,28 has the potential to Serum 11-ketotestosterone assays are now
cure 21-OHD and fully eliminate the need for offered by some commercial laboratories, and its
glucocorticoids. measurement can be useful in cases with discrepant
traditional biomarkers. For example, in men
with 21-OHD, 11-ketotestosterone correlates
Monitoring Therapy
inversely with testosterone, and the former can
In women with 21-OHD, normal concentrations be high when testosterone is normal. In female
of testosterone and androstenedione typically patients with 21-OHD, 11-ketotestosterone and
indicate good disease control, although these testosterone are positively correlated, as they are
steroids correlate poorly with clinical stigmata of both produced primarily in the adrenal gland. In
hyperandrogenism. Disproportionately elevated patients with either classic and nonclassic 21-OHD,
androstenedione suggests suboptimally controlled 11-oxyandrogens are disproportionately higher
21-OHD. Elevated testosterone, however, can be than conventional androgens.33,34 In contrast,
seen in women with 21-OHD and concomitant androstenedione and testosterone concentrations
polycystic ovary syndrome. Excessive adrenal- are similar in patients with nonclassic 21-OHD and
derived progesterone contributes to irregular in those with hyperandrogenism of other etiologies
menses and infertility. In preparation for with similar clinical presentations, including
conceiving, the goal of glucocorticoid therapy is polycystic ovary syndrome.34
to suppress serum progesterone to concentrations
less than 0.6 ng/mL (<2.0 nmol/L), which is
associated with higher fertility rates in women Clinical Case Vignettes
with classic 21-OHD.29 Case 1
In men with 21-OHD, serum testosterone
A 29-year-old woman with nonclassic 21-OHD
concentrations have little value when used in
presents for routine follow-up. Menarche was at age
isolation. In healthy men, testosterone is produced
10 years. She struggles with irregular menses, acne,
primarily by the testes; conversely, uncontrolled
and facial hirsutism. All of her symptoms improved
21-OHD results in larger proportions of adrenal
while taking oral contraceptives. However, she
testosterone. An androstenedione-to-testosterone
stopped oral contraceptives approximately 1 year
ratio greater than 0.5 is suggestive of adrenal
ago when she got married and started trying to
androgen excess. Excessive amounts of adrenal
conceive. The couple met with a genetic counselor,
androgens lead to gonadotropin suppression.
and the husband’s genetic testing shows no
Normal serum testosterone concentrations along
pathogenic CYP21A2 variants. The patient’s only
with suppressed gonadotropins in men with
medication is a prenatal multivitamin.
21-OHD suggest abundant adrenal androgen
production and low gonadal testosterone.
ENDO 2022 • Adrenal 41
Laboratory test results (obtained 7 days after her a low dosage of prednisolone at bedtime (Answer
last menses): D). Prednisolone is preferred over its precursor,
prednisone, because the in vivo activation of
Testosterone = 81 ng/dL (8-60 ng/dL)
(SI: 2.8 nmol/L [0.3-2.1 nmol/L]) prednisone at such low dosages is inconsistent.
Androstenedione = 250 ng/dL (30-200 ng/dL) Studies suggest that hydrocortisone once or
(SI: 8.7 nmol/L [1.05-6.98 nmol/L]) twice daily is also associated with a decreased
DHEA-S = 90 µg/dL (44-333 µg/dL) (SI: 2.44 µmol/L risk of miscarriages in women with nonclassic
[1.19-9.00 µmol/L])
21-OHD.35 However, the mechanism of day-time
Progesterone = 2.0 ng/mL (≤1.0 ng/mL [follicular];
2.0-20.0 ng/mL [luteal]) (SI: 6.36 nmol/L hydrocortisone use in these patients is poorly
[≤3.2 nmol/L (follicular); 6.4-63.6 nmol/L understood. Because hydrocortisone is short acting
(luteal)]) (half-life of approximately 1.5 hours), the potential
17-Hydroxyprogesterone = 1230 ng/dL (<80 ng/dL of bedtime dosing (Answer C) to suppress early-
[follicular]) (SI: 37.3 nmol/L [<2.42 nmol/L])
morning ACTH is negligible.
Serum cortisol = 11.6 µg/dL (5-25 µg/dL)
(SI: 320.0 nmol/L [137.9-689.7 nmol/L]) Bedtime dexamethasone (Answer A) is
ACTH = 61 pg/mL (10-60 pg/mL) (SI: 13.4 pmol/L effective in suppressing the early-morning
[2.2-13.2 pmol/L]) ACTH rise, although it has a higher risk of
off-target adverse effects from nonphysiologic
glucocorticoid exposure. In addition, unlike
Which of the following treatment options
prednisone, prednisolone, and hydrocortisone,
should be recommended for this patient?
dexamethasone is not inactivated by the placental
A. Dexamethasone, 0.25 mg at bedtime 11β-hydroxysteroid dehydrogenase type 2, and
B. Flutamide, 250 mg daily it can suppress the fetal hypothalamic-pituitary-
C. Hydrocortisone, 5 mg at bedtime adrenal axis. Dexamethasone has been used in
D. Prednisolone, 2 mg at bedtime the first trimester of pregnancy in women at
risk for having a baby girl with classic 21-OHD
E. Spironolactone, 50 mg daily
to prevent virilization. Prenatal treatment with
Answer: D) Prednisolone, 2 mg at bedtime dexamethasone is controversial, as the maternal
and fetal risks are considered by many to outweigh
Patients with nonclassic 21-OHD have the benefits.36 In this vignette, the patient’s partner
normal production of glucocorticoids and had no identified CYP21A2 pathogenic variants, so
mineralocorticoids and do not need replacement prenatal treatment with dexamethasone does not
therapy for these hormones. However, to deserve consideration.
maintain normal cortisol synthesis, elevated Spironolactone (Answer E) is a mineral
ACTH levels are needed to overcome the defective ocorticoid receptor antagonist, and, at high
21-hydroxylase. The combination of elevated dosages, is also an androgen receptor antagonist.
ACTH and 21-hydroxylase insufficiency result Spironolactone can be used in women with
in overproduction of adrenal progesterone and hyperandrogenism of various causes. However,
17α-hydroxyprogesterone, and their diversion spironolactone crosses the placenta and can cause
to formation of adrenal androgens. Limited data feminization of a male fetus. Hence, in reproductive-
suggest that the rates of subfertility are modest aged women, spironolactone must be used in
in women with nonclassic 21-OHD, but the risk conjunction with a reliable contraceptive method.
of early miscarriages is lower in women taking Flutamide (Answer B) is a nonsteroidal
glucocorticoids than in those who do not.35 The androgen receptor antagonist that has been
goal of therapy is to suppress the early rise of used in castration-resistant prostate cancer. Its
ACTH to overcome adrenal progesterone and adverse effects include hepatotoxicity, and it is not
androgen excess. This goal can be achieved with considered safe during pregnancy.
42 ENDO 2022 • Endocrine Case Management
Case 2 women with classic 21-OHD, abiraterone acetate
administered once daily along with hydrocortisone
A 61-year-old woman with classic congenital
replacement decreased circulating androgen
adrenal hyperplasia due to 21-OHD presents to
levels.26 As patients with classic 21-OHD already
establish care after recently relocating. Her medical
require full replacement doses of glucocorticoids,
history also includes type 2 diabetes mellitus,
the concern about interference with cortisol
osteoporosis, dyslipidemia, obesity, obstructive
synthesis by abiraterone is irrelevant. Abiraterone
sleep apnea, anxiety, and insomnia. She currently
also blocks the synthesis of gonadal sex steroids,
takes hydrocortisone, 10 mg in the morning
but this concern is limited to reproductive-aged
and 5 mg at 2 PM, and dexamethasone, 0.25 mg
women and not to postmenopausal women.
at bedtime. She is particularly concerned about
Preliminary data from clinical trials show that
diffuse alopecia and facial hirsutism.
CRF-1 receptor antagonists (Answer B) effectively
Laboratory test results: reduce ACTH and 17-hydroxyprogesterone in
patients with uncontrolled classic 21-OHD.37
ACTH = 190 pg/mL (10-60 pg/mL) (SI: 41.8 pmol/L
[2.2-13.2 pmol/L])
By directly blocking ACTH production, such
17-Hydroxyprogesterone = 4210 ng/dL nonsteroidal oral CRF-1 receptor antagonists
(<51 ng/dL [postmenopausal]) (SI: 127.6 nmol/L offer promising therapeutic advantages over
[<1.55 nmol/L]) supraphysiologic doses and times of administration
Testosterone = 92 ng/dL (8-60 ng/dL) of glucocorticoids.
(SI: 3.2 nmol/L [0.3-2.1 nmol/L])
DHEA-S = <15 μg/dL (15-157 μg/dL)
In this patient, increasing the bedtime dose of
(SI: <0.41 μmol/L [0.41-4.25 μmol/L]) dexamethasone (Answer C) further enhances the
risk of adverse effects, many of which the patient
already experiences (worsening hyperglycemia,
Which of the following treatment osteoporosis, sleep disturbances).
options should be considered next? Spironolactone (Answer A) is a mineral
A. Add spironolactone, 25 mg daily ocorticoid and androgen receptor inhibitor. Adding
B. Enroll in a clinical trial of a corticotropin- spironolactone to the regimen of a patient with
releasing factor type 1 (CRF-1) receptor androgen excess is a reasonable option, but the
antagonist antiandrogenic effects are relatively modest and
C. Increase bedtime dexamethasone dosage to begin to occur at dosages of at least 100 mg daily.38
0.5 mg nightly
D. Stop dexamethasone and add abiraterone Case 3
acetate A 34-year-old man with classic CAH due to
E. B or D 21-OHD is seen for infertility. He and his wife
have been trying to conceive for about a year. He
Answer: E) Enroll in a clinical trial of a corticotropin- has small bilateral testicular adrenal rest tumors,
releasing factor type 1 (CRF-1) receptor antagonist (B) which have been stable according to recent
or stop dexamethasone and add abiraterone acetate (D) ultrasonography.
Abiraterone (Answer D) is a potent inhibitor His medications include hydrocortisone,
of CYP17A1 (17α-hydroxylase/17,20-lyase), an 10 mg in the morning and 10 mg at 2 PM, and
enzyme required for both cortisol and androgen prednisolone, 2 mg at bedtime.
synthesis. Abiraterone acetate improves survival in On physical examination, the patient is
patients with castration-resistant prostate cancer,23 well virilized. His blood pressure is 102/75 mm Hg,
and it was approved by the US FDA for this and pulse rate is 90 beats/min. His height is
indication in 2018. In a small, open-label study of 68 in (173 cm), and weight is 192 lb (87.1)
ENDO 2022 • Adrenal 43
(BMI = 29.2 kg/m2). Testicular volume is and by the adrenal glands. In men with 21-OHD,
approximately 26 mL bilaterally, and testes are a testosterone-to-androstenedione ratio less than
heterogeneous and firm. 2 is suggestive of suboptimal disease control.
11α-Hydroxyandrostenedione and its bioactive
Laboratory test results: peripheral metabolite, 11-ketotestosterone,
Serum total testosterone = 393 ng/dL are produced in large amounts in patients with
(300-900 ng/dL) (SI: 13.6 nmol/L uncontrolled 21-OHD. 11-Ketotestosterone
[10.4-31.2 nmol/L]) correlates inversely with testosterone in men with
LH = <0.20 mIU/mL (1.0-9.0 mIU/mL)
classic 21-OHD—the excess adrenal androgens
(SI: <0.20 IU/L [1.0-9.0 IU/L])
FSH = <0.20 mIU/mL (1.0-13.0 mIU/mL) suppress gonadotropins and, consequently,
(SI: <0.20 IU/L [1.0-13.0 IU/L]) gonadal testosterone. Thus, the best next step is to
ACTH = 135 pg/mL (10-60 pg/mL) measure gonadotropins and 11-ketotestosterone
(SI: 29.7 pmol/L [2.2-13.2 pmol/L]) (Answer A). While in healthy reproductive-
17-Hydroxyprogesterone = 4100 ng/dL (<220 ng/dL)
aged men, the adrenal contribution to the pool
(SI: 124.2 nmol/L [<6.67 nmol/L])
Androstenedione = 2200 ng/dL (65-210 ng/dL) of circulating testosterone is negligible, the
(SI: 76.8 nmol/L [2.27-7.33 nmol/L]) adrenal component might be significant in men
DHEA-S = 66 µg/dL (65-334 µg/dL) (SI: 1.79 μmol/L with uncontrolled CAH, as in this case, making
[1.76-9.05 μmol/L]) testosterone an unreliable biomarker (thus,
Answer C is incorrect).
Which of the following is the best next Serum 17-hydroxyprogesterone (Answer
step in this patient’s management? B) is useful for the diagnosis of CAH, but its
role in monitoring therapy is limited. A serum
A. Measure gonadotropins and
17-hydroxyprogesterone value in the normal range
11-ketotestosterone
typically indicates glucocorticoid overtreatment
B. Measure serum 17-hydroxyprogesterone and risk for associated adverse effects.39
C. Recommend fertility evaluation for the Testicular adrenal rest tumors develop in up
patient’s wife, as his testosterone concentration to 50% of adolescent boys and men with poorly
is normal controlled classic 21-OHD.13 These tumors
D. Refer for surgery for testicular adrenal rest contribute to male infertility via mass effect, Sertoli-
tumor removal cell and cell damage, and blood flow interruptions.
Intensified glucocorticoid treatment can be helpful
Answer: A) Measure gonadotropins in reducing the size of testicular adrenal rest tumors
and 11-ketotestosterone and restoring fertility, particularly if initiated before
Although this man with classic 21-OHD has a permanent fibrosis develops. Surgical removal
normal testosterone concentration, he most likely can be offered to alleviate pain if intensified
has hypogonadotropic hypogonadism due to glucocorticoid therapy fails, but restoration of
suppression of the hypothalamic-pituitary-gonadal fertility is uncommon.40 This patient had small and
axis by adrenal androgen excess. Testosterone stable testicular adrenal rest tumors, so surgical
and androstenedione are secreted by the gonads removal (Answer D) is not indicated.
References
1. Speiser PW, White PC. Congenital adrenal hyperplasia. N Engl J Med. 3. Speiser PW, Dupont B, Rubinstein P, Piazza A, Kastelan A, New MI. High
2003;349(8):776-788. PMID: 12930931 frequency of nonclassical steroid 21-hydroxylase deficiency. Am J Hum Genet.
2. Therrell BL Jr, Berenbaum SA, Manter-Kapanke V, et al. Results of screening 1985;37(4):650-667. PMID: 9556656
1.9 million Texas newborns for 21-hydroxylase-deficient congenital adrenal
hyperplasia. Pediatrics. 1998;101(4 Pt 1):583-590. PMID: 9521938
44 ENDO 2022 • Endocrine Case Management
4. Stikkelbroeck NM, Hermus ARMM, Braat DDM, Otten BJ. Fertility in 23. de Bono JS, Logothetis CJ, Molina A, et al. Abiraterone and increased survival
women with congenital adrenal hyperplasia due to 21-hydroxylase deficiency. in metastatic prostate cancer. N Engl J Med. 2011;364(21):1995-2005. PMID:
Obstet Gynecol Surv. 2003;58(4):275-284. PMID: 12665708 21612468
5. Cabrera MS, Vogiatzi MG, New MI. Long term outcome in adult males 24. Ryan CJ, Smith MR, de Bono JS, et al. Abiraterone in metastatic prostate
with classic congenital adrenal hyperplasia. J Clin Endocrinol Metab. cancer without previous chemotherapy. N Engl J Med. 2013;368(2):138-148.
2001;86(7):3070-3078. PMID: 11443169 PMID: 23228172
6. Reisch N, Flade L, Scherr M, et al. High prevalence of reduced fecundity 25. Ryan CJ, Smith MR, Fizazi K, et al. Abiraterone acetate plus prednisone
in men with congenital adrenal hyperplasia. J Clin Endocrinol Metab. versus placebo plus prednisone in chemotherapy-naive men with metastatic
2009;94(5):1665-1670. PMID: 19258407 castration-resistant prostate cancer (COU-AA-302): final overall survival
7. Claahsen-van der Grinten HL, Otten BJ, Hermus ARMM, Sweep FCGJ, analysis of a randomised, double-blind, placebo-controlled phase 3 study.
Hulsbergen-van de Kaa CA. Testicular adrenal rest tumors in patients with Lancet Oncol. 2015;16(2):152-160. PMID: 25601341
congenital adrenal hyperplasia can cause severe testicular damage. Fertil Steril. 26. Auchus RJ, Buschur EO, Chang AY, et al. Abiraterone acetate to lower
2008;89(3):597-601. PMID: 17543962 androgens in women with classic 21-hydroxylase deficiency. J Clin Endocrinol
8. Horrocks PM, London DR. Effects of long term dexamethasone treatment Metab. 2014;99(8):2763-2770. PMID: 24780050
in adult patients with congenital adrenal hyperplasia. Clin Endocrinol (Oxf). 27. Perdomini M, Dos Santos C, Goumeaux C, Blouin V, Bougneres P.
1987;27(6):635-642. PMID: 2843311 An AAVrh10-CAG-CYP21-HA vector allows persistent correction
9. Young MC, Hughes IA. Dexamethasone treatment for congenital adrenal of 21-hydroxylase deficiency in a Cyp21(-/-) mouse model. Gene Ther.
hyperplasia. Arch Dis Child. 1990;65(3):312-314. PMID: 2334212 2017;24(5):275-281. PMID: 28165447
10. El-Maouche D, Collier S, Prasad M, Reynolds JC, Merke DP. Cortical bone 28. Naiki Y, Miyado M, Horikawa R, et al. Extra-adrenal induction of Cyp21a1
mineral density in patients with congenital adrenal hyperplasia due to ameliorates systemic steroid metabolism in a mouse model of congenital
21-hydroxylase deficiency. Clin Endocrinol (Oxf). 2014. PMID: 24862755 adrenal hyperplasia. Endocr J. 2016;63(10):897-904. PMID: 27432820
11. Falhammar H, Filipsson Nystrom H, Wedell A, Brismar K, Thoren M. Bone 29. Casteras A, De Silva P, Rumsby G, Conway GS. Reassessing fecundity
mineral density, bone markers, and fractures in adult males with congenital in women with classical congenital adrenal hyperplasia (CAH):
adrenal hyperplasia. Eur J Endocrinol. 2013;168(3):331-341. PMID: 23211577 normal pregnancy rate but reduced fertility rate. Clin Endocrinol (Oxf).
12. Arlt W, Willis DS, Wild SH, et al; United Kingdom Congenital Adrenal 2009;70(6):833-837. PMID: 19250265
Hyperplasia Adult Study Executive (CaHASE). Health status of adults with 30. Rege J, Turcu AF, Kasa-Vubu JZ, et al. 11-Ketotestosterone is the dominant
congenital adrenal hyperplasia: a cohort study of 203 patients. J Clin Endocrinol circulating bioactive androgen during normal and premature adrenarche. J
Metab. 2010;95(11):5110-5121. PMID: 20719839 Clin Endocrinol Metab. 2018;103(12):4589-4598. PMID: 30137510
13. Finkielstain GP, Kim MS, Sinaii N, et al. Clinical characteristics of a cohort 31. Pretorius E, Africander DJ, Vlok M, Perkins MS, Quanson J, Storbeck KH.
of 244 patients with congenital adrenal hyperplasia. J Clin Endocrinol Metab. 11-Ketotestosterone and 11-ketodihydrotestosterone in castration resistant
2012;97(12):4429-4438. PMID: 22990093 prostate cancer: potent androgens which can no longer be ignored. PloS One.
14. Speiser PW, Dupont J, Zhu D, et al. Disease expression and molecular 2016;11(7):e0159867. PMID: 27442248
genotype in congenital adrenal hyperplasia due to 21-hydroxylase deficiency. J 32. Turcu AF, Mallappa A, Elman MS, et al. 11-Oxygenated androgens
Clin Invest. 1992;90(2):584-595. PMID: 1644925 are biomarkers of adrenal volume and testicular adrenal rest tumors in
15. Krone N, Braun A, Roscher AA, Knorr D, Schwarz HP. Predicting phenotype 21-hydroxylase deficiency. J Clin Endocrinol Metab. 2017;102(8):2701-2710.
in steroid 21-hydroxylase deficiency? Comprehensive genotyping in 155 PMID: 28472487
unrelated, well defined patients from southern Germany. J Clin Endocrinol 33. Turcu AF, Nanba AT, Chomic R, et al. Adrenal-derived 11-oxygenated
Metab. 2000;85(3):1059-1065. PMID: 10720040 19-carbon steroids are the dominant androgens in classic 21-hydroxylase
16. Quinkler M, Miodini Nilsen R, Zopf K, Ventz M, Oksnes M. Modified- deficiency. Eur J Endocrinol. 2016;174(5):601-609. PMID: 26865584
release hydrocortisone decreases BMI and HbA1c in patients with primary 34. Turcu AF, El-Maouche D, Zhao L, et al. Androgen excess and diagnostic
and secondary adrenal insufficiency. Eur J Endocrinol. 2015;172(5):619-626. steroid biomarkers for nonclassic 21-hydroxylase deficiency without
PMID: 25656494 cosyntropin stimulation. Eur J Endocrinol. 2020;183(1):63-71. PMID:
17. Mallappa A, Sinaii N, Kumar P, et al. A phase 2 study of Chronocort, 32487778
a modified-release formulation of hydrocortisone, in the treatment of 35. Bidet M, Bellanne-Chantelot C, Galand-Portier MB, et al. Fertility in women
adults with classic congenital adrenal hyperplasia. J Clin Endocrinol Metab. with nonclassical congenital adrenal hyperplasia due to 21-hydroxylase
2015;100(3):1137-1145. PMID: 25494662 deficiency. J Clin Endocrinol Metab. 2010;95(3):1182-1190.
18. Merza Z, Rostami-Hodjegan A, Memmott A, et al. Circadian hydrocortisone 36. Miller WL, Witchel SF. Prenatal treatment of congenital adrenal hyperplasia:
infusions in patients with adrenal insufficiency and congenital adrenal risks outweigh benefits. Am J Obstet Gynecol. 2013;208(5):354-359. PMID:
hyperplasia. Clin Endocrinol (Oxf). 2006;65(1):45-50. PMID: 16817818 23123167
19. Nella AA, Mallappa A, Perritt AF, et al. A phase 2 study of continuous 37. Auchus RJ, Sarafoglou K, Fechner PY, et al. Crinecerfont lowers elevated
subcutaneous hydrocortisone infusion in adults with congenital adrenal hormone markers in adults with 21-hydroxylase deficiency congenital adrenal
hyperplasia. J Clin Endocrinol Metab. 2016;101(12):4690-4698. PMID: hyperplasia. J Clin Endocrinol Metab. 2022;107(3):801-812. PMID: 34653252
27680873 38. Cumming DC. Use of spironolactone in treatment of hirsutism. Cleve Clin J
20. Turcu AF, Auchus RJ. Novel treatment strategies in congenital adrenal Med. 1990;57(3):285-287. PMID: 2357784
hyperplasia. Curr Opin Endocrinol Diabetes Obes. 2016;23(3):225-232. PMID: 39. Silva IN, Kater CE, Cunha CF, Viana MB. Randomised controlled trial of
27032061 growth effect of hydrocortisone in congenital adrenal hyperplasia. Arch Dis
21. Gehrand AL, Phillips J, Malott K, Raff H. A long-acting neutralizing Child. 1997;77(3):214-218. PMID: 9370898
monoclonal ACTH antibody blocks corticosterone and adrenal gene 40. Claahsen-van der Grinten HL, Otten BJ, Takahashi S, et al. Testicular adrenal
responses in neonatal rats. Endocrinology. 2019;160(7):1719-1730. PMID: rest tumors in adult males with congenital adrenal hyperplasia: evaluation
31166572 of pituitary-gonadal function before and after successful testis-sparing
22. Sanders K, Mol JA, Kooistra HS, Galac S. Melanocortin 2 receptor surgery in eight patients. J Clin Endocrinol Metab. 2007;92(2):612-615. PMID:
antagonists in canine pituitary-dependent hypercortisolism: in vitro studies. 17090637
Vet Res Commun. 2018;42(4):283-288. PMID: 30187173
ENDO 2022 • Adrenal 45
Hypercortisolism: A
Challenging Disease to
Diagnose and Manage
Ricardo R. Correa, MD, EdD. Division of Endocrinology, Diabetes, and Metabolism,
Department of Medicine, University of Arizona College of Medicine Phoenix and Phoenix
VAMC, Phoenix, AZ; E-mail:
[email protected]Katherine A. Araque, MD, MSCR. University of Arizona College of Medicine, Phoenix, AZ;
E-mail: [email protected]
Learning Objectives the ability of clinicians to confidently recommend
next steps in management. Part of the problem in
As a result of participating in this session, learners
the diagnosis of Cushing syndrome is that there
should be able to:
are numerous pitfalls in hormonal diagnostic
• Perform the appropriate hormonal workup in assays that often preclude an early and correct
the assessment of hypercortisolism. diagnosis. For example, it is well established that
cortisol measurements by immunoassays can be
• Demonstrate an accurate clinical approach to
thwarted by cross-reactivity with other adrenal
the diagnosis of Cushing syndrome.
steroids (eg, cortisone) and other interferences
inherent to binding-protein concentrations.3
Surgical resection continues to be first-line
therapy for patients with hypercortisolism unless
Main Conclusions precluded by comorbidities. Medical treatment
Hypercortisolism, or Cushing syndrome, can be is usually reserved for patients with persistent or
a challenging diagnosis for clinicians to make. recurrent disease not amenable to curative surgical
It requires some level of expertise from the resection or when the clinical condition precludes
physician and the possibility of repeating tests surgery. Medical treatment is classified in various
several times. For example, in the evaluation of categories: (1) medications that directly affect
endogenous Cushing syndrome, the guideline the ACTH-secreting corticotroph adenoma, (2)
algorithm recommends documenting a positive steroidogenesis inhibitors, and (3) glucocorticoid
result for 2 of 3 tests to diagnose hypercortisolism. receptor antagonists. Finally, a third-line option in
Currently available tests include 24-hour urinary some patients is the use of radiation.
free cortisol measurement, late-night or bedtime The presentation of hypercortisolism varies
salivary cortisol measurement, and the 1-mg from patient to patient and sometimes only 1
dexamethasone-suppression test (DST).1 sign or symptom is present or the characteristics
In contrast, when adrenal hypercortisolism develop very quickly. This complicates the
is suspected, the most accurate test to detect diagnosis and is considered an uncommon
adrenal secretion of cortisol is the 1-mg DST.2 presentation of the disease.
Improving understanding of the utility of current
hypercortisolism diagnostic tests could enhance
46 ENDO 2022 • Endocrine Case Management
• Inconsistent patient access to multidisciplinary
Significance of the teams, recommended laboratory tests, or
imaging modalities.
Clinical Problem
ACTH-dependent Cushing syndrome is a rare
disorder caused by an ACTH-secreting pituitary Strategies for Diagnosis,
adenoma or other ectopic ACTH-secreting tumor
Therapy, and/or Management
leading to endogenous hypercortisolism. ACTH-
independent Cushing syndrome is usually caused Endogenous Cushing syndrome has a variable
by an adrenal adenoma or, in rare cases, by an clinical presentation.5 Biochemical screening and
adrenocortical carcinoma.1,2 Signs and symptoms diagnostic tests include: (1) late-night or bedtime
can overlap with many other comorbidities such as salivary cortisol measurement (≥2 tests), (2)
obesity, metabolic syndrome, alcohol dependence, overnight 1-mg DST or low-dose 2-day DST, and
or depression. Optimal clinical outcomes require (3) 24-hour urinary free cortisol measurement
a detail-oriented diagnosis, management of (≥2 tests). Current guidelines advocate a
comorbidities, and careful treatment selection. combination of the described tests accounting for
Clinical judgment, index of suspicion, and clinical judgment, index of suspicion, and local test
laboratory and imaging results allow for early availability.3
diagnosis and guide management.3 Despite the After endogenous Cushing syndrome is
advancement of disease awareness, education confirmed, increased plasma ACTH measurements
for early recognition, availability of laboratory (>20 pg/mL [>4.4 pmol/L]) suggest ACTH-
testing, and newer imaging modalities, the mean dependent Cushing syndrome, while values
time to diagnosis is 38 months for patients with between 10 and 20 pg/mL (2.2 and 4.4 pmol/L)
an ACTH-secreting pituitary adenoma, 14 months can also be seen in patients with ACTH-
for patients with an ectopic ACTH-secreting independent Cushing syndrome. Values less than
tumor, and 30 months for adrenal Cushing 5 pg/mL (<1.1 pmol/L) are exclusively seen in
syndrome. Moreover, the time to early diagnosis ACTH-independent Cushing syndrome.
has not improved for recently treated patients in A summary of the recommended testing
comparison with the timelines in studies published modalities to distinguish between Cushing disease
before 2000.4 Ultimately, we need better education and an ectopic ACTH-secreting tumor are as
for nonendocrine and endocrine practitioners follows (Figure):
in pattern recognition of this condition and the • Inferior petrosal vein sampling (IPSS): IPSS
development of a more sensitive and specific test is the gold standard procedure to rule out
to detect hypercortisolism. ectopic ACTH production, particularly
when there are discordant findings from
Barriers to Optimal Practice noninvasive localization tests (pituitary
MRI, corticotropin-releasing hormone
• Physicians’ lack of awareness and insufficient (CRH)–stimulation test, 8-mg DST). IPSS
early recognition of the disease. is an invasive procedure that measures
• Lack of simple and effective biochemical ACTH in pituitary vs peripheral venous
screening approaches and reliance on a drainage. A basal ACTH inferior petrosal
combination of the recommended diagnostic sinus-to-peripheral ratio of 2 or greater or of
tests because of varying sensitivity and 3 or greater after stimulation suggests Cushing
specificity. disease. Prolactin measurements may improve
diagnostic accuracy.6
ENDO 2022 • Adrenal 47
Figure. Algorithm of Recent Consensus on Diagnosis and Management of Cushing Syndrome3
Reprinted from Fleseriu M et al. The Lancet Diabetes & Endocrinology, 2021; 9(12) © Elsevier Ltd. All rights reserved.
48 ENDO 2022 • Endocrine Case Management
• Ovine CRH-stimulation test: Increased plasma Treatment Modalities
ACTH of 35% or more from mean baseline to
The first-line treatment option for Cushing
15 to 30 minutes and increased serum cortisol
syndrome is surgery. Treatment of cortisol-
concentrations of 20% or more from mean
induced comorbidities should also be high priority.
baseline to 30 to 45 minutes after CRH suggest
Medical treatment is usually reserved for patients
Cushing disease.7 Additional protocols with
with persistent or recurrent disease not amenable
CRH or desmopressin administration can aid
to curative surgical resection or when the clinical
the confirmation Cushing disease.8,9
condition precludes surgery. Radiotherapy is
• High-dose (8-mg) DST: Serum cortisol reserved for patients with persistent or recurrent
suppression greater than 69% is observed in Cushing disease after transsphenoidal surgery.
77% of patients with Cushing disease. This test Bilateral adrenalectomy is used in selected patients
has variable accuracy. with persistent or recurrent Cushing disease who
• Pituitary MRI: Dedicated protocols to optimize do not respond to medical therapy.
the detection of pituitary microadenomas that Pharmacologic treatments for Cushing
may not be captured by 1.5T MRI are used. syndrome are classified in various categories:
Examples of these protocols include spoiled (1) medications that directly affect the ACTH-
gradient recalled acquisition echo with 1-mm secreting corticotroph adenoma (only for Cushing
slice intervals, fluid attenuation inversion disease), (2) steroidogenesis inhibitors, and (3)
recovery, T1-weighted turbo spin echo glucocorticoid receptor antagonists (Table).
sequences, and use of 3T and 7T magnets.
Pituitary adenomas smaller than 6 mm require Clinical Case Vignettes
IPSS, while adenomas 10 mm or larger may
not need IPSS if there is agreement with Case 1
additional biochemical localization test results. A 38-year-old woman seeks evaluation for a 30-lb
For 6- to 9-mm lesions, additional research is (14-kg) weight gain during the past year. She
required; however, most experts in a recent walks for 30 minutes daily and follows a healthy
consensus guideline advocated IPSS to confirm diet. Medical history is relevant for type 2 diabetes
the diagnosis in this situation.3 mellitus treated with metformin. Over the past
6 months, her hemoglobin A1c level has increased
Biochemical localization studies must be
from 6.5% to 8.0% (48 to 64 mmol/mol). Glipizide
performed while the patient is hypercortisolemic.
has been added to her treatment plan.
When results of noninvasive tests suggest Cushing
On physical examination, her blood pressure
disease, IPSS does not need to be performed.10
is 150/90 mm Hg, pulse rate is 75 beats/min,
Clinical presentation in combination with the
temperature is 98.4°F (37°C), and respiratory rate is
described test results determines diagnosis and
18 breaths/min. Her height is 65 in (165 cm), and
management. In cases where testing is suggestive of
weight is 220 lb (100 kg) (BMI = 36.6 kg/m2). She
ectopic ACTH-secreting tumors, combination
has erythematous, rounded facies, and prominent
of anatomical (CT and/or MRI) and functional
fat deposition in the supraclavicular area.
(68Ga-DOTATATE) imaging aids tumor
localization. Laboratory test results (after fasting 8 hours):
After biochemical confirmation of ACTH-
Hemoglobin A1c = 9.0% (4.0%-5.6%) (75 mmol/mol
independent Cushing syndrome, CT with adrenal
[20-38 mmol/mol])
protocol is the best next diagnostic step (Figure). Sodium = 137 mEq/L (136-142 mEq/L)
(SI: 137 mmol/L [136-142 mmol/L])
Potassium = 3.3 mEq/L (3.5-5.0 mEq/L)
(SI: 3.3 mmol/L [3.5-5.0 mmol/L])
ENDO 2022 • Adrenal 49
Table. Summary of Pharmacologic Medical Treatment Options for Cushing Syndrome
Agent Mechanism of Action Dosage Common adverse events FDA approval
Adrenal steroidogenesis inhibitors
Ketoconazole CYP11B1, CYP17 400 to 1600 mg daily every Hepatic dyscrasia, gastrointestinal No
inhibitor; requires acid 8 to 12 hours distress, male hypogonadism, drug
for biological activity interactions
Levoketoconazole CYP11B1, CYP17, and 150 to 600 mg every 12 Nausea, headache, peripheral No
CYP21A2 inhibitor hours edema, hypertension
Metyrapone CYP11B1 inhibitor; 500 mg to 6 g daily every 6 Gastrointestinal distress, hirsutism, No
variable access to 8 hours hypertension, hypokalemia
Mitotane Adrenolytic 250 mg to 8 g daily divided Gastrointestinal distress, Adrenocortical
every 6 to 8 hours central nervous system toxicity, cancer
gynecomastia, low white blood cell
count, high free T4, transaminitis,
high corticosteroid-binding globulin,
drug interactions, teratogenic
Etomidate CYP11B1 inhibitor Intravenous; validated Myoclonus, thrombophlebitis, No
intensive care unit protocols hypnotic effects
Osilodrostat CYP11B1 inhibitor Starting dosage of 1 mg Nausea, headache, adrenal Yes
twice daily in patients of insufficiency, QT prolongation,
Asian descent hirsutism
Starting dosage of 2 mg
twice daily in patients of
nonAsian descent
Maximum dosage: unlimited
Most patients require 4 to
14 mg total daily dose
Medications that target the pituitary tumor
Pasireotide Somatostatin analogue 600 to 900 mcg Hyperglycemia, bradycardia, Yes
(SSTR5); affinity for subcutaneous every QTc prolongation, transaminitis,
SSTR1, 2, 3, and 5, but 12 hours or LAR 10 to cholelithiasis, hypopituitarism
particularly for SSTR5 30 mg intramuscularly
Drug interactions (cyclosporin,
every 28 days
bromocriptine)
Cabergoline Dopamine agonist 1.5 to 7.0 mg weekly Transaminitis, cardiac valvular Yes
(DR2) (divided into 2 doses twice fibrosis, hypertension,
a week) gastrointestinal distress,
headaches, behavioral
abnormalities
Retinoic acid/ POMC downregulation Retinoic acid, 10 to 80 mg Liver injury, conjunctival irritation, No
isotretinoin daily, and isotretinoin, 20 to mucositis, arthralgia, transient
80 mg daily diarrhea
NOT CONSENSUS
GUIDELINES
R-roscovitine Selective inhibitor of 400 mg twice daily, 4 days Teratogenic, gastrointestinal No
the cyclin-dependent per week distress, hypokalemia
NOT CONSENSUS
kinase 2 (CDK-2)/cyclin
GUIDELINES
E signaling pathway
Glucocorticoid receptor antagonists
Mifepristone Glucocorticoid and 300 to 1200 mg daily Adrenal insufficiency, hypokalemia, Yes, in patients
progesterone receptor (but do not exceed 20 mg/kg vaginal bleeding, QTc prolongation, with type 2
antagonist per day) drug interactions diabetes mellitus
or glucose
intolerance
Relacorilant Glucocorticoid receptor 100 to 400 mg daily Back pain, headaches, peripheral No
antagonist edema, gastrointestinal distress,
NOT CONSENSUS
dizziness
GUIDELINES
50 ENDO 2022 • Endocrine Case Management
Calcium = 8.8 mg/dL (8.2-10.2 mg/dL) adrenal CT (Answer D) or pituitary MRI (Answer
(SI: 2.2 mmol/L [2.1-2.6 mmol/L]) E), but neither is a first step in diagnosis.
Glucose = 187 mg/dL (70-99 mg/dL)
(SI: 0.04 mmol/L [3.9-5.5 mmol/L])
Creatinine = 0.8 mg/dL (0.6-1.1 mg/dL) Case 2
(SI: 70.7 µmol/L [53.0-97.2 µmol/L])
Total cholesterol = 300 mg/dL (<200 mg/dL) A 34-year-old man with a history of superior
(SI: 7.77 mmol/L [<5.18 mmol/L] sagittal venous thrombosis, recurrent deep venous
TSH = 1.2 mIU/L (0.5-5.0 mIU/L) thrombosis (treated with apixaban), hypertension,
Estimated glomerular filtration rate = >60 mL/min type 2 diabetes mellitus, 70-lb (31.8-kg) weight
per 1.73 m2 (>60 mL/min per 1.73 m2)
gain over 5 months, and fatty liver is referred for
additional workup.
Which of the following is the most
Laboratory test results:
appropriate next diagnostic step?
A. Measure late-night salivary cortisol Urinary free cortisol = 220 µg/24 h (4-50 µg/24 h)
(SI: 607.2 nmol/d [11-138 nmol/d])
B. Measure morning ACTH Repeated measurement = 200 µg/24 h
C. Measure random serum cortisol (SI: 552 nmol/d)
Late-night salivary cortisol (by liquid
D. Perform adrenal CT chromatography/mass spectrometry) = 0.42 µg/dL
E. Perform pituitary MRI (<0.13 µg/dL) (SI: 11.59 nmol/L [<3.6 nmol/L])
Repeated measurement = 0.53 µg/dL
Answer: A) Measure late-night salivary cortisol (SI: 14.62 nmol/L)
ACTH = 60 pg/mL (10-60 pg/mL) (SI: 13.2 pmol/L
Late-night salivary cortisol concentrations [2.2-13.2 pmol/L])
(Answer A) are thought to correlate with level 8-mg DST = serum cortisol suppression by 87% with
of free circulating plasma cortisol. An increase in therapeutic dexamethasone levels
blood cortisol is reflected by a change in the salivary Pituitary 1.5T MRI reveals no pituitary adenoma.
cortisol concentration within a few minutes.
Several factors may lead to false-positive results: Which of the following is the best next step?
• Disruption of normal circadian rhythm in A. Dexamethasone/CRH-stimulation test
patients with depression or in shift workers B. DST (1 mg)
• Licorice, chewing tobacco, contamination C. IPSS
from bleeding gums/recent tooth brushing, or D. Repeated pituitary MRI
cigarette smoking may cause elevated late-night
cortisol Answer: C) IPSS
• Use of exogenous steroid-containing lotions or This patient has ACTH-dependent endogenous
oral gels hypercortisolemia with negative findings on MRI.
• Stress immediately before collection Localization with an 8-mg DST is suggestive of
Cushing disease. Due to incongruence between
Neither measuring ACTH (Answer B) nor random the 2 reported noninvasive localization tests, the
serum cortisol (Answer C) is a screening test next step is to perform IPSS (Answer C). IPSS is
for hypercortisolism. After hypercortisolism is the gold standard invasive test in the evaluation of
diagnosed, ACTH can be measured to determine ACTH-dependent Cushing syndrome.
whether it is ACTH-dependent or independent. This patient already underwent 2 screening
Depending on the result, the Endocrine Society tests and had positive results, thus confirming the
guideline algorithm indicates whether to perform diagnosis of hypercortisolism. There is therefore
no need to do a 1-mg DST (Answer B).
ENDO 2022 • Adrenal 51
Repeating the MRI (Answer D) would not yield ACTH inferior petrosal sinus-to-peripheral
any more information because the patient already (IPS/P) ratio of 2 or greater or of 3 or greater after
had a recent MRI that did not identify an adenoma. stimulation suggests Cushing disease. The best next
step is transsphenoidal surgery (Answer D). This
patient underwent transsphenoidal surgery, and
Case 2 (Continued)
the pathology report described a 2-mm adenoma
IPSS is performed during hypercortisolemia (see Table): in the right lobe of the pituitary. Biochemical
remission was subsequently achieved. The findings
Which of the following is the best next step? during transsphenoidal surgery confirmed that the
A. Another IPSS patient had pituitary hypercortisolism.
B. Chest CT There is no need to repeat IPSS (Answer A) or
C. 68Ga-DOTATATE scan to evaluate the patient for ectopic hypercortisolism
with a chest CT (Answer B) or 68Ga-DOTATATE
D. Transsphenoidal surgery
scan (Answer C).
Answer: D) Transsphenoidal surgery
Prolactin measurement during IPSS can improve Case 3
diagnostic accuracy and decrease the chance A 37-year-old woman without comorbidities
of false-negative results. A baseline prolactin presents to the emergency department with
inferior petrosal sinus-to-peripheral (IPS/P) ratio acute psychosis that rapidly evolves to depression
(ipsilateral to the dominant post-CRH ACTH with suicidal ideation and anorexia. Upon
IPS/P ratio) of 1.8 or greater suggests successful further interview, the patient reports weakness
catheterization during the procedure. A basal when walking upstairs, alopecia, and secondary
Time (min) Right petrosal-to- Left petrosal-to-
ACTH Right petrosal Left petrosal Peripheral peripheral ratio peripheral ratio
–5 260.3 pg/mL 78.5 pg/mL 27.0 pg/mL 9.64 2.90
(SI: 57.3 pmol/L) (SI: 17.3 pmol/L) (SI: 5.9 pmol/L)
0 4766.3 pg/mL 211.7 pg/mL 28.6 pg/mL 166.65 7.40
(SI: 1048.6 pmol/L) (SI: pmol/L) (SI: 6.3 pmol/L)
3 6218.3 pg/mL 433.7 pg/mL 24.7 pg/mL 251.75 17.55
(SI: 1368.0 pmol/L) (SI: 46.6 pmol/L) (SI: 5.4 pmol/L)
5 10,650.2 pg/mL 687.3 pg/mL 41.4 pg/mL 257.25 16.60
(SI: 2343.0 pmol/L) (SI: 151.2 pmol/L) (SI: 9.1 pmol/L)
10 9391.8 pg/mL 682.6 pg/mL 63.0 pg/mL 149.07 10.83
(SI: 2066.2 pmol/L) (SI: 150.2 pmol/L) (SI: 13.9 pmol/L)
Time (min) Right petrosal-to- Left petrosal-to-
prolactin Right petrosal Left petrosal Peripheral peripheral ratio peripheral ratio
–5 631.4 ng/mL 145.0 ng/mL 66.5 ng/mL 9.49 2.18
(SI: 27.5 nmol/L) (SI: 6.3 nmol/L) (SI: 2.9 nmol/L)
0 719.1 ng/mL 165.1 ng/mL 67.4 ng/mL 10.66 2.44
(SI: 31.3 nmol/L) (SI: 7.2 nmol/L) (SI: 2.9 nmol/L)
3 735.2 ng/mL 107.5 ng/mL 62.8 ng/mL 11.70 1.71
(SI: 32.0 nmol/L) (SI: 4.7 nmol/L) (SI: 2.7 nmol/L)
5 662.8 ng/mL 75.4 ng/mL 63.0 ng/mL 10.52 1.19
(SI: 28.8 nmol/L) (SI: 3.3 nmol/L) (SI: 2.7 nmol/L)
10 787.6 ng/mL 68.0 ng/mL 62.2 ng/mL 12.66 1.09
(SI: 34.2 nmol/L) (SI: 3.0 nmol/L) (SI: 2.7 nmol/L)
52 ENDO 2022 • Endocrine Case Management
amenorrhea. She has been taking valproic acid, Which of the following is the best next step?
600 mg daily; risperidone, 3 mg daily; sertraline, A. ACTH measurement
100 mg daily; diazepam, 5 mg daily; vitamin B12, B. Adrenal CT
100 mcg daily; vitamin D, 1000 IU daily; and
calcium, 1000 mg daily. C. Pituitary MRI
On physical examination, her blood pressure is D. Repeated 1-mg DST with dexamethasone
130/80 mm Hg. She has a rounded face, abdominal measurement
adiposity, muscle atrophy of the limbs, and
Answer: A) ACTH measurement
spontaneous bruising of the lower limbs.
After the diagnosis of endogenous Cushing
Laboratory workup (after 8 hours of fasting):
syndrome is established, serum ACTH is measured
Plasma glucose = 82 mg/dL (70-99 mg/dL) as a first step (Answer A) to determine the cause.
(SI: 4.6 mmol/L [3.9-5.5 mmol/L]) Patients with evidence of hypercortisolism and
White blood cell count = 4896/µL (4500-11,000/µL) elevated ACTH levels should undergo additional
(SI: 4.89 × 109/L [4.5-11.0 × 109/L])
Hematocrit = 42.6% (35%-45%) (SI: 0.426 [0.35-0.45])
testing, usually with pituitary MRI (Answer C).
Sodium = 141 mEq/L (136-142 mEq/L) Further testing after pituitary MRI may include
(SI: 141 mmol/L [136-142 mmol/L]) IPSS, CRH and desmopressin testing, and whole-
Potassium = 2.5 mEq/L (3.5-5.0 mEq/L) body CT.
(SI: 2.5 mmol/L [3.5-5.0 mmol/L]) Patients with low serum ACTH concentrations
Creatinine = 0.54 mg/dL (0.6-1.1 mg/dL)
(SI: 47.7 µmol/L [53.0-97.2 µmol/L])
should undergo adrenal imaging with CT
Serum cortisol (8 AM) = 20.7 µg/dL (5-25 µg/dL) (Answer B) and/or MRI to identify unilateral
(SI: 571.1 nmol/L [137.9-689.7 nmol/L]) masses with adjacent and contralateral atrophy or
Cortisol after 1-mg DST = 20.2 µg/dL bilateral disease.
(SI: 557.3 nmol/L) This patient already has documented elevated
Late-night salivary cortisol = 0.55 µg/dL
(<0.13 µg/dL) (SI: 15.2 nmol/L [<3.6 nmol/L])
urinary free cortisol and late-night salivary
Repeated measurement = 0.52 µg/dL cortisol, satisfying 2 of 3 screening tests necessary
(SI: 14.3 nmol/L) to diagnose hypercortisolism. Cortisol after 1-mg
Urinary free cortisol = 600 µg/24 h (4-50 µg/24 h) DST was elevated too. She has hypercortisolism,
(SI: 1656 nmol/d [11-138 nmol/d]) so there is no need to repeat a 1-mg DST
Repeated measurement = 550 µg/24 h
(SI: 1518 nmol/d)
(Answer D).
References
1. Carroll TB, Findling JW. The diagnosis of Cushing’s syndrome. Rev Endocr 7. Nieman LK, Oldfield EH, Wesley R, Chrousos GP, Loriaux DL, Cutler GB
Metab Disord. 2010;11(2):147-153. PMID: 20821267 Jr. A simplified morning ovine corticotropin-releasing hormone stimulation
2. Biller BM, Grossman AB, Stewart PM, et al. Treatment of test for the differential diagnosis of adrenocorticotropin-dependent Cushing’s
adrenocorticotropin-dependent Cushing’s syndrome: a consensus statement. J syndrome. J Clin Endocrinol Metab. 1993;77(5):1308-1312. PMID: 8077325
Clin Endocrinol Metab. 2008;93(7):2454-2462. PMID: 18413427 8. Tsagarakis S, Tsigos C, Vasiliou V, et al. The desmopressin and combined
3. Fleseriu M, Auchus R, Bancos I, et al. Consensus on diagnosis and CRH-desmopressin tests in the differential diagnosis of ACTH-dependent
management of Cushing’s disease: a guideline update. Lancet Diabetes Cushing’s syndrome: constraints imposed by the expression of V2 vasopressin
Endocrinol. 2021;9(12):847-875. PMID: 34687601 receptors in tumors with ectopic ACTH secretion. J Clin Endocrinol Metab.
4. Rubinstein G, Osswald A, Hoster E, et al. Time to diagnosis in Cushing’s 2002;87(4):1646-1653. PMID: 11932296
syndrome: a meta-analysis based on 5367 patients. J Clin Endocrinol Metab. 9. Ritzel K, Beuschlein F, Berr C, et al. ACTH after 15 min distinguishes
2020;105(3):dgz136. PMID: 31665382 between Cushing’s disease and ectopic Cushing’s syndrome: a proposal for
5. Nieman LK, Biller BM, Findling JW, et al. The diagnosis of Cushing’s a short and simple CRH test. Eur J Endocrinol. 2015;173(2):197-204. PMID:
syndrome: an Endocrine Society clinical practice guideline. J Clin Endocrinol 25953828
Metab. 2008;93(5):1526-1540. PMID: 18334580 10. Frete C, Corcuff J-B, Kuhn E, et al. Non-invasive diagnostic strategy
6. Sharma ST, Raff H, Nieman LK. Prolactin as a marker of successful in ACTH-dependent Cushing’s syndrome. J Clin Endocrinol Metab.
catheterization during IPSS in patients with ACTH-dependent Cushing’s 2020;105(10):dgaa409. PMID: 32594169
syndrome. J Clin Endocrinol Metab. 2011;96(12):3687-3694. PMID: 22031511
ENDO 2022 • Adrenal 53
Pheochromocytoma and
Paraganglioma: Diagnosis and
Perisurgical Management
Annika M. A. Berends, MD. Department of Endocrinology, University Medical Center
Groningen, University of Groningen, Groningen, The Netherlands; E-mail: m.a.berends@
umcg.nl
Michiel N. Kerstens, MD, PhD. Department of Endocrinology, University Medical Center
Groningen, University of Groningen, Groningen, The Netherlands; E-mail: m.n.kerstens@
umcg.nl
Learning Objectives • Genetic testing should be offered to all patients
with a new diagnosis of PPGL, regardless of age.
As a result of participating in this session, learners
should be able to: • Management of PPGL is complex and requires
a multidisciplinary team of dedicated specialists
• Describe the main steps in the diagnostic in centers with broad expertise.
workup of a pheochromocytoma or
• Surgical resection of a PPGL is a high-risk
sympathetic paraganglioma (PPGL).
procedure for which optimal pretreatment
• Illustrate the optimal approach to the with antihypertensive drugs, preferably
perioperative management of patients with a α-adrenergic blockade, is required in
PPGL with respect to preoperative evaluation, combination with state-of-the-art surgical
surgical approach, and perioperative medical procedures and anesthetic techniques.
treatment.
• Identify drugs that can be given safely and drugs
that should be avoided in patients with PPGL.
Significance of the
Clinical Problem
PPGLs are rare chromaffin-cell tumors originating
Main Conclusions in the adrenal medulla (pheochromocytomas)
and sympathetic paraganglia (paragangliomas),
• Liquid chromatography/tandem mass
which share the capacity to synthesize and release
spectrometry is the preferred assay method for
catecholamines. If not recognized or treated in
measurement of metanephrines.
a timely manner, patients with PPGL are at risk
• A pheochromocytoma is unlikely in a to develop fatal cardiovascular complications
patient with an adrenal tumor that has an from the excessive amounts of circulating
unenhanced attenuation value on CT less than catecholamines, which may result in myocardial
10 Hounsfield units (HU). infarction, cardiomyopathy, arrhythmias,
cerebrovascular disease, or hypertensive
54 ENDO 2022 • Endocrine Case Management
emergencies with multiorgan failure. Another of all solid tumors in humans. Targeted next-
cause of mortality is related to metastatic disease. generation sequencing should be offered to all
Most PPGLs are benign, but approximately 15% patients with a new diagnosis of PPGL, regardless
are metastatic, and metastases are present at the of age. It is worth considering repeated next-
initial diagnosis in 10% to 30% of cases.1 generation sequencing in patients who tested
Diagnosis of PPGL can be challenging for negative in the past, as the number of newly
several reasons. Symptoms and signs are highly identified susceptibility genes is still growing.1
variable, which may impede early recognition. Surgery is the only curative treatment option
Importantly, PPGLs belong to the group for benign PPGL, but it should only be performed
of orphan diseases and, as such, the pretest in centers with a multidisciplinary management
probability of this disease is low. Biochemical team with sufficient expertise. During surgery,
confirmation is based on the demonstration of a patient is exposed to several stimuli that may
increased levels of metanephrines in plasma or evoke an uncontrolled and massive release
urine. Reliable assessment of metanephrines of catecholamines from the PPGL into the
requires advanced laboratory techniques, which circulation. This surge may result in potentially
are not always available. Even when using a life-threatening cardiovascular complications
robust assay with high specificity, the positive (“pheo crisis”). Hazardous stimuli can be either
predictive value is limited because of the low mechanical (eg, endotracheal intubation, incision,
pretest probability of PPGL. Therefore, selecting peritoneal insufflation, tumor manipulation)
patients with an estimated higher risk of PPGL or pharmacological (eg, metoclopramide, some
is important to improve the positive predictive neuromuscular-blocking agents). Over the years,
value of biochemical testing.1 Measurement of the perioperative mortality rate has dropped
metanephrines is recommended in the following significantly from approximately 40% in the early
clinical scenarios or patients: days to 0% to 3% when performed in a center with
extensive expertise. Major developments in PPGL
• Patients with signs and symptoms of PPGL management that most likely have contributed to
• Patients with a history of cardiovascular events this notable improvement in surgical outcomes
(including Takotsubo cardiomyopathy) with are advancements in medical imaging allowing
suggestive signs or symptoms of PPGL precise tumor localization, introduction of
• When an adrenal incidentaloma is identified if minimally invasive surgery, optimization of
the unenhanced attenuation value is 10 HU or presurgical treatment care, and refinement of
greater anesthetic techniques.2
• Patients who are lean (BMI <25 kg/m2) with
type 2 diabetes Barriers to Optimal Practice
• Patients with history of PPGL or are a known
• Due to the low incidence of PPGL, the
carrier of a germline pathogenic variant in one
diagnosis is often delayed and expertise in
of the PPGL susceptibility genes
its management is not widely available. In
• Patients who have features suggesting addition, the rarity of this disease is a barrier to
genetically determined or syndromic PPGL conducting well-controlled clinical trials.
(eg, medullary thyroid carcinoma, multiple
• There is insufficient awareness that
café-au-lait spots)1
measurement of metanephrines should be
About 40% of PPGL cases are hereditary with an considered when an adrenal incidentaloma
autosomal dominant pattern of inheritance. In is found.
fact, PPGLs have the highest rate of heritability
ENDO 2022 • Adrenal 55
• There is no uniformly accepted definition Imaging of PPGL
of hemodynamic instability, which impedes
In the diagnostic workup of PPGL, biochemical
comparative research to establish optimal
confirmation is followed by imaging studies to
perisurgical management.
locate the tumor. Cross-sectional imaging by
• The perisurgical risk of a patient with PPGL contrast-enhanced CT or MRI is usually sufficient
is difficult to predict, as determinants of to visualize the tumor. The addition of functional
hemodynamic instability during surgical imaging studies, however, improves the sensitivity
resection are largely unknown. and specificity, in particular for demonstrating
multifocal disease or metastases. Moreover,
functional imaging studies can be used to evaluate
Strategies for Diagnosis, the possibility of peptide-receptor radionuclide
Therapy, and/or Management therapy. Several radiotracers are available, and
the optimal choice depends on tumor genotype,
Biochemical Diagnosis of PPGL biology, size, and biochemical phenotype. The
Measurement of the O-methylated metabolites of European Association of Nuclear Medicine and the
catecholamines (ie, free metanephrines) in either Society of Nuclear Medicine and Molecular Imaging
plasma or 24-hour urine is considered to be the have recently proposed a clinical algorithm for
cornerstone for the biochemical diagnosis of PPGL. nuclear imaging investigations of PPGL (Table 1).4
Caffeine, cigarette smoking, and alcohol intake
Table 1. Proposed Clinical Algorithm for
should be withheld for approximately 24 hours Nuclear Imaging Investigations in Patients With
before testing to avoid false-positive results. Sample Pheochromocytoma and Paraganglioma.4
collection for plasma metanephrines should be done Third choice
after at least 20 minutes of supine rest. Laboratory (if 18F-DOPA
methods for assessment of metanephrines differ in First Second or 68Ga-SSA is
Diagnosis choice choice not available)
diagnostic performance. Liquid chromatography–
Pheochromocytoma 18
F-DOPA 68
Ga-SSA 18
F-FDG
tandem mass spectrometry has the highest (sporadic) or
sensitivity (98%-100%) and specificity (94%-96%) 123
I-MIBG
and is virtually unaffected by analytical interference.3 Inherited 18
F-DOPA I-MIBG
123 18
F-FDG
pheochromocytoma or 68Ga-
Liquid chromatography with electrochemical (except SDHx): NF1/ SSA
detection has good sensitivity (95%-100%), but it RET/VHL/MAX
might be affected by analytical interference from Head and neck 68
Ga-SSA 18
F-DOPA In-
111
medications such as acetaminophen, α-methyldopa, paraganglioma SSA/99mTc-SSA
(sporadic)
sulfasalazine, and sotalol. Immunoassays for
Extraadrenal 68
Ga-SSA 18
F-FDG 18
F-FDG and
metanephrines demonstrate insufficient diagnostic sympathetic and/ and -123I-MIBG
performance and should preferably not be used. or multifocal and/or 18
F-DOPA or 18F-FDG
metastatic and/or and 111In-
Apart from analytical interference, which is assay SDHx mutation SSA/99mTc-SSA
dependent, pharmacodynamic interference by
medications causing falsely elevated plasma or The algorithm can be proposed as per the clinical situation. This algorithm
should be practically adapted in each institution and evolve with time.
urinary metanephrines should also be considered. Abbreviations: 123I-MIBG, iodine-123-labeled metaiodobenzylguanidine;
Such interference is independent from the assay 18
F-FDA, fluorine-18-labeled fluorodopamine; 18F-DOPA, fluorine-18-
labeled fluorodihydroxyphenylalanine; 18F-FDG, fluorine-18-labeled
applied and can be encountered with drugs fluorodeoxyglucose; 68Ga-SSA, gallium-68-labeled somatostatin
analogue; 111In-SSA, indium-111-labeled somatostatin analogue; 99mTc-
such as antidepressants, phenoxybenzamine, SSA, technetium-99-labeled somatostatin analogue. Adapted with
MAO-inhibitors, sympathomimetics, and permission from Taïeb D et al. Eur J Nucl Med Mol Imaging, 2019;46(10)
© Springer-Verlag GmbH Germany, part of Springer Nature.
levodopa. Withdrawal of these medications
might be considered when initial testing yields a
positive result.
56 ENDO 2022 • Endocrine Case Management
Genetics of PPGL Perisurgical Management
PPGL has the highest known heritability rate Treatment of PPGL requires a multidisciplinary
among all solid tumors in humans; almost 40% team of dedicated specialists in centers that
of PPGLs are hereditary. Genetic testing should have extensive experience in managing
therefore be offered to patients with a new these complex patients. Minimally invasive
diagnosis of PPGL. Lack of family history of PPGL adrenalectomy (including the laparoscopic
does not preclude the presence of a germline transperitoneal, posterior retroperitoneal, and
pathogenic variant. Next-generation sequencing lateral retroperitoneal technique) is currently
is the preferred technique to analyze all relevant the preferred surgical approach for most
genes in a single testing panel. It is important to pheochromocytomas. Laparoscopic surgery
engage patients in shared decision-making before was found to be safe and resulted in a shorter
genetic testing.1 Currently, more than 20 different duration of surgery, shorter hospital stay, and
causative germline variants have been identified. lower complication rate, including less blood
Based on their transcriptional profile, PPGLs loss, when compared with open adrenalectomy.1
have been divided into 2 clusters. Cluster 1, or Bleeding complications, however, should not
the pseudohypoxia subgroup, includes pathogenic be neglected with the laparoscopic approach
variants in genes encoding the succinate (reported frequency of approximately 4%). Lateral
dehydrogenase complex (SDHx [ie, SDHA, SDHAF2, transperitoneal adrenalectomy and posterior
SDHB, SDHC, SDHD), VHL, EPAS1, FH, MDH2, retroperitoneoscopic adrenalectomy have been
or EGLN1 (PHD2). These pathogenic variants evaluated in 2 relatively small randomized
stabilize hypoxia-inducible factor 1α and 2α, which controlled trials, demonstrating either no
in turn activate genes that induce angiogenesis, difference or superiority of the posterior approach
metabolism, and proliferation. Cluster 2, or the with respect to perioperative and recovery
kinase signaling subgroup, includes pathogenic outcomes. Open adrenalectomy is advised as the
variants in the RET protooncogene, NF1, MAX, primary approach in case of large (usually >6 cm)
and TMEM127. In addition, 30% to 40% of sporadic or invasive PPGL.1
PPGLs have somatic driver variants in cluster Another important aspect of presurgical
1 genes (eg, VHL, EPAS1) or cluster 2 genes (eg, management is medical treatment to provide
NF1, RET). Cluster 3 genes have been identified symptom relief and control of hypertension. For
as a separate group of somatic variants resulting this purpose, α-adrenergic receptor blockers are
in increased expression of genes in the Wnt often considered the treatment of choice. Table 2
signaling pathway. These include genes include displays the proposed presurgical oral treatment
CSDE1 and MAML3.5 Germline pathogenic variants options for patients with PPGL.2 Instead of
in PPGL susceptibility genes demonstrate an orally administered medications, there are also
autosomal dominant inheritance pattern. In the some intravenous alternatives that could be
case of SDHD, SDAF2, and MAX variants, there considered in specific cases (Table 3).2 Tachycardia
is maternal imprinting. Thus, the disease is only frequently occurs in patients with PPGL, either as
manifest when the pathogenic variant is paternally a direct consequence of catecholamines or as an
inherited. SDHx variants are the most prevalent adverse effect of α-adrenergic receptor blockers.
(20%-30% of cases of PPGL), with the highest Tachycardia is effectively treated with either
frequency in SDHB (9%-10%) and SDHD (2%-9%). selective or nonselective β-adrenergic receptor
The risk of metastatic PPGL is also highest in blockers. An important point to keep in mind
cluster 1 tumors. In particular, development of when starting β-adrenergic receptor blockers is
metastatic PPGL is most often seen in patients that these drugs should only be started after a
with SDHB (~25%) and SDHA variants (~12%).6 patient is already receiving α-adrenergic receptor
blockade for several days. Otherwise, a crisis could
ENDO 2022 • Adrenal 57
be provoked due to unopposed stimulation of the is routinely performed, and placement of a
α-adrenergic receptors. It is usually advised to central venous catheter is strongly recommended
give intravenous saline during the last 24 hours for central venous pressure measurement and
prior to surgery. The hypothesis underlying this administration of vasoactive agents and fluid
practice is that high sodium should reduce the management. Transesophageal echocardiography
risk of preoperative orthostatic hypotension and might be considered for more optimal real-
postoperative hypotension. Supportive evidence time monitoring of intravascular volume status
for this is limited and based on only a few and early detection of myocardial wall motion
retrospective studies. abnormalities.7 Blood pressure should be kept within
In general, the anesthetic management aims safe limits during surgery. There is, however, no
to provide optimal hemodynamic stability during consensus with respect to the exact blood pressure
PPGL surgery and the postoperative stage. threshold values that require intervention by
Intraarterial blood pressure measurement is essential, the anesthesiologist and no clear agreement on
as it allows for real-time monitoring of hemodynamic the definition of hemodynamic instability. In the
fluctuations and frequent blood sampling. Urine PRESCRIPT study, these thresholds were defined as
output monitoring via urinary bladder catheterization a mean arterial pressure less than 60 mm Hg and a
Table 2. Suggested Presurgical Oral Treatment of Patients With PPGL2
Incremental dose
Drug Starting dosage steps* Dose range Comments
Phenoxybenzamine 10 mg daily 20 mg 10-140 mg Preferably started at least 7 to 14 days prior
to surgery, also in case of normotension;
or doses higher than starting dose are
Doxazosin ER 4 mg daily 4 mg 4-56 mg administered twice daily
Nifedipine ER 30 mg daily 30 mg 30-90 mg Add-on to α-adrenergic receptor blockade
in case of persistent hypertension (blood
or pressure supine >130/80 mm Hg, systolic
amlodipine 5 mg daily 5 mg 5-10 mg blood pressure upright >110 mm Hg)
or
Metyrosine 250 mg 3 times daily 250-500 mg 750-2000 mg
Metoprolol ER 50 mg daily 50 mg 50-200 mg Add-on in case of tachycardia (heart rate
supine >80 beats/min, heart rate upright
or >100 beats/min); preferably started after
Propranolol 20 mg 3 times daily 20 mg 20-240 mg sufficient preparation with α-adrenergic
receptor blockade (≥3 to 4 days)
or
Atenolol 25 mg daily 25 mg 25-100 mg
High-sodium ≥15 g … … Restoration of intravascular volume
chloride diet depletion; prevention of preoperative
orthostatic hypotension and postoperative
and hypotension
Saline, 0.9% 2 L/24 h … … Diet should be started >7 to 14 days
intravenously before surgery
Intravenous saline should be started
24 hours before surgery
*Dosage adjustments preferably every 2 to 4 days at the discretion of the clinician and guided by the response of blood pressure and/or heart rate
If supine blood pressure is greater than 160/100 mm Hg 24 hours before planned surgery, consider postponing surgery. Surgery is usually performed in the
morning, and the last dose of each oral drug should preferably be administered the evening before surgery. In case surgery begins after 12 AM, the last dose
of each oral drug should be administered at 7 AM the day of surgery. Only the administration of 0.9% saline should be continued during surgery. Adapted
from Berends AMA et al. J Clin Endocrinol Metab, 2020; 105(9) © Endocrine Society.
58 ENDO 2022 • Endocrine Case Management
Table 3. Suggested Intravenous Treatment of Hypertension and
Tachyarrhythmia in Case of PPGL Crisis or During Surgery2
Duration of
Onset of action (after
Indication Drug Dosage action discontinuation)
Hypertension
Step 1a Magnesium sulfateb Loading dose: 40-60 mg/kg followed by infusion of Immediate 30 min
1-4 g/h
Step 2 Phentolaminec Bolus: 2.5-5 mg at 1 mg/min, repeated every 3-5 min 1-2 min 10-30
2.5-5 mg at 1 mg/min, repeated every 3 to 5 min min
Continuous: 100 mg in 500 mL of 5% dextrose 1-5 min 5-11 h
20-100 mg/h
or
Urapidil Bolus: initial dose, 25-50 mg orolus: initial dose: 1-5 min 15-30 min, may
25-50 mg exceed 12 h after
prolonged infusion
or
Continuous: 10-15 mg/h
Step 3 Nicardipine Starting dose: 5 mg/h, increased by 2.5 mg/h every 2-4 min 5-15 min
5 min (if needed), maximum dose 15 mg/h
or
Clevidipine Starting dose: 1-2 mg/h, increase by doubling the dose Immediate 2-3 min
every 90 seconds (if needed), maximum dose 32 mg/h
Step 4 Sodium Starting dose: 0.5-1.5 mcg/kg per min, dosage range: 2-5 min 5-10 min
nitroprusside 0.5-4 mcg/kg per min; stop administration if no results
are achieved after 10 min of infusion, maximal dose for
10 min only
or
Nitroglycerine Infusion adjusted according to response within the
range of 10-200 mcg/mind
Tachyarrhythmias
Step 1 Esmolole Bolus: 500 mcg/kg in 1 min, repeat bolus after 5 min (if 1-5 min 15-30 min
needed)
Continuous: 25-100 mcg/kg per min, increase infusion
rate to 300 mcg/kg per min (if needed)
Step 2 Amiodarone Loading dose (bolus): 5 mg/kg, followed by infusion of 1-30 min 1-3 h
15 mcg/kg per min
or
Lidocaine Loading dose (bolus): 1 mg/kg, repeat after 5-10 min <2 min 15-20 min
(if needed)
Continuous: 2-4 mg/min (1-2 mg/mL),
maximum 300 mg/h
Suggested steps need to be individualized based on comorbidity.
a
Check for adequate pain treatment and depth of anesthesia.
b
Also prevention and treatment of tachyarrhythmias.
c
In case of norepinephrine-producing tumor, 2 mg before tumor manipulation.
d
Tachyphylaxis can occur after a continuous infusion greater than 24 hours.
e
In case of epinephrine- or dopamine-producing tumor, 20 mg before tumor manipulation.
Adapted from Berends AMA et al. J Clin Endocrinol Metab, 2020; 105(9) © Endocrine Society.
ENDO 2022 • Adrenal 59
systolic blood pressure greater than 160 mm Hg.8 D. The plasma metanephrine concentration
The number of vasoactive agents and volume is affected by ingestion of certain food
therapy (infusion of fluids, predominantly products
intravenous saline, to correct hypotension) required
to maintain the blood pressure within these limits Answer: C) Renal insufficiency may increase
may vary substantially among patients. Blood pressure the plasma metanephrine concentration
levels alone are therefore insufficient to evaluate Clinical manifestations of PPGL are highly variable
hemodynamic stability during PPGL resection. To and are related to the amounts, proportions, and
overcome this problem, a hemodynamic instability secretion patterns of catecholamines (ie, episodic
score has been developed consisting of 3 components or continuous). In a prospective cohort of patients
(hemodynamic variables including blood pressure with (n = 245) and without (n =1820) PPGL
and heart rate, volume therapy, and vasoactive (participants in whom metanephrines had been
agents), which provides a tool to quantify the determined), it was demonstrated that only 19%
degree of intraoperative hemodynamic instability.9 of patients with PPGL presented with the classic
triad of headache, palpitations, and sweating.10 In
Clinical Case Vignettes the same study, a score system was developed to
estimate PPGL risk. The likelihood was increased
Case 1 with a symptom score of 3 or greater, where 1
A 51-year-old woman with chronic hypertension, point was assigned to each of 7 features (ie, BMI
palpitations, and intermittent headaches is <25 kg/m2, heart rate ≥85 beats/min, pallor,
evaluated for the presence of a pheochromocytoma. sweating, palpitations, tremor, nausea), with a
She follows a diet rich in fruits and vegetables. She negative point for obesity. The patient in this
has no family history of pheochromocytoma. case had a symptom score of at least 3, so further
On physical examination, her blood pressure evaluation for PPGL was justified.
is 176/105 mm Hg and pulse rate is 88 beats/min. The interpretation of plasma or urinary
BMI is 22 kg/m2. metanephrine measurement is optimized when
using sex- and age-specific reference intervals
Laboratory test results: (thus, Answer A is incorrect).11 Negative test
Serum creatinine = 1.4 mg/dL (0.6-1.1 mg/dL) results virtually exclude PPGL, the exception being
(SI: 125.0 µmol/L [53.0-97.2 µmol/L]) very small or nonsecreting tumors. Elevations of
Plasma normetanephrine = 220 pg/mL (<165 pg/mL) any plasma metabolite greater than 2 times the
(SI: 1.20 nmol/L [<0.90 nmol/L])
upper reference limit or increases in 2 or more
Plasma metanephrine = 108 pg/mL (<99 pg/mL)
(SI: 0.55 nmol/L [<0.50 nmol/L]) metabolites are very rare in the absence of PPGL
3-Methoxytyramine = <0.04 nmol/L (<0.04 nmol/L) but occur in more than 80% of patients with PPGL
(thus, Answer B is incorrect).1
Catecholamine-rich foods may increase
Which of the following is correct regarding the plasma or urine concentration of
biochemical analysis for PPGL? 3-methoxytyramine, but they do not affect the
A. Plasma metanephrines are not affected by age plasma or urine concentration of metanephrine
or sex (thus, Answer D is incorrect).
B. PPGL can be safely excluded when the plasma A common problem in clinical practice is the
metanephrine elevation is less than 2 times the presence of kidney impairment. Plasma levels
upper normal limit of free metanephrines are frequently elevated
C. Renal insufficiency may increase the plasma in patients with stage 3 chronic kidney disease
metanephrine concentration (or worse) and in those on long-term dialysis
treatment (thus, Answer C is correct). In addition,
60 ENDO 2022 • Endocrine Case Management
PPGL is frequently suspected in patients with unenhanced attenuation value of the adrenal
chronic kidney disease, as both conditions share tumor also predicts a very low pheochromocytoma
similar signs and symptoms such as hypertension, risk. In a meta-analysis, it was shown that a
blood pressure swings, palpitations, and precontrast HU value less than 10 had a sensitivity
pulmonary edema. Optimized reference intervals of 99% for excluding pheochromocytoma.13 To
for plasma metanephrines in patients with chronic find 1 pheochromocytoma with a density less than
kidney disease have been established.12 10 HU, it was estimated that 1232 patients would
have to be screened by measuring metanephrines.
A limitation of this meta-analysis, however, is
Case 2
that all of the included studies were retrospective
A 65-year-old man with obesity is admitted to in design.
the thoracic surgery department for coronary In general, it is probably prudent to measure
artery bypass grafting the following day. The metanephrines in any patient with an adrenal
thoracic surgeon asks for advice on the finding incidentaloma before undergoing surgery. This
of a tumor in the right adrenal gland, which was decision should be balanced, however, against the
demonstrated 6 months earlier on CT of the risk and inconvenience of postponing surgery,
thorax and has not been evaluated. The adrenal as results of metanephrine measurement are
tumor measures 2.1 × 2.6 cm, has sharp margins, often not available within a day after blood
and has an unenhanced attenuation value of 8 sampling. In this case, information on the very
HU. The patient does not report any symptoms small risk of a pheochromocytoma provided
suggestive of PPGL and has hypertension the anesthesiologist the opportunity to take
controlled with an ACE inhibitor and a precautionary measures and be prepared for
β-adrenergic blocker. the unlikely event of hemodynamic instability
occurring during coronary artery bypass grafting
Which of the following is the best (thus, Answer A is correct and Answers B and C
advice for the thoracic surgeon? are incorrect). Apart from the absence of a clear
A. Continue with coronary artery bypass grafting indication for starting doxazosin, a further drop in
after informing the anesthesiologist about the already well-controlled blood pressure carries the
presence of the adrenal incidentaloma risk of inducing cardiac ischemia in a patient with
B. Measure plasma metanephrines and postpone coronary artery disease.
coronary artery bypass grafting
C. Start doxazosin, 8 mg once daily, and continue Case 3
with coronary artery bypass grafting as planned
A 35-year-old man is referred by the clinical
Answer: A) Continue with coronary artery bypass geneticist because of the recent finding of a
grafting after informing the anesthesiologist pathogenic variant in the SDHB gene identified on
about the presence of the adrenal incidentaloma family screening. He feels well, has no concerns,
and is normotensive. His father was the index
The median reported frequency of patient in whom the SDHB variant was first
pheochromocytoma in adrenal incidentalomas detected after resection of a PPGL at age 62 years.
is 7% (range, 1.4%-14%), although this
figure is probably an overestimation, as
pheochromocytoma is a rare disease. A recent
nationwide study in the Netherlands demonstrated
an incidence rate for pheochromocytoma of
0.46 per 100,000 person years. This patient
had a symptom score less than 3, and the low
ENDO 2022 • Adrenal 61
Which of the following tests is carriers of SDHA, SDHC, and SDHD variants (only
NOT indicated as part of the initial in case of parental inheritance for SDHD carriers).
workup of an asymptomatic carrier
of an SDHB pathogenic variant?
Case 4
A. 24-Hour ambulatory blood pressure monitoring
A 45-year-old man presents to the outpatient clinic
B. Examination by a urologist for evaluation of an incidentally found adrenal
C. Measurement of plasma metanephrines mass on ultrasonography performed for abdominal
D. MRI of the head and neck and thoracic, pain for which no cause was eventually established.
abdominal, and pelvic regions Other than abdominal pain, the patient has no
E. PET with 68Ga-DOTATATE concerns. Biochemical analysis confirms the diagnosis
of a pheochromocytoma. CT reveals a right
Answer: B) Examination by a urologist adrenal mass measuring 2.1 × 2.5 × 3.0 cm, with no
evidence for locally invasive or metastatic disease.
Apart from PPGL, including head and neck
The patient is referred to the endocrine surgeon
paragangliomas, patients with SDHx germline
for adrenalectomy.
pathogenic variants are also at increased risk for renal
cell carcinoma and gastrointestinal stromal tumors.
Which of the following additional
A recently issued consensus statement on initial
examinations should be
screening and follow-up of asymptomatic SDHx performed preoperatively?
variant carriers recommends initial screening with
24-hour ambulatory blood pressure monitoring A. Ambulatory blood pressure monitoring
(Answer A), symptom questionnaire, measurement B. Electrocardiography
of plasma metanephrines (Answer C), and MRI C. Echocardiography
of the head and neck and thoracic, abdominal, D. A and B
and pelvic regions (Answer D).14 In addition, E. B and C
asymptomatic adults who carry an SDHx variant
should have a baseline PET (Answer E). The expert F. A and C
panel did not reach consensus on the optimal PET G. All of the above
tracer to be used for this purpose, but it did refer
Answer: G) All of the above
to the recent EANM-SNMMI guideline, which
recommends the use of 68Ga-DOTA-somatostatin Preoperative evaluation of a patient with PPGL
analogue PET as the first-choice functional imaging should include detailed history taking, physical
modality (Table 1). For screening purposes, examination, electrocardiography (Answer B),
examination by a urologist does not have echocardiography (Answer C), and 24-hour
additional value, as renal cell carcinoma is best ambulatory blood pressure monitoring (Answer A).
diagnosed by imaging (thus, Answer B is correct). Thus, the best answer is “all of the above” (Answer G).
Follow-up consists of annual measurement Examination of cardiac function should be performed
of blood pressure, measurement of plasma in every patient regardless of physical performance,
metanephrines, and symptom questionnaire. MRI clinical picture, tumor size, cardiovascular risk factors,
of the aforementioned regions should be repeated or age. A smaller tumor size (<3 cm) does not
every 2 to 3 years. Recommended follow-up in protect against the occurrence of cardiovascular
children is similar, except for measurement of complications, and it has been shown that there is
metanephrines every 2 years. The first tumor no relationship between tumor size and the rate of
screening should be performed between age 6 and perisurgical complications. The clinical picture of
10 years for asymptomatic carriers of SDHB variants catecholamine-induced cardiomyopathy is often
and between 10 and 15 years in asymptomatic atypical, and this disorder might also develop in a
62 ENDO 2022 • Endocrine Case Management
patient at a young age in the absence of cardiovascular given orally. Doxazosin has been studied more
risk factors. Notably, preoperative abnormalities extensively than either prazosin or terazosin.
on electrocardiography or echocardiography Compared with doxazosin, phenoxybenzamine
are associated with an increased risk for cardiac has a longer duration of action and causes more
complications.15 reflex tachycardia due to the irreversible receptor
binding and blockade of the presynaptic α2
receptor, respectively. A practical disadvantage
Case 4 (continued)
of phenoxybenzamine is the relatively high cost
Nifedipine, 30 mg once daily, is started. Supine and its limited availability in several countries.
blood pressure is 152/85 mm Hg, and it drops In a recently published meta-analysis, there
to 140/75 mm Hg after 3 minutes in the upright were no differences found between groups with
position. Pulse rate is 88 beats/min. Laboratory respect to overall morbidity and mortality. There
measurements demonstrate normal electrolytes is only one randomized controlled trial on this
and normal kidney function. Electrocardiography topic (the PRESCRIPT study).8 In this study,
shows a sinus rhythm of 70 beats/min with no no differences were found in the total duration
abnormalities. Transthoracic echocardiography of blood pressure outside a predefined target
reveals a left ventricular ejection fraction of 58%, range. There was, however, less hemodynamic
stroke volume of 90 mL, and normal diastolic and instability during surgery after pretreatment
valvular function with no additional strain pattern with phenoxybenzamine. Furthermore, there
visible. were no differences between groups with
respect to postoperative hypotension, duration
Which of the following is the preferred of hospital stay, perioperative complications, or
next step in this patient’s management? adverse events.
A. Administer no pretreatment β-adrenergic receptor blockers (Answer D)
B. Increase the nifedipine dose to 60 mg once daily should only be started after a patient is already
C. Start doxazosin 4 mg once daily receiving α-adrenergic receptor blockade
D. Start metoprolol, 50 mg once daily for several days. Otherwise, a crisis could be
provoked due to unopposed stimulation of the
E. Start metyrosine, 250 mg 3 times daily α-adrenergic receptors.
Answer: C) Start doxazosin 4 mg once daily There are a few alternative drugs for
α-receptor blockers, such as calcium channel
Table 2 displays an overview of the suggested blockers (Answer B) and metyrosine (Answer E).
presurgical oral treatment of patients with Calcium channel blockers are most often used as
PPGL.2 In general, α-adrenergic receptor an add-on drug to α-receptor blockers if blood
blockers are advised as the treatment of first pressure control is not achieved sufficiently. These
choice. There is longstanding experience with drugs inhibit the catecholamine-mediated calcium
these agents, and their use is a good example influx in vascular smooth muscle cells, thereby
of targeted therapy, as these drugs specifically reducing vasoconstriction. Calcium channel
block the overstimulation of the α receptors by blockers do not cause tachycardia and have only
catecholamines. These drugs are started at least a moderate effect on preload reduction, which
7 to 14 days before surgery in a stepwise manner could be an advantage in case of coexisting heart
until blood pressure targets are achieved. Either failure. Presurgical monotherapy with oral calcium
a competitive, selective α1 receptor blocker (eg, channel blockers has been described, but they
doxazosin [Answer C], prazosin, or terazosin), should probably be used in only selected patients.
or a noncompetitive, nonselective α1 and α2 Another suggested preparation drug is metyrosine
receptor blocker (ie, phenoxybenzamine) may be (α-methyl-para-tyrosine), which significantly
ENDO 2022 • Adrenal 63
reduces circulating catecholamine levels through How should this patient be advised
inhibition of tyrosine hydroxylase, the key enzyme regarding antiemetic treatment?
catalyzing the rate-limiting step in catecholamine A. Data regarding safety of antiemetic therapy are
biosynthesis. In general, metyrosine should be conflicting
prescribed as an add-on drug to α-adrenergic B. Metoclopramide is superior to ondansetron
blockade, since monotherapy has been shown to be
C. Most antiemetic agents should only be given
less effective. Compared with α-receptor blockers
after sufficient α-adrenergic receptor blockade
and calcium channel blockers, metyrosine has
has been achieved
some disadvantages: it is expensive, has limited
availability, and has several adverse effects such D. No antiemetic medication is safe to use
as depression, anxiety, and somnolence. Of notice, preoperatively
the absence of hypertension does not preclude the Answer: C) Most antiemetics should only
occurrence of cardiovascular complications, and be given after sufficient α-adrenergic
normotensive patients therefore also require adequate receptor blockade has been achieved
medical pretreatment. The level of evidence
supporting pretreatment in patients with PPGL Several antiemetic medications have been
is moderate and mainly based on observational suggested to elicit a pheochromocytoma crisis,
studies and expert opinion. There are no either by direct or indirect stimulation of
randomized placebo-controlled trials on this topic. catecholamine release. Importantly, frequently
Several investigators have recently questioned prescribed antiemetics such as metoclopramide
the need for presurgical treatment, especially with and other dopamine receptor antagonists are
α-adrenergic receptor blockade.16 The few available contraindicated in this setting. Table 4 displays an
comparative studies suggesting that these drugs overview of the drugs that should be avoided and
could be omitted safely are without exception small- those drugs that can be given safely in patients
sized, retrospective in design, and flawed by several with PPGL. Most of these observations are based
methodological shortcomings. The debate about the on case reports. It is, however, best to avoid these
clinical value of presurgical α-receptor blockade can drugs or to only introduce them after sufficient
only be resolved by a well-designed randomized α-adrenergic receptor blockade has been achieved
controlled trial. Until then, it is advised to follow (Answer C).
the Endocrine Society’s guideline that all patients
with a functional PPGL, regardless of tumor size,
Case 6
degree of catecholamine production, or blood
pressure should undergo pretreatment (thus, A 56-year-old man is admitted to the hospital
Answer A is incorrect).1,3 because of cardiogenic shock. Physical
examination shows a strongly fluctuating blood
pressure, ranging from addition 112/70 mm Hg to
Case 5 193/95 mm Hg, pulse rate of 60 beats/min, sinus
A 70-year-old woman with pheochromocytoma rhythm, and peripheral oxygen saturation (SpO2)
is admitted to the hospital. Surgery is scheduled of 90% on room air. He is noted to have mild
in 2 weeks. She has been experiencing nausea and nonpitting bilateral ankle edema. He has a history
vomiting, which has compromised her adherence of coronary artery disease and has been taking
to doxazosin, 4 mg twice daily. On physical enalapril, metoprolol, acetylsalicylic acid, and a
examination, her blood pressure is 140/80 mm Hg, statin. Electrocardiography shows ST elevations,
and pulse rate is 75 beats/min with a normal and serum levels of cardiac-specific troponins
sinus rhythm. She would like a prescription for are increased. Transthoracic echocardiography
antiemetic treatment. is notable for a diffusely dyskinetic left ventricle,
64 ENDO 2022 • Endocrine Case Management
Table 4. Overview of Medications That Can Be Safely Used and Potential Precipitants of Adverse Outcomes
During Perioperative Anesthesia in Patients With PPGL2
Sedative-hypnotic agents used for anesthetic induction/ Sedative-hypnotic agents used for anesthetic induction/
maintenance maintenance
Inhalation agents Anxiolytic agents
Safe Comments Safe Comments
Sevoflurane Midazolam No anxiolytic agent is superior
Isoflurane Lorazepam
Enflurane Diazepam
Nitrous oxide Analgesic agents
Not safe Comments Safe Comments
Halothane Arrhythmogenic, sensitizing Fentanyl
the myocardium to circulating
catecholamines Sufentanil
Alfentanil
Desflurane Used without incident in some cases;
however, not recommended, can induce Remifentanil
sympathomimetic effects
Not safe Comments
Intravenous agents
Morphine These opioids cause histamine release,
Safe Comments thereby provoking catecholamine
Hydromorphone
release
Propofol
Pethidine*
Etomidate
Antiemetic agents
Dexmedetomidine
Safe Comments
Not safe Comments
Ondansetron
Ketamine Sympathomimetic effects
Not safe Comments
Thiopental Used without incident in some cases;
(thiopentone) however, not recommended, can Metoclopramide D2 receptor antagonists: cause indirect
precipitate hypertensive crises, linked to Chlorpromazine but potent release of catecholamines by
histamine-releasing properties Prochlorperazine stimulation of presynaptic D2 receptors
Neuromuscular-blocking agents Domperidone Can induce hypertensive crisis
Safe Comments Droperidol Severe hypertension due to increased
Haloperidol catecholamine efflux
Rocuronium
Cisapride Can stimulate catecholamine release
Vecuronium from PPGL cells through activation of
Not safe Comments serotonin type 4 receptors
Succinylcholine Can precipitate a hypertensive crisis and Miscellaneous
(suxamethonium) severe cardiac arrhythmias Not safe Comments
Pancuronium Histamine release, thereby causing Ephedrine Sympathomimetic effects, stimulates
severe pressor response catecholamine release from PPGL
Atracurium Cause histamine release, thereby Atropine Inhibits cardiac vagus and potentiates
Tobucurarine provoking catecholamine release, chronotropic effects of catecholamines
Mivacurium associated with severe arterial
hypertension and ventricular
*Pethidine is also known as meperidine, isonipecaine, lidol, operidine,
arrhythmias pethanol, and piridosal.
Cisatracurium Adverse reactions linked to histamine- Adapted from Berends AMA et al. J Clin Endocrinol Metab, 2020; 105(9)
releasing properties, thereby provoking © Endocrine Society.
catecholamine release
ENDO 2022 • Adrenal 65
consistent with the diagnosis of Takotsubo and emergency resection of a pheochromocytoma
cardiomyopathy with a reduced ejection fraction of (Answer F) should be avoided given the associated
15%. Coronary angiography reveals no significant high perioperative morbidity and mortality.
coronary stenosis. Cardiac ischemia is mainly Control of hypertension should be the initial
attributed to coronary spasms. An underlying treatment goal here, preferably achieved through
diagnosis of pheochromocytoma is suspected and medication that has rapid onset with a relatively
confirmed by additional biochemical testing and short duration of action, thereby enabling effective
CT that demonstrates a 3.5-cm right adrenal mass. dosage titration. There are no evidence-based
The patient is transferred to the intensive care unit recommendations with respect to the preferred
for hemodynamic stabilization. medical management of a pheochromocytoma
crisis. Several drugs have been described in
Which of the following is the best the literature (Table 3). In view of the patient’s
treatment recommendation? medical history of coronary artery disease,
A. Bolus of phentolamine intravenously urapidil (Answer D) should be the preferred
B. Bolus of sodium nitroprusside intravenously option. Urapidil is a competitive α1-adrenergic
receptor antagonist with central agonistic action at
C. Continuous administration of esmolol
serotonin 5-HT1A receptors and it does not cause
intravenously
reflex tachycardia.
D. Continuous administration of urapidil Phentolamine (Answer A) is contraindicated
intravenously in patients with coronary artery disease because it
E. Doxazosin orally twice daily can induce reflex tachycardia.
F. Emergency resection of the Sodium nitroprusside (Answer B) may reduce
pheochromocytoma coronary perfusion and induce intracoronary steal
G. No additional treatment is feasible given the due to a strong decline in mainly diastolic arterial
strong blood pressure swings pressure, which is undesirable in case of heart
failure and concomitant coronary artery spasm.
Answer: D) Continuous administration A β1 selective adrenergic receptor blocker,
of urapidil intravenously such as esmolol (Answer C), should be given in
case of tachyarrhythmia after achievement of
Pharmacotherapy is the mainstay of medical
sufficient α-adrenergic receptor blockade.
stabilization in case of a pheochromocytoma crisis,
References
1. Lenders JWM, Kerstens MN, Amar L, et al. Genetics, diagnosis, management 5. Berends AMA, Eisenhofer G, Fishbein L, et al. Intricacies of the molecular
and future directions of research of phaeochromocytoma and paraganglioma: machinery of catecholamine biosynthesis and secretion by chromaffin cells of
a position statement and consensus of the Working Group on Endocrine the normal adrenal medulla and in pheochromocytoma and paraganglioma.
Hypertension of the European Society of Hypertension. J Hypertens. Cancers (Basel). 2019;11(8):1121. PMID: 31390824
2020;38(8):1443-1456. PMID: 32412940 6. Amar L, Pacak K, Steichen O, et al. International consensus on initial
2. Berends AMA, Kerstens MN, Lenders JWM, Timmers HJLM. screening and follow-up of asymptomatic SDHx mutation carriers. Nat Rev
Approach to the patient: perioperative management of the patient with Endocrinol. 2021;17(7):435-444. PMID: 34021277
pheochromocytoma or sympathetic paraganglioma. J Clin Endocrinol Metab. 7. Naranjo J, Dodd S, Martin YN. Perioperative management of
2020;105(9):dgaa441. PMID: 32726444 pheochromocytoma. J Cardiothorac Vasc Anesth. 2017;31(4):1427-1439. PMID:
3. Lenders JWM, Duh Q-Y, Eisenhofer G, et al; Endocrine Society. 28392094
Pheochromocytoma and paraganglioma: an Endocrine Society clinical 8. Buitenwerf E, Osinga TE, Timmers HJLM, et al. Efficacy of α-blockers on
practice guideline. J Clin Endocrinol Metab. 2014;99(6):1915-1942. PMID: hemodynamic control during pheochromocytoma resection: a randomized
24893135 controlled trial. J Clin Endocrinol Metab. 2020;105(7):2381-2391. PMID:
4. Taïeb D, Hicks RJ, Hindié E, et al. European Association of Nuclear Medicine 31714582
Practice Guideline/Society of Nuclear Medicine and Molecular Imaging 9. Buitenwerf E, Boekel MF, van der Velde MI, et al. The haemodynamic
Procedure Standard 2019 for radionuclide imaging of phaeochromocytoma instability score: development and internal validation of a new rating
and paraganglioma. Eur J Nucl Med Mol Imaging. 2019;46(10):2112-2137. method of intra-operative haemodynamic instability. Eur J Anaesthesiol.
PMID: 31254038 2019;36(4):290-296. PMID: 30624247
66 ENDO 2022 • Endocrine Case Management
10. Geroula A, Deutschbein T, Langton K, et al. Pheochromocytoma and 13. Buitenwerf E, Berends AMA, van Asselt ADI, et al. Diagnostic accuracy of
paraganglioma: clincial feature-based disease probability in relation to computed tomography to exclude pheochromocytoma: a systematic review,
catecholamine biochemistry and reason for disease suspicion. Eur J Endocrinol. meta-analysis, and cost analysis. Mayo Clin Proc. 2019;94(10):2040-2052.
2019;181(4):409-420. PMID: 31370000 PMID: 31515105
11. Eisenhofer G, Peitzsch M, Kaden D, et al. Reference intervals for LC-MS/ 14. Amar L, Pacak K, Steichen O, et al. International consensus on initial
MS measurements of plasma free, urinary free and urinary acid-hydrolyzed screening and follow-up of asymptomatic SDHx mutation carriers. Nat Rev
deconjugated normetanephrine, metanephrine and methoxytyramine. Clin Endocrinol. 2021;17(7):435-444. PMID: 34021277
Chim Acta. 2019;490:46-54. PMID: 30571948 15. Shen J, Yu R. Perioperative management of pheochromocytoma: the heart of
12. Pamporaki C, Prejbisz A, Małecki R, et al. Optimized reference intervals the issue. Minerva Endocrinol. 2013;38(1):77-93. PMID: 23435444
for plasma free metanephrines in patients with CKD. Am J Kidney Dis. 16. Groeben H, Walz MK, Nottebaum BJ, et al. International multicentre review
2018;72(6):907-909. PMID: 30146420 of perioperative management and outcome for catecholamine-producing
tumours. Br J Surg. 2020;107(2):e170-e178. PMID: 31903598
ENDO 2022 • Adrenal 67
Below the Tip of the
Iceberg: Just How Much
Aldosterone Is Out There?
Martin Reincke, MD. Medizinische Klinik und Poliklinik IV, Munich University Hospital,
Munich, Germany; E-mail:
[email protected]Learning Objectives patients with arterial hypertension and
hypokalemia, patients with arterial hypertension
As a result of participating in this session, learners
and obstructive sleep apnea, and patients with
should be able to:
arterial hypertension and adrenal incidentaloma.
• Describe the prevalence of and populations at Screening is performed using the sensitive, but
risk for endocrine hypertension. not specific, aldosterone-to-renin ratio (ARR).
The diagnosis requires confirmatory testing
• Describe the target population that should be
in most patients, using tests such as the saline-
screened for primary aldosteronism (PA).
loading test or the captopril challenge test. Subtype
• Order appropriate tests to diagnose and differentiation is carried out by means of CT
subtype PA. imaging of the adrenal glands in combination
• Initiate individualized treatment depending on with, as a gold standard, adrenal venous sampling
PA subtype. to distinguish between unilateral and bilateral
aldosterone production. Patients with PA have
an increased risk of stroke, myocardial infarction,
atrial fibrillation, and renal insufficiency compared
Main Conclusions with patients who have essential hypertension.
Early diagnosis and appropriate targeted therapy
PA, also called Conn syndrome, is a common
are critically important. Adrenalectomy is
cause of secondary hypertension associated with
indicated for patients with unilateral PA. In
high cardiovascular morbidity and mortality. PA
patients with BAH, therapy with spironolactone
remains underdiagnosed because it does not have
(starting dosage 25 mg daily, up-titrated according
a specific, easily identifiable feature and clinicians
to blood pressure, hypokalemia, and renin
are often unaware of the disease. The causes of PA
concentration) should be initiated.
can be a unilateral aldosterone-producing adrenal
adenoma (APA) or bilateral idiopathic adrenal
cortex hyperplasia (BAH). However, advances in
molecular histopathology challenge this traditional Significance of the
concept of PA as a binary disease. The classic Clinical Problem
hypokalemic form occurs more frequently in the
Once considered rare, PA is now thought to be
case of APA, whereas the normokalemic form
the most common secondary endocrine form
is mostly observed in patients with BAH. PA
of hypertension. The introduction of ARR as a
screening is required in patients with refractory
screening tool and its application to a widening
hypertension, younger patients with hypertension,
population of patients with hypertension have
68 ENDO 2022 • Endocrine Case Management
led to a marked increase in the detection of PA. Asymmetric bilateral primary aldosteronism
Most studies report PA prevalence rates between with some lateralization is common, and this
5% and 15%, with most affected patients being poses challenges in diagnosis and treatment.
normokalemic. Recent studies have suggested that
autonomous aldosterone secretion could be even
more frequent than currently accepted in patients Strategies for Diagnosis,
with hypertension. Variability in prevalence rates Therapy, and/or Management
reported in different studies is explained by the use
of different diagnostic methods and cutoffs. Introduction
Aldosterone excess in persons with PA PA is defined as hypertension caused by
is associated with deleterious effects on the disproportionately high aldosterone secretion
heart, vessels, brain, and kidney, which are with consequent low plasma renin and increased
partly independent of elevated blood pressure ARR. PA is the most common cause of endocrine
determined by hyperaldosteronism. These effects arterial hypertension: recent data indicate a
result in disproportionate degrees of organ prevalence of 5.8% in unselected hypertensive
damage for a given blood pressure and lead to left patients in primary care, 6% to 12% in patients in
ventricular hypertrophy, diastolic dysfunction, hypertensive outpatient clinics, and up to 30% in
heart failure, increased intima media thickening, patients with resistant hypertension.1
arterial stiffness, arterial wall inflammation, Compared with essential hypertension, PA is
stroke, renal hyperfiltration, albuminuria, and associated with increased morbidity and mortality,
glomerulosclerosis. In terms of metabolic effects, even when patients have the same blood pressure.
aldosterone excess causes decreased insulin This underscores the importance of a timely
secretion and insulin resistance leading to diabetes diagnosis of the affected patients, as there is an
mellitus, lower serum calcium and higher urinary excellent long-term prognosis after adequate
calcium excretion leading to hyperparathyroidism, therapy. A recently published meta-analysis
increased prevalence of osteoporosis and risk showed a decrease in mortality, which started to
of bone fractures, and sleep apnea through fluid become noticeable 5 to 7 years after initiation of
retention. therapy, especially in the group of patients who
underwent unilateral adrenalectomy. Mortality
Barriers to Optimal Practice decreased below that of patients with essential
hypertension.2
• The patient groups for whom screening is Diagnosing PA is a 3-part process: screening,
recommended account for approximately 50% confirmatory testing, and subtyping. The 2 most
of the hypertensive population. However, common causes of PA are APAs (accounting
currently fewer than 1% of patients with PA for approximately one-third of cases), which
are screened and treated during their lifetime. can be treated surgically by means of unilateral
adrenalectomy, and BAH (accounting for
• Subtyping using imaging and adrenal approximately two-thirds of cases), which can
venous sampling is driven by the intention be treated with mineralocorticoid receptor
to identify suitable candidates for unilateral antagonists. Rare causes are unilateral aldosterone-
adrenalectomy. Adrenal venous sampling is producing carcinomas and familial forms of the
complex, costly, and not widely available. disease caused by germline pathogenic variants
• The 2 subtypes of PA, unilateral and bilateral (familial hyperaldosteronism [FH] I-IV).
forms, represent extremes of a spectrum In recent years, important new articles on
of histomorphological and biochemical the diagnosis and treatment of PA have been
phenotypes and are not totally distinct. published.
ENDO 2022 • Adrenal 69
Diagnosis 100%. These results suggest that ARR has a limited
ability to identify patients with PA.4
Who should be screened? According to the
Endocrine Society guidelines, groups at risk for PA
should be screened, which applies to up to 50% of Confirmatory Testing
hypertensive patients.3 Screening is recommended Since ARR as a screening test has limited
for patients with the following: sensitivity and specificity, most patients with a
• Blood pressure >150/100 mm Hg on positive screening test should be further evaluated
3 different days with confirmatory testing. The principle of a
dynamic confirmation test is used here, which is
• Therapy-resistant hypertension intended to demonstrate the lack of aldosterone
(3 antihypertensive agents, including a suppressibility.
diuretic) Note: The intravenous saline-load test, captopril
• Controlled hypertension on challenge test, oral saline-load test, and fludrocortisone-
≥4 antihypertensive drugs suppression test are available as confirmation tests.
• Hypertension and hypokalemia These tests have similar diagnostic accuracies, but they
differ in adverse effects and feasibility.
• Hypertension and adrenal incidentaloma The diagnostic algorithm allows the
• Hypertension and sleep apnea confirmatory test to be bypassed in certain
• Hypertension and positive family history of situations: patients with high plasma aldosterone
early-onset hypertension or cerebrovascular (>20 ng/dL [>555 pmol/L]), low renin, and
event occurring at younger than 40 years spontaneous hypokalemia can proceed directly
to subtype differentiation.3,5 Very recently, a
• First-degree relatives with PA
study involving 240 patients undergoing the
saline-infusion test assessed discordance between
automated immunoassay and mass spectrometry–
Screening based measurements of plasma aldosterone. Plasma
Patients are screened using the ARR. In some aldosterone measured by immunoassays were
settings, renin activity has been replaced by respectively 86% and 58% higher than by mass
renin concentration because of its rapidity and spectrometry. Discordance was noted in 16% to
reproducibility. Plasma aldosterone measurement 32% of patients, who by adrenal venous sampling
using radioimmunoassay is also being gradually were classified as having BAH. Discordance was
replaced by much more precise mass spectrometry. eliminated by plasma purification to remove
Note: To achieve a high level of reliability when interferents. These findings raise concerns about
screening using ARR, care should be taken to ensure a the validity of automated immunoassay-based
liberal salt intake, to compensate for any hypokalemia diagnosis of PA.6
that may be present, and to take into consideration To simplify the diagnosis, the use of a so-
antihypertensive drugs that potentially interfere with called prediction score has been investigated
the renin-angiotensin-aldosterone system. several times. Recently, an Italian working group
In particular, β-adrenergic blockers and was able to create a flow chart and score with
mineralocorticoid receptor antagonists lead to various parameters (sex, antihypertensive drugs,
false-positive or false-negative screening results potassium, end-organ damage, renin, aldosterone)
due to their influence on renin secretion. A with which 22.8% of all confirmatory tests could
recently published meta-analysis, which included be bypassed.7
10 studies on a total of 4110 patients, showed
that the sensitivity of ARR varies between 10%
and 100% and the specificity ranges from 70% to
70 ENDO 2022 • Endocrine Case Management
Subtyping unilateral disease with a high specificity of 89%
to 100%.13-15 To simplify the diagnostic flow,
The rate of incidentaloma increases with age,
“prediction scores” based on steroid profiling or
so that bilateral adrenal venous sampling is
clinical parameters were also examined. Several
recommended for subtype differentiation.3 The
studies have demonstrated a difference in the
details of the adrenal vein catheter protocol
secretion of the hybrid steroids 18-hydroxycortisol
(sequential vs simultaneous withdrawal or
and 18-oxocortisol between patients with PA
cosyntropin-stimulated vs unstimulated
vs primary hypertension, but also between the
withdrawal) differ among centers. Adrenal venous
PA subtypes.16 A Japanese working group was
sampling is a costly and invasive procedure and is
able to show that the 18-oxocortisol values in
very rarely associated with severe adverse effects
patients with unilateral APAs were significantly
(hemorrhage). It requires a high level of expertise
higher than in patients with BAH (diagnostic
and is therefore not widely available. Imaging
sensitivity 83%, specificity 99%).17 The authors
of the adrenal glands is always performed to not
proposed that by measuring 18-oxocortisol in
only visualize adrenal morphology, but also as a
their diagnostic model, mineralocorticoid receptor
planning tool to visualize adrenal vein anatomy.
antagonist therapy could be started directly in
Imaging also rules out the very rare adrenal
43% of patients with bilateral disease without the
carcinoma. Some older studies were able to show
need for adrenal venous sampling. Subsequently,
that adrenal venous sampling leads to more
another European study demonstrated a reduced
accurate lateralization than imaging (sensitivity
ability of 18-hydroxycortisol and 18-oxocortisol
95% vs 78%; specificity 100% vs 75%).8,9 A more
to discriminate between the 2 subtypes. Mass
recent European randomized controlled study
spectrometry measurement of 12 different steroids
of 200 patients with PA (SPARTACUS trial)
led to correct subtype classification in 80% of
showed no significant difference in the clinical
patients with PA.18 Although some of the steroids
and biochemical outcome when the therapeutic
used were different, these study findings suggest
decision for adrenalectomy was based on adrenal
that peripheral blood steroid profiling may make
venous sampling or CT10; however, this result has
adrenal venous sampling unnecessary in the future
not been confirmed in most other studies. In a
in patients with bilateral disease.
retrospective data analysis of a large multicenter
Modified criteria from confirmatory testing
cohort that included 18 centers, there were
are also useful in subtype differentiation. Various
again indications of inferiority of CT-based
studies have shown that subtype differentiation
decision-making vs adrenal venous sampling–
is possible based on results of the saline-infusion
based diagnostics.11,12 In additional studies, the
test using a score.19,20 The addition of clinical
discrepancy of CT vs catheter-based diagnostics
parameters (age, potassium, adrenal mass) may
could be demonstrated several times, so taken
increase the discriminatory ability of the saline-
together, adrenal venous sampling remains firmly
infusion test.21 Recently, Burello et al validated
anchored as the preferred test in the differential
prediction models for PA subtype diagnostics in a
diagnosis of PA.
large study using machine learning.22
Note: Adrenal venous sampling continues to be
A high expression of CXCR4 (CXC chemokine
the gold standard in the differential diagnosis of
receptor type 4) in APA could be demonstrated,
PA and paves the way for subtype-specific therapy:
correlating with the expression of CYP11B2
adrenalectomy for APA and mineralocorticoid receptor
(aldosterone synthase). In one study, the specific
antagonist treatment for BAH.
CXCR4 tracer 68Ga-pentixafor PET-CT showed
The guidelines recommend proceeding
a stronger uptake in patients with known APA.23
directly to adrenalectomy in certain situations
Functional imaging using ¹¹C-metomidate
(young age with pronounced clinical signs, adrenal
PET-CT also represents a promising method
mass in imaging).3 Various scores can predict
ENDO 2022 • Adrenal 71
for noninvasive subtype differentiation, as patients with nonclassic histopathology. Patients
recently published case reports suggest.24,25 The with nonclassic histopathology showed a higher
11
C-metomidate labels CYP11B2 activity in the absolute aldosterone concentration in the vein of
adrenal cortex. However, other studies show the (unresected) contralateral adrenal gland during
limited suitability, so further analyses appear adrenal venous sampling.
necessary.26 Prior to adrenalectomy, therapy with MR
antagonists should be initiated to compensate
for hypokalemia and control blood pressure.
Therapy
After adrenalectomy, follow-up is necessary
Current data suggest that adrenalectomy is to verify biochemical and clinical remission of
superior to mineralocorticoid receptor antagonist PA. Medical therapy with mineralocorticoid
therapy in terms of mortality, cardiovascular and receptor antagonists is recommended to treat
metabolic risk, kidney function, and quality of BAH. Drug therapy can also be prescribed for
life.27-29 Patients with PA who are treated with patients who cannot undergo operation for
mineralocorticoid receptor antagonists have a medical reasons or who decline to do so. A study
higher risk of atrial fibrillation than patients with by Hundemer et al compared cardiovascular
essential hypertension, while adrenalectomized outcomes among 3 groups: 205 patients with
patients no longer have an increased risk.30 unilateral disease who underwent adrenalectomy,
A large population-based study was able to 602 patients with BAH who received medical
demonstrate a reduced risk of diabetes mellitus in treatment, and more than 40,000 patients
adrenalectomized patients vs the risk in patients with primary hypertension.33 Despite similar
with essential hypertension, while patients blood pressure control, patients treated with
treated with medication showed an increased mineralocorticoid receptor antagonists had a
risk.29 The PASO study recently defined the significantly higher risk of cardiovascular events
criteria for surgical cure.10 The complete clinical than patients who underwent adrenalectomy,
cure represents the postoperative normalization while patients who underwent adrenalectomy
of blood pressure; complete biochemical cure had a significantly lower risk than patients with
represents normalization of serum potassium and primary hypertension. However, patients who
ARR. In the PASO study, 37% of 700 patients experienced renin normalization induced by
studied met criteria for complete clinical cure, mineralocorticoid receptor antagonist therapy
while 94% had complete biochemical remission. had a similar cardiovascular risk to that of
A recent nomenclature consensus workshop patients who underwent adrenalectomy. A similar
(HISTALDO, histopathology of primary conclusion was reached by another study showing
aldosteronism) supported use of CYP11B2 that the increased risk of atrial fibrillation in drug-
(aldosterone synthase)-based immunohistochemistry treated patients with PA was associated with renin
for classification of adrenal histopathology lesions suppression.34 This illustrates the importance of
in PA.31 It distinguished classic histopathologic adequate dosing of mineralocorticoid receptor
findings (APA, dominant aldosterone-producing antagonists.
nodules) from nonclassic ones (multiple Note: Mineralocorticoid receptor antagonists
aldosterone-producing micronodules, diffuse should be gradually titrated: the starting dosage of
adrenal hyperplasia). A prospective study spironolactone is 25 to 50 mg daily. The aim is to
over 3 years in 60 adrenalectomized patients32 normalize blood pressure, serum potassium levels, and
showed that APA or dominant aldosterone- renin levels. In addition, care should be taken to restrict
producing nodes were associated with higher salt intake.
initial aldosterone and ARR values and a higher
probability of postoperative remission (97.6% vs
66.7% [P = .002]) compared with observations in
72 ENDO 2022 • Endocrine Case Management
Summary Which of the following is the most likely
pretest probability for PA in this patient?
PA is the most common cause of endocrine arterial
hypertension, affecting 6% of the hypertensive A. <5%
population, most of whom are undiagnosed. The B. 5%-10%
diagnostic workup continues to consist of a 3-step C. 10%-15%
process: screening using ARR, confirmatory D. >15%
testing, and subtyping (with CT and adrenal venous
sampling). To simplify the diagnosis, so-called Answer: D) >15%
prediction scores are increasingly used. Steroid
This patient has resistant hypertension, as defined
profiling from peripheral blood (using liquid
by the American Heart Association and European
chromatography–mass spectrometry) could reduce
Society of Cardiology classification, and she should
the need for adrenal venous sampling in patients
be screened for PA according to all guidelines.
with bilateral disease. Functional imaging is also
The pretest probability for such a patient in most
a promising method for noninvasive subtype
studies between 15% and 30% (Answer D).
differentiation. Adrenalectomy offers the potential
for a cure by eliminating aldosterone excess, leading
to improved patient outcomes. In the case of drug Case 2
therapy, mineralocorticoid receptor antagonists A 33-year-old man with recent-onset grade 2 WHO
should be dosed adequately: the aim is to normalize hypertension has been evaluated for primary
blood pressure, potassium levels, and renin levels. aldosteronism. His height is 69 in (175 cm), and
weight is 165 lb (75 kg) (BMI = 24.4 kg/m2). He
Clinical Case Vignettes has spontaneously hypokalemia (lowest potassium
concentration, 2.6 mEq/L [2.6 mmol/L]), and he
Case 1 does not currently take any antihypertensive drugs.
A 44-year-old woman presents with an
8-year history of arterial hypertension. Her His biochemical workup shows the following
height is 67 in (170 cm), and weight is 181 lb hormone levels:
(82 kg) (BMI = 28.3 kg/m2). She has tried many Plasma aldosterone = 455 ng/dL (50-200 ng/dL)
antihypertensive medications, but her blood (SI: 12,622 pmol/L [1387-5548 pmol/L])
pressure has become increasingly unresponsive Plasma renin = <2 pg/mL (3-40 pg/mL)
(SI: <0.05 pmol/L [0.07-0.95 pmol/L])
in recent years. Currently, she takes candesartan, ARR = 152 (<20)
8 mg twice daily; chlortalidone, 25 mg daily;
and amlodipine, 5 mg twice daily. Her office These values are similar to those documented 1 week
blood pressure is 170/99 mm Hg and automated earlier. Abdominal CT without contrast shows a
ambulatory blood pressure monitoring reading is 1.5-cm adrenal mass with a density of 5 Hounsfield
161/95 mm Hg without night dipping. units. The contralateral adrenal gland is normal in
appearance.
Laboratory test results:
Serum creatinine, normal Which of the following diagnostic
Serum potassium = 3.8 mEq/L (SI: 3.8 mmol/L) procedures is most appropriate?
(on a low-salt diet)
A. Adrenal MRI
Serum sodium = 144 mEq/L (SI: 144 mmol/L)
B. Adrenal venous sampling
C. Confirmatory testing using saline-infusion test
D. No diagnostic testing; the patient can be
directly referred for right adrenalectomy
ENDO 2022 • Adrenal 73
Answer: D) No diagnostic testing; the patient can D. The results suggest probable cortisol secretion
be directly referred for right adrenalectomy in the left adrenal vein
E. All of the above
This young patient has a pronounced form of PA.
According to guideline recommendations, he does Answer: E) All of the above
not need confirmatory testing (Answer C) because
of the severe biochemical phenotype. He also does Answers A through D are all correct. The
not need adrenal venous sampling (Answer B) selectivity index (ratio of cortisol right and left
because he is younger than 35 years and the right adrenal / cortisol peripheral vein) is greater than
adrenal nodule is suggestive of a fat-rich adenoma 2.0, which suggests selective catheterization.
(Hounsfield units <10). Adrenal MRI (Answer A) There is strong lateralization to the right side
is not superior to adrenal CT in localizing adrenal (ratio aldosterone/cortisol right adrenal vein /
adenomas in patients with PA. Thus, the patient ratio aldosterone/cortisol left adrenal vein of 296
can be referred directly for right adrenalectomy [>4.0 indicate lateralization]). The contralateral
(Answer D). aldosterone/cortisol ratio is below the peripheral
aldosterone/cortisol ratio, and the absolute
aldosterone concentration in the right adrenal vein
Case 3 (400 ng/dL [11,096 pmol/L]) is minimally higher
A 55-year-old man with recent-onset grade 2 than in the periphery (320 ng/dL [8877 pmol/L]).
WHO hypertension has been diagnosed with PA The large adrenal mass on the right side, however,
based on an elevated ARR and nonsuppressed and the high cortisol concentration in the left
aldosterone in response to intravenous saline adrenal vein could suggest cortisol cosecretion. A
loading. Abdominal CT demonstrates a 3.5-cm 1-mg dexamethasone-suppression test should be
adrenal mass in the left adrenal gland and a 1.5-cm performed to rule this out.
right adrenal lesion. The patient undergoes
bilateral simultaneous unstimulated adrenal
venous sampling (see Table).
Which of the following interpretations
is most appropriate?
A. The adrenal venous sampling procedure is
technically successful for both adrenal glands,
in terms of selectivity index
B. The results suggest lateralization to the right
adrenal vein
C. The results suggest contralateral suppression
in the left adrenal vein
Selectivity
index adrenal
Source Cortisol Aldosterone veins Lateralization index
Right adrenal vein 35 µg/dL 14,500 ng/dL 7 414
(SI: 965.6 nmol/L) (SI: 402,230 pmol/L)
Peripheral vein 15 µg/dL 320 ng/dL 21
(SI: 413.8 nmol/L) (SI: 8877 pmol/L)
Left adrenal vein 277 µg/dL 400 ng/dL 18 1.4
(SI: 7641.9 nmol/L) (SI: 11,096 pmol/L)
74 ENDO 2022 • Endocrine Case Management
References
1. Yang Y, Reincke M, Williams TA. Prevalence, diagnosis and outcomes of in patients with primary aldosteronism. Hypertension. 2015;65(5):1096-1102.
treatment for primary aldosteronism. Best Pract Res Clin Endocrinol Metab. PMID: 25776074
2020;34(2):101365. PMID: 31837980 18. Eisenhofer G, Dekkers T, Peitzsch M, et al. Mass spectrometry-based adrenal
2. Meng Z, Dai Z, Huang K, et al. Long-term mortality for patients of primary and peripheral venous steroid profiling for subtyping primary aldosteronism.
aldosteronism compared with essential hypertension: a systematic review and Clin Chem. 2016;62(3):514-524. PMID: 26787761
meta-analysis. Front Endocrinol (Lausanne). 2020;11:121. PMID: 32210920 19. Nagano H, Kono T, Saiga A, et al. Aldosterone reduction rate after
3. Funder JW, Carey RM, Mantero F, et al. The management of primary saline infusion test may be a novel prediction in patients with primary
aldosteronism: case detection, diagnosis, and treatment: an Endocrine Society aldosteronism. J Clin Endocrinol Metab. 2020;105(3):dgz092.
Clinical Practice Guideline. J Clin Endocrinol Metab. 2016;101(5):1889-1916. 20. Hashimura H, Shen J, Fuller PJ, et al. Saline suppression test parameters may
PMID: 26934393 predict bilateral subtypes of primary aldosteronism. Clin Endocrinol (Oxf).
4. Hung A, Ahmed S, Gupta A, et al. Performance of the aldosterone to renin 2018;89(3):308-313. PMID: 29873811
ratio as a screening test for primary aldosteronism: a systematic review 21. Kocjan T, Janez A, Stankovic M, Vidmar G, Jensterle M. A new clinical
and meta-analysis. J Clin Endocrinol Metab. 2021;106(8):2423-2435. PMID: prediction criterion accurately determines a subset of patients with bilateral
34008000 primary aldosteronism before adrenal venous sampling. Endocr Pract.
5. Wang K, Hu J, Yang J, et al. Development and validation of criteria for 2016;22(5):587-594. PMID: 26789347
sparing confirmatory tests in diagnosing primary aldosteronism. J Clin 22. Burrello J, Burrello A, Pieroni J, et al. Development and validation
Endocrinol Metab. 2020;105(7):dgaa282. PMID: 32449927 of prediction models for subtype diagnosis of patients with primary
6. Eisenhofer G, Kurlbaum M, Peitzsch M, et al. The saline infusion test for aldosteronism. J Clin Endocrinol Metab. 2020;105(10):dgaa379. PMID:
primary aldosteronism: implications of immunoassay inaccuracy. J Clin 32561919
Endocrinol Metab. 2021 [Online ahead of print] PMID: 34963138 23. Heinze B, Fuss CT, Mulatero P, et al. Targeting CXCR4 (CXC chemokine
7. Burrello J, Amongero M, Buffolo F, et al. Development of a prediction receptor type 4) for molecular imaging of aldosterone-producing adenoma.
score to avoid confirmatory testing in patients with suspected primary Hypertension. 2018;71(2):317-325. PMID: 29279316
aldosteronism. J Clin Endocrinol Metab. 2020;106(4):e1708-e1716. PMID: 24. Burton TJ, Mackenzie IS, Balan K, et al. Evaluation of the sensitivity and
33377974 specificity of (11)C-metomidate positron emission tomography (PET)-CT for
8. Young WF, Stanson AW, Thompson GB, Grant CS, Farley DR, van Heerden lateralizing aldosterone secretion by Conn’s adenomas. J Clin Endocrinol Metab.
JA, Role for adrenal venous sampling in primary aldosteronism. Surgery. 2012;97(1):100-109. PMID: 22112805
2004;136(6):1227-1235. PMID: 15657580 25. O’Shea PM, O’Donoghue D, Bashari W, et al.,= (11) C-Metomidate PET/
9. Nwariaku FE, Miller BS, Auchus R, et al. Primary hyperaldosteronism: effect CT is a useful adjunct for lateralization of primary aldosteronism in routine
of adrenal vein sampling on surgical outcome. Arch Surg. 2006;141(5):497- clinical practice. Clin Endocrinol (Oxf). 2019;90(5):670-679. PMID: 30721535
502. PMID: 16702522 26. Soinio, M., et al., Functional imaging with 11C-metomidate PET for subtype
10. Dekkers T, Prejbisz A, Schultze Kool LJ, et al; SPARTACUS Investigators. diagnosis in primary aldosteronism. Eur J Endocrinol. 2020;183(6):539-550.
Adrenal vein sampling versus CT scan to determine treatment in primary PMID: 33055298
aldosteronism: an outcome-based randomised diagnostic trial. Lancet Diabetes 27. Hundemer GL, Curhan GC, Yozamp N, Wang M, Vaidya A. Renal outcomes
Endocrinol. 2016;4(9):739-746. PMID: 27325147 in medically and surgically treated primary aldosteronism. Hypertension.
11. Williams TA, Lenders JWM, Mulatero P, et al; Primary Aldosteronism 2018;72(3):658-666. PMID: 29987110
Surgery Outcome (PASO) Investigators. Outcomes after adrenalectomy for 28. Velema MS, de Nooijer AH, Burgers VWG, et al. Health-related quality of life
unilateral primary aldosteronism: an international consensus on outcome and mental health in primary aldosteronism: a systematic review. Horm Metab
measures and analysis of remission rates in an international cohort. Lancet Res. 2017;49(12):943-950. PMID: 29202493
Diabetes Endocrinol. 2017;5(9):689-699. PMID: 28576687 29. Wu VC, Chueh S-CJ, Chen L, et al; TAIPAI Study Group. Risk of new-onset
12. Williams TA, Burrello J, Sechi LA, et al. Computed tomography and adrenal diabetes mellitus in primary aldosteronism: a population study over 5 years. J
venous sampling in the diagnosis of unilateral primary aldosteronism. Hypertens. 2017;35(8):1698-1708. PMID: 28661412
Hypertension. 2018;72(3):641-649. PMID: 29987100 30. Rossi GP, Maiolino G, Flego A, et al; PAPY Study Investigators.
13. Riester A, Fischer E, Degenhart C, et al. Age below 40 or a recently proposed Adrenalectomy lowers incident atrial fibrillation in primary aldosteronism
clinical prediction score cannot bypass adrenal venous sampling in primary patients at long term. Hypertension. 2018;71(4):585-591. PMID: 29483224
aldosteronism. J Clin Endocrinol Metab. 2014;99(6):E1035-E1039. PMID: 31. Williams TA, Gomez-Sanchez CE, Rainey WE, et al. International
24601689 histopathology consensus for unilateral primary aldosteronism. J Clin
14. Kupers EM, Amar L, Raynaud A, Plouin P-F, Steichen O. A clinical Endocrinol Metab. 2021;106(1):42-54. PMID: 32717746
prediction score to diagnose unilateral primary aldosteronism. J Clin 32. Meyer LS, Handgriff L, Lim JS, et al. Single-center prospective cohort study
Endocrinol Metab. 2012;97(10):3530-3537. PMID: 22918872 on the histopathology, genotype, and postsurgical outcomes of patients with
15. Sze WCC, Soh LM, Lau JH, et al. Diagnosing unilateral primary primary aldosteronism. Hypertension. 2021;78(3):738-746. PMID: 34024122
aldosteronism - comparison of a clinical prediction score, computed 33. Hundemer GL, Curhan GC, Yozamp N, Wang M, Vaidya A. Cardiometabolic
tomography and adrenal venous sampling. Clin Endocrinol (Oxf). outcomes and mortality in medically treated primary aldosteronism: a
2014;81(1):25-30. PMID: 24274335 retrospective cohort study. Lancet Diabetes Endocrinol. 2018;6(1):51-59. PMID:
16. Mulatero P, di Cella SM, Monticone S, et al. 18-Hydroxycorticosterone, 29129576
18-hydroxycortisol, and 18-oxocortisol in the diagnosis of primary 34. Hundemer GL, Curhan GC, Yozamp N, Wang M, Vaidya A. Incidence of
aldosteronism and its subtypes. J Clin Endocrinol Metab. 2012;97(3):881-889. atrial fibrillation and mineralocorticoid receptor activity in patients with
PMID: 22238407 medically and surgically treated primary aldosteronism. JAMA Cardiol.
17. Satoh F, Morimoto R, Ono Y, et al. Measurement of peripheral plasma 2018;3(8):768-774. PMID: 30027227
18-oxocortisol can discriminate unilateral adenoma from bilateral diseases
ENDO 2022 • Adrenal 75
BONE AND MINERAL
METABOLISM
Denosumab Discontinuation
Bente L. Langdahl, MD, PhD, DMSc. Endocrinology and Internal Medicine, Aarhus
University Hospital, Aarhus, Denmark; E-mail:
[email protected]Learning Objectives collagen (CTX) and, in some cases, multiple
administrations of zoledronate during the first
As a result of participating in this session, learners
year after denosumab discontinuation to prevent
should be able to:
bone loss.
• Explain and discuss the pathophysiology
underlying the rebound increase in
bone turnover and bone loss following Significance of the
discontinuation of denosumab.
Clinical Problem
• Apply the current guidelines regarding
Denosumab is a fully human IgG2 monoclonal
follow-on treatment in patients discontinuing
antibody that neutralizes RANKL and thereby
denosumab.
potently decreases osteoclast recruitment and
activity, which leads to reduced bone turnover
and increased bone mineral density.1 Denosumab
reduces the risk of new vertebral fractures by 70%
Main Conclusions and the risk of hip and nonvertebral fractures by
Denosumab is an effective treatment of 40% and 20%, respectively, in postmenopausal
osteoporosis that leads to substantial increases women and is approved for the treatment of
in bone mass and clinically important reductions osteoporosis in postmenopausal women and
in fracture risk with few adverse effects. Most men and treatment of glucocorticoid-induced
patients should continue treatment; however, osteoporosis. Furthermore, the risk of adverse
some patients may need to discontinue treatment effects is low and a positive benefit-to-risk
due to comorbidities or adverse effects. ratio over 10 years has been demonstrated in
Furthermore, some patients want to discontinue postmenopausal women with osteoporosis.2
treatment because of normal or near-normal bone Denosumab is therefore a potent antiresorptive
mineral density (BMD) and absence of fractures treatment of osteoporosis; however, unlike
after years on treatment. Unlike bisphosphonates, bisphosphonates, denosumab’s effect is not
but like most other pharmacological treatments, maintained after discontinuation, as bone
denosumab has no continuing effect after it resorption increases rapidly leading to bone loss
is discontinued. Due to the accumulation of (the rebound phenomenon).3 This increase in
receptor activator of nuclear factor kappaB bone resorption increases fracture risk, especially
ligand (RANKL) and possibly osteoclasts during vertebral fractures. Post hoc analyses of data from
treatment with denosumab, a rebound activation the FREEDOM trial (phase 3 pivotal fracture trial)
of bone resorption is observed in patients who demonstrated that discontinuation of denosumab
discontinue denosumab after more than 1 to 2 treatment was associated with an increased risk
years of treatment. This can partly be prevented by of vertebral and multiple vertebral fractures
the administration of bisphosphonates, primarily compared with risk in women discontinuing
zoledronate. Patients need frequent monitoring placebo and that the risk was higher among
with measurement of C-telopeptide of type I women with prior vertebral fractures, longer
78 ENDO 2022 • Endocrine Case Management
duration of therapy, and more prominent BMD observational data are available. Pretreatment with
loss at the hip after discontinuation.4 Other bisphosphonates appears to be associated with a
studies have suggested that younger age and diminished increase in bone turnover markers
chronic kidney disease may also be risk factors in patients with osteoporosis who discontinue
for vertebral fractures after discontinuation of denosumab when compared with bone turnover
denosumab.5 markers in patients without bisphosphonate
Physicians should consider the issues exposure. However, no studies have found an
associated with discontinuation of denosumab effect of prior bisphosphonate treatment on BMD
when making the decision to initiate denosumab. loss following denosumab discontinuation.
Denosumab treatment has many benefits, Longer duration of denosumab treatment is
including prominent and continuing increases in associated with a more pronounced rebound in
BMD and easy administration, which supports bone resorption and subsequent bone loss after
adherence. However, the patient and the physician denosumab withdrawal. Therefore, duration of
should also be aware of the challenges associated denosumab treatment should be considered when
with denosumab’s discontinuation.5 interpreting the studies investigating the rebound
phenomenon and measures should be taken to
prevent or diminish this. A number of studies have
Barriers to Optimal Practice investigated the effects of other antiresorptive
drugs after denosumab discontinuation. The
• Lack of knowledge among physicians and
effects of alendronate after denosumab were
patients about the rebound phenomenon when
assessed in the DAPS study (Denosumab
denosumab is discontinued.
Adherence Preference Satisfaction). Alendronate
• Limited availability or reimbursement of for 1 year after 1 year of denosumab therapy
measuring CTX or bone turnover markers maintained the BMD gained with denosumab.6
in general or as frequently as is needed to
optimally manage patients in the first year
following denosumab discontinuation. Prevention of the Rebound
• Logistical challenges in the administration of
Phenomenon After Short-Term
zoledronate in general practitioner practices Denosumab Treatment
and/or lack of resources in the hospital setting Several studies have investigated the effects of
to see all patients discontinuing denosumab. zoledronate use after short-term denosumab
treatment. In general, zoledronate is more
effective in maintaining BMD when denosumab
Strategies for Diagnosis, has only been given for a few years. An
Therapy, and/or Management observational study of 11 women who received
zoledronate after discontinuation of denosumab
Denosumab Discontinuation for 2 years reported minimal loss of BMD gained
Bone turnover markers increase rapidly 6 months during denosumab treatment at all skeletal sites.
after the last denosumab injection and remain Similarly, a retrospective observational study
elevated until slowly decreasing to baseline of 18 postmenopausal women who received
levels after 1 to 2 years.3 BMD is lost and returns zoledronate after denosumab for 1.5 years on
to or even below pretreatment levels.3 There average showed preservation of BMD gained
is speculation whether the increase in bone during the denosumab treatment period.
turnover following denosumab discontinuation Furthermore, the initial results of the DATA-
is modified by treatment with bisphosphonates HD Extension trial indicated that a single
prior to denosumab therapy; however, only dose of zoledronate maintains the BMD gains
ENDO 2022 • Bone and Mineral Metabolism 79
achieved with a combination of teriparatide last denosumab injection found that 66% of the
and denosumab for 9 months followed by BMD gained with denosumab was retained at the
denosumab alone for 9 months at all skeletal spine and 49% was retrained at the hip 2 years
sites. A randomized controlled trial investigated after denosumab discontinuation. The bone
the efficacy of a single intravenous infusion of loss occurred within the first 18 months after
zoledronate in 57 postmenopausal women who zoledronate infusion.9
had been treated with denosumab for a mean The effect of bisphosphonates on the rebound
duration of 2.4 years and had achieved BMD phenomenon was speculated to be better if
in the osteopenic range. At 24 and 36 months, given when the increase in resorption was
BMD at the lumbar spine and femoral neck was already evident. To investigate this further, we
maintained. Patients treated with zoledronate conducted the ZOLARMAB study, a randomized
had small but significant increases in procollagen controlled trial evaluating zoledronate treatment
type 1 aminoterminal propeptide and CTX after discontinuation of long-term denosumab.10
during the first year but stabilization thereafter.7 The study included 61 postmenopausal women
Based on these results, it seems that the rebound and elderly men who had received denosumab
phenomenon after treatment with denosumab for for a mean duration of 4.6 years. The patients
up to 2.5 years can be controlled by administration were randomly assigned to receive a single 5-mg
of bisphosphonates, with most of the evidence zoledronate infusion either 6 months after the
focused on the use of zoledronate. last denosumab injection (6-month group, n =
20), 9 months after (9-month group, n = 20), or
when bone turnover markers were increased
Prevention of the Rebound
(observation group, n = 21). Patients in the 2 latter
Phenomenon After Long- groups were monitored monthly and promptly
Term Denosumab received an infusion of zoledronate if CTX
Several studies have investigated the effects of increased substantially (50% above the range for
antiresorptive drugs after discontinuation of postmenopausal women and elderly men) or if
denosumab following more than 2.5 years of they sustained a fragility fracture. Twelve months
treatment. Small observational studies and case after the initial zoledronate infusion, BMD had
reports seem to indicate that selective estrogen decreased significantly at the lumbar spine, total
receptor modulators and oral bisphosphonates do hip, and femoral neck in all 3 groups without
not prevent bone loss following discontinuation differences between groups. The decline in BMD
of long-term denosumab treatment.5 Most was more rapid in the 9-month group and in the
studies show that zoledronate is less effective observation group, which may at least in theory
in preventing BMD loss when the denosumab result in a higher fracture risk. Two women, both
treatment period exceeds 2.5 years compared with in the 9-month group, experienced a vertebral
a shorter duration of denosumab therapy. In a fracture. In addition, 10 patients in the 6-month
case series of 6 women receiving a single infusion group needed retreatment with zoledronate 6 or
of 5 mg zoledronate after 7 years of denosumab, 12 months after the initial treatment compared
a partial preservation of the BMD gained during with 5 patients in each of the 2 other groups.
denosumab treatment was observed at the spine, Based on these findings, we concluded that
while a complete loss of the bone gain was noted treatment with zoledronate, regardless of timing,
at the hip 2 years after zoledronate infusion.8 did not fully prevent BMD loss in patients with
An observational study of 120 postmenopausal osteopenia who had been treated with denosumab
women with osteoporosis treated for 2 to for 4.6 years.10
5 years with denosumab who received a single Similar to the findings in the ZOLARMAB
infusion of 5 mg zoledronate 6 months after the study, an observational study of 20 postmenopausal
80 ENDO 2022 • Endocrine Case Management
women who received a single 5-mg infusion of suggested as a possible treatment target. Hip
zoledronate after discontinuing denosumab when BMD between –2.5 and –1.5 has been proposed
bone turnover markers exceeded the upper limit by different working groups and societies.
of the premenopausal reference ranges also found However, a BMD-only–based concept is not
a secondary increase in bone turnover markers universally embraced and more data are needed.
after an initial decrease following zoledronate In addition, it is important to remember that the
administration. This suggests that a single infusion concept of a limited duration of treatment for
of zoledronate may not be sufficient to completely osteoporosis was developed based on the retention
control bone turnover in all patients discontinuing of bisphosphonates in bone and therefore cannot
long-term denosumab treatment. automatically be applied to denosumab or other
The rapid reversal of the beneficial skeletal treatments. The decision about treatment duration
effects of denosumab attributed to a transient should be individualized and should, in addition
increase in bone turnover overriding pretreatment to fracture-free period and BMD, also include
status has commonly been described as the an evaluation of additional risk factors of a
“rebound phenomenon.” It has been postulated high fracture-risk profile. These risk factors are
that osteoclast precursors remaining dormant mainly linked to prevalent osteoporotic fractures
during the treatment period retain their activity and concomitant comorbidities (eg, continuous
and/or that a high RANKL-to-osteoprotegerin use of glucocorticoids or aromatase inhibitors,
ratio ensues after denosumab is cleared from diabetes, inflammatory diseases, frailty, etc). In
the patient’s circulation. Although the exact patients considered to be at high risk for fracture,
pathophysiology of the rebound phenomenon is the efficacy and safety profile of denosumab
yet to be fully elucidated, the clinical data reviewed allows for long-term treatment, with existing
above suggest that denosumab discontinuation data supporting a duration of up to 10 years.
results in increased bone turnover and bone loss Regarding serious adverse effects, the FREEDOM
and in an augmented fracture risk compared with study investigating denosumab for up to 10 years
continuation of the drug. This observation gives reported 2 atypical femur fractures and 13 cases of
rise to the following questions related to clinical osteonecrosis of the jaw.
practice: Pending data on longer treatment duration
from registries, decisions regarding treatment
• What is the optimal treatment duration beyond 10 years should be individualized, as is the
with denosumab for patients at high risk for case with most medical treatments beyond the first
fracture? few years. These considerations also underscore
• What should be done after denosumab the importance of a careful assessment of the
discontinuation? indications to start denosumab treatment in the
• How should vertebral fractures be managed first place, especially in younger patients, who may
that occur after denosumab discontinuation? be at higher risk of fractures or bone loss following
discontinuation.
Regarding elective dental procedures where
Optimal Duration of Denosumab treatment discontinuation is demanded by the
dentist, it is suggested to perform the procedure
Treatment in Patients at
approximately 5 months after the last denosumab
High Risk for Fracture injection and resume treatment as soon as the
The goal of osteoporosis treatment is to achieve lesion is healed. This suggestion is based on expert
an acceptable level of fracture risk. As increase opinion only.
in BMD is a very strong mediator of fracture
risk reduction on treatment, BMD has been
ENDO 2022 • Bone and Mineral Metabolism 81
Optimal Treatment Strategy After keeping in mind that 1 to 2 years of treatment with
Denosumab Discontinuation bisphosphonates most likely has not led to sufficient
accumulation of bisphosphonates in the bone to
Current evidence regarding use of raloxifene and prevent future bone loss and therefore age- and/or
risedronate to prevent the rebound phenomenon menopause-related bone loss is likely to resume.
is not convincing and therefore the following In patients with gastrointestinal intolerability
will focus on the use of potent bisphosphonates. of oral bisphosphonates, inadequate response to oral
Current data suggest that the duration of bisphosphonates, or a long duration of denosumab
denosumab treatment is an important determinant treatment, the European Calcified Tissue Society
of the extent of the rebound phenomenon, position statement recommends administering
which may have potential implications for the zoledronate intravenously. The timing of this
type and duration of subsequent treatment with infusion is of great importance. In the ZOLARMAB
bisphosphonates. Very little data are available study, we found very rapid increases in CTX and
to help define the optimal timing of initiation of rapid bone loss in patients with delayed infusions
oral bisphosphonates in relation to denosumab of zoledronate (9 to 12 months after the last
discontinuation. Most of the few studies available denosumab injection). In addition, case series have
describe initiation of oral bisphosphonates reported multiple vertebral fractures at very early
6 months after the last denosumab injection. time points after 1 missed denosumab injection.
The European Calcified Tissue Society position The current recommendation is therefore to
statement on denosumab discontinuation5 initiate treatment with zoledronate 6 months
suggests performing DXA 6 months after the last after the last denosumab injection and to monitor
denosumab injection and starting treatment with the effect with measurement of bone turnover
an oral bisphosphonate, preferably alendronate, markers, for example, at 3 and 6 months after the
at that time. Measuring bone turnover markers zoledronate infusion. If bone turnover markers are
3 months after initiating the oral bisphosphonate increased (ie, above the mean found in age- and
is recommended to monitor adherence and efficacy. gender-matched cohorts) repeated infusion of
Regarding bisphosphonate treatment, an zoledronate should be considered. If bone turnover
adequate response is defined as a CTX level below markers are not available for monitoring a patient,
the mean in healthy premenopausal women, a pragmatic approach could be giving a second
the exact values depending on the assays. This infusion of zoledronate 6 months after the first
recommendation is supported by data from the infusion. Similar to oral bisphosphonates, and in
ReoLaus cohort in which patients with stable cases of otherwise low fracture risk, the duration
BMD at the spine after 1 year had mean CTX values of zoledronate treatment should be tailored to the
close to the premenopausal mean, significantly lower duration of increased bone turnover, although
than the mean CTX level in patients who lost BMD. as mentioned above, the optimal duration of
In patients with adequate response and low fracture zoledronate treatment may be adjusted when more
risk, treatment with an oral bisphosphonate can be data become available.
continued for 1 to 2 years (ie, the estimated time of
the rebound upregulation of bone turnover). It is,
however, recommended to continue the assessment Management of Vertebral
of bone turnover markers, initially after another Fractures Occurring After
3 months and if stable, every 6 months to ensure Denosumab Discontinuation
that they remain within the lower half of the
No data currently support the optimal
premenopausal reference range. At 1 to 2 years, a
management of vertebral fractures occurring
reevaluation including DXA should be performed
after denosumab discontinuation. The fractures
and the decision to discontinue oral bisphosphonate
seem to be caused by the high bone turnover
treatment should align with current guidelines,
82 ENDO 2022 • Endocrine Case Management
and/or the rapid bone loss that most likely of treatment with denosumab. In patients with
also comprises an element of deterioration of low fracture risk and a shorter duration of
bone microarchitecture. Quickly counteracting denosumab treatment (ie, up to 2.5 years), the
the increased bone turnover is therefore rebound phenomenon can probably be controlled
important, and since denosumab is a very potent by treatment with alendronate for 1 to 2 years.
antiresorptive agent with the ability to suppress If treatment with alendronate is not possible,
bone turnover markers in a matter of days, prompt zoledronate can be given once and repeated if
reinitiation of denosumab treatment may be bone turnover is inappropriately high after 3 to
the best solution. Alternatively, a combination 6 months. Patients who have been treated with
of denosumab with an osteoanabolic agent (eg, denosumab for a longer period (ie, more than
teriparatide) can be prescribed to stimulate 2.5 years) or who are still at high risk for fracture
bone formation and at the same time avoid the should receive zoledronate. Bone turnover
transient but rapid decrease in BMD, especially markers can provide clinical guidance on the
at cortical sites, reported with teriparatide timing and duration of zoledronate treatment.
subsequent to denosumab therapy. This is why Patients who experience vertebral fractures after
monotherapy with teriparatide after denosumab stopping denosumab need prompt treatment to
should be discouraged at the present time. reduce the high bone turnover.
Indeed, the combination of denosumab with
teriparatide, although currently off-label in most Figure. Osteoporosis Management With Denosumab
countries, has been shown to be highly effective
in improving BMD. However, the combination
treatment period should be followed by a potent
antiresorptive therapy (ie, zoledronate) to
consolidate BMD gains. Moreover, novel data
report that romosozumab following denosumab
for 1 year also results in BMD gains, although
more modest than with romosozumab treatment
in treatment-naïve patients. Vertebroplasty has
been identified as a potential precipitating risk
factor for subsequent vertebral fractures and
should therefore be avoided in this situation.
Summary and Conclusion
Discontinuation of denosumab following Abbreviations: BPs, bisphosphonates; BTMs, bone turnover markers.
at least 2 denosumab injections carries a Reprinted from Tsourdi E, Zillikens MC, Meier C et al. J Clin Endocrinol
Metab, 2021; 106(1) © Endocrine Society.
significant risk of a rebound effect, manifesting
as rapidly increasing bone resorptive activity,
considerable loss of the bone mass gained during Clinical Case Vignettes
denosumab treatment, and an augmented risk Case
for (multiple) vertebral fractures. To limit the
consequences of this phenomenon, it is currently A 70-year-old woman is diagnosed with
recommended to continue denosumab therapy osteoporosis. Alendronate is initiated, but she
or to prescribe a potent bisphosphonate when develops upper gastrointestinal adverse effects
denosumab is discontinued (see Figure). The and her regimen is switched to denosumab
rebound phenomenon depends on the duration after a few months. She responds well and has
increases in BMD at the spine and hip of 12.9%
ENDO 2022 • Bone and Mineral Metabolism 83
and 7.6%, respectively, after 6 years of treatment. I recommended continuing denosumab; however,
She is doing well, with no fractures and no the patient was determined to discontinue, and
adverse effects, but her BMD T-scores are still she did not want another parentally administrated
low: –3.4 at the spine and –3.8 at the hip. The treatment. She was therefore started on
patient has heard about patients discontinuing alendronate 6 months after the last denosumab
bisphosphonate treatment and would like to injection. Bone turnover markers, CTX, and
discontinue denosumab. procollagen type 1 aminoterminal propeptide
increased from unmeasurable levels at the time of
Which of the following is the alendronate initiation to 150% to 200% above the
best recommendation? premenopausal reference range after 6 months.
A. Continue denosumab BMD at the spine and hip decreased by 2.6% and
B. Discontinue denosumab and monitor CTX 20.4%, respectively. The patient then accepted
zoledronate treatment.
C. Give alendronate for 1 to 2 years and monitor
Continuing denosumab (Answer A) or giving
CTX; give zoledronate if CTX is uncontrolled
zoledronate for 1 to 2 years and monitoring CTX
D. Give zoledronate for 1 to 2 years and (repeating administration of zoledronate if CTX is
monitor CTX; repeat zoledronate if CTX is uncontrolled) (Answer D) are the recommended
uncontrolled options. However, giving alendronate for 1
E. Give zoledronate 6 and 12 months after the to 2 years and monitoring CTX (and giving
last denosumab injection zoledronate if CTX is uncontrolled) (Answer C)
and giving zoledronate 6 and 12 months after the
Answers: A) Continue denosumab or D) Give
last denosumab injection (Answer E) would also
zoledronate for 1 to 2 years and monitor CTX:,
reduce the rebound phenomenon but probably less
repeat zoledronate if CTX is uncontrolled
effectively than Answer D.
References
1. Cummings SR, San Martin J, McClung MR, et al; FREEDOM Trial. 6. Freemantle N, Satram-Hoang S, Tang ET, et al; DAPS Investigators. Final
Denosumab for the prevention of fractures in postmenopausal women with results of the DAPS (Denosumab Adherence Preference Satisfaction)
osteoporosis. N Engl J Med. 2009;361(8):756-765. PMID: 19671655 study: a 24-month, randomized, crossover comparison with alendronate in
2. Ferrari S, Lewiecki EM, Butler PW, et al. Favorable skeletal benefit/risk of postmenopausal women. Osteoporos Int. 2012;23(1):317-326. PMID: 21927922
long-term denosumab therapy: a virtual-twin analysis of fractures prevented 7. Anastasilakis AD, Papapoulos SE, Polyzos SA, Appelman-Dijkstra
relative to skeletal safety events observed. Bone. 2020;134:115287. PMID: NM, Makras P. Zoledronate for the prevention of bone loss in women
32092479 discontinuing denosumab treatment. A prospective 2-year clinical trial. J Bone
3. Bone HG, Bolognese MA, Yuen CK, et al. Effects of denosumab treatment Miner Res. 2019;34(12):2220-2228. PMID: 31433518
and discontinuation on bone mineral density and bone turnover markers 8. Reid IR, Horne AM, Mihov B, Gamble GD. Bone loss after denosumab: only
in postmenopausal women with low bone mass. J Clin Endocrinol Metab. partial protection with zoledronate. Calcif Tissue Int. 2017;101(4):371-374.
2011;96(4):972-980. PMID: 21289258 PMID: 28500448
4. Cummings SR, Ferrari S, Eastell R, et al. Vertebral fractures after 9. Everts-Graber J, Reichenbach S, Ziswiler HR, Studer U, Lehmann T. A single
discontinuation of denosumab: a post hoc analysis of the randomized infusion of zoledronate in postmenopausal women following denosumab
placebo-controlled FREEDOM trial and its extension. J Bone Miner Res. discontinuation results in partial conservation of bone mass gains. J Bone
2018;33(2):190-198. PMID: 29105841 Miner Res. 2020;35(7):1207-1215. PMID: 31991007
5. Tsourdi E, Zillikens MC, Meier C, et al. Fracture risk and management of 10. Sølling AS, Harsløf T, Langdahl B. Treatment with zoledronate subsequent
discontinuation of denosumab therapy: a systematic review and position to denosumab in osteoporosis: a randomized trial. J Bone Miner Res.
statement by ECTS. J Clin Endocrinol Metab. 2021 [online ahead of print]. 2020;35(10):1858-1870. PMID: 32459005
PMID: 33103722
84 ENDO 2022 • Endocrine Case Management
Using the Best Available
Evidence to Personalize
Osteoporosis Treatment
E. Michael Lewiecki, MD. University of New Mexico Health Sciences Center, Albuquerque,
NM; E-mail:
[email protected]Micol S. Rothman, MD. University of Colorado School of Medicine, Aurora, CO; E-mail:
[email protected]
Learning Objectives medications used to treat them,5 casting into doubt
the applicability of the studies for some patients.
As a result of participating in this session, learners
Guidelines developed by experts interpreting the
should be able to:
best available medical evidence may be simpler,
• Assess fracture risk and tailor medication more intuitive, and easier to apply in clinical
choice accordingly. practice, but are open to criticism that they are
overly subjective and based on a lower level
• Identify strategies for treatment sequence with
of evidence. The application of any guideline
osteoporosis medications.
for making decisions for individual patients
• Recognize the onset and offset of treatment must consider all available clinical information.
effects of different osteoporosis medications. Individual fracture risk should be examined when
making treatment decisions. Additionally, prior
medication exposure and duration of treatment,
as well as patient preference, must be taken into
Main Conclusions account when thinking about future therapies.
Many clinical practice guidelines address the
management of patients with osteoporosis.1-4
These guidelines are often developed by societies Significance of the
or organizations that focus on specific patient
populations using divergent approaches to
Clinical Problem
analyzing and interpreting the medical evidence, Osteoporosis is a common disorder characterized
with or without consideration of cost-effectiveness, by low bone density and poor bone quality
and with differing views on inclusion of expert resulting in increased risk of fractures. Despite the
opinion. Guidelines that rely exclusively on availability of excellent tools to assess fracture risk
data derived from large, randomized, placebo- and approved medications that reduce fracture
controlled clinical trials are scientifically rigorous risk, most patients at risk for fractures are not
but tend to be restrictive, complex, and lacking being treated.6 The large treatment gap is now
the flexibility needed to address the needs of some recognized as a global crisis in the care of patients
patients. Many or most patients seen in clinical with osteoporosis.7
practice would not qualify for participation in
the clinical trials supporting the approval of the
ENDO 2022 • Bone and Mineral Metabolism 85
Barriers to Optimal Practice daily. He reports no fractures, but old chest x-rays
suggest prior compression fractures. Workup
Many factors contribute to the treatment gap,
for secondary causes, including vitamin D
including fear of medication adverse effects, poor
measurement, reveals no additional abnormalities
understanding of the balance of benefits and
other than a low morning testosterone
risks with treatment, inadequate communication
concentration of 250 ng/dL (8.7 nmol/L).
skills to quantify risk, lack of appreciation of the
potentially serious consequences of fractures, Which of the following treatments
failure to recognize that a prior fracture is due would best reduce this patient’s
to osteoporosis, reductions in reimbursement future risk of vertebral fracture?
for bone density testing (in the United States)
A. Calcium and vitamin D, weight-bearing
resulting in the closure of many office-based DXA
exercise as tolerated, and steroid taper (discuss
facilities, conflicting clinical practice guidelines,
with transplant team)
limited available time for physician-patient
encounters, and competing health care priorities. B. Subcutaneous teriparatide
C. Weekly oral alendronate
Strategies for Diagnosis, D. Weekly testosterone injections
Therapy, and/or Management Answer: B) Subcutaneous teriparatide
Many strategies to reduce the treatment gap This patient has severe osteoporosis with
have been proposed,8 including technology- very high fracture risk. Prevention of further
enabled collaborative learning, the prototype of compression fractures could be crucial to protect
which is Bone Health TeleECHO.9 Tools and his lung function. Pharmacologic therapy is
algorithms have been developed to help patients indicated (thus, Answer A is incorrect).
and providers alike better quantify the risks Given his low testosterone level, he should
and benefits of osteoporosis therapy. However, be worked up for true hypogonadism. However,
providers must recognize that there is not a “one even if further workup indicates he would benefit
size fits all” answer for many patients. At times, from testosterone replacement, there are no data
more than 1 visit and a variety of materials may be regarding the effect of testosterone on fracture risk
needed for patients to understand their diagnosis (thus, Answer D is incorrect). Endocrine Society
of osteoporosis and feel comfortable with guidelines suggest that hypogonadal men who are
initiation of medication. Clinical judgment, patient at high risk of fracture be treated with an agent with
preference, and other factors are key aspects of fracture-proven efficacy and that testosterone should
managing complex cases of osteoporosis. only be given “in lieu of a bone drug” if serum levels
are below 200 ng/dL (<6.9 nmol/L) on more than
Clinical Case Vignettes 1 occasion with symptoms of androgen deficiency
and if the fracture risk is borderline-high.10
Case 1 An 18-month, randomized, head-to-head trial
A 54-year-old man with a history of lung in 414 patients comparing teriparatide (Answer
transplant for chronic obstructive pulmonary B) to alendronate (Answer C) in patients with
disorder 1 year ago presents after DXA shows a glucocorticoid-induced osteoporosis showed larger
lowest T-score of –3.1 in the lumbar spine. He gains in spine bone mineral density (7.2% vs 3.4%)
had been treated with prednisone bursts prior and lower rates of vertebral compression fracture
to his lung transplant, has had several episodes (0.6 vs 6.1%) in the teriparatide-treated group.11
of rejection that required pulse doses of steroids, Although oral alendronate is a reasonable
and remains on a prednisone dosage of 10 mg choice if teriparatide is not financially feasible,
86 ENDO 2022 • Endocrine Case Management
is contraindicated, or is otherwise not desired not exist for any other osteoporosis therapy. We
by the patient, the strongest data for secondary know that stopping denosumab can lead to rapid
fracture prevention in this scenario would be rise in bone turnover markers, rapid decrease
with teriparatide. The American College of in bone mineral density, return of fracture risk
Rheumatology guidelines suggest alendronate to baseline, and possible increase in the risk of
for first-line therapy based on cost, while other multiple vertebral fractures.14 Ideally, there would
guidelines from Europe advocate for teriparatide have been discussion with the oral surgeon before
as first-line therapy in a patient at high risk.12 the tooth extraction to discuss the possibilities of a
Note that other anabolic agents (abaloparatide short delay of therapy, changing to an alternative
and romosozumab) are not US FDA-approved for medication, or performing extraction without
osteoporosis in men at this time. delaying or stopping denosumab.
When patients present with vertebral fracture
in the setting of stopping denosumab, kyphoplasty
Case 2
has not been shown to be beneficial and may,
A 67-year-old woman with a history of a in fact, lead to further compression fractures
Colles fracture at age 63 years was treated with (therefore, Answer B is correct, as this step should
denosumab for 4 years. On the advice of her NOT be advised now).15
oral surgeon, she stopped denosumab before It is crucial for this patient to resume some
having a tooth extraction 10 month ago and has type of osteoporosis therapy; however, there are
not resumed treatment since then. She returns multiple options as outlined in the remaining
for follow-up after a recent hospitalization for answer choices. If the patient has done well
multiple painful vertebral compression fractures. on denosumab, resumption of this medication
She has normal kidney function and no history of (Answer C) is reasonable. If she cannot commit
cardiovascular disease or malignancy. to resuming therapy every 6 months, giving
intravenous zoledronic acid (Answer A) could
Which of the following also be an option considering its antifracture
should NOT be advised? benefits and ease of administration. Although
A. Administer intravenous zoledronic acid there are concerns about bone loss when switching
B. Consult with interventional radiology for from denosumab to an anabolic agent,16 given
kyphoplasty this patient’s compression fractures and possible
C. Resume denosumab now need for ongoing dental work, anabolic therapy
D. Start anabolic therapy with teriparatide, (Answer D) could be an option as well. This
abaloparatide, or romosozumab would perhaps be best done in combination with
antiresorptive therapy.15
Answer: B) Consult with interventional
radiology for kyphoplasty Case 3
There has been much recent discussion of how A 56-year-old woman presents after recently
and when to stop/bridge denosumab if needed or stopping menopausal hormone therapy with patch
desired. First, it must be emphasized that there estradiol and daily progestin. She has been on this
is no role for a drug holiday with denosumab. therapy since the onset of menopausal symptoms
After stopping denosumab, treatment effect will in her early 50s. To her knowledge, she has never
be rapidly lost if another agent is not started. The had a fracture. She is not having hot flashes or
properties of bisphosphonates are unique in that other bothersome menopausal symptoms, but she
they are slowly released from the bone, providing worries about bone loss because her mother had
continuing protection for variable amounts of a hip fracture at age 78 years. Recent DXA shows
time after administration is stopped.13 This does
ENDO 2022 • Bone and Mineral Metabolism 87
T-scores of –2.1 in the spine and –1.8 in the femoral option would not be advised at this time.
neck. She wants to know how best to mitigate bone However, studies looking at bone mineral density
loss in the setting of stopping estrogen. and bone turnover markers confirm a rapid return
to high bone turnover marker levels, a different
Which of the following profile than that observed after stopping long-
statements is most correct? term bisphosphonate therapy, where a slow rise
A. A bisphosphonate could provide skeletal benefit of bone turnover markers can be expected.17,18
B. If spine imaging shows a previously Therefore, stopping estrogen would likely lead to
unrecognized vertebral fracture, she should an increase in serum C-telopeptide (thus, Answer
resume estrogen D is incorrect).
In a healthy patient with osteopenia and
C. She should avoid weight training due to her
no known fractures, weight training is not
low spine bone mineral density
contraindicated (thus, Answer C is incorrect).
D. Stopping estrogen will most likely decrease The latest Endocrine Society guidelines for
serum C-telopeptide, a marker of bone turnover postmenopausal osteoporosis suggest starting
Answer: A) A bisphosphonate could alternative treatments when estrogen is stopped
provide skeletal benefit in women with osteoporosis,13 but this does
not cover women at lower risk, such as this
This patient’s current fracture risk is likely low. patient. However, bisphosphonates can benefit
However, if imaging were to show compression postmenopausal women with osteopenia.19,20
fractures, a more aggressive option should be Given this patient’s concerns based on family
considered (thus, Answer B is incorrect). history, bisphosphonate therapy (Answer A) could
Since she has stopped menopausal hormone be an attractive option.21
therapy without symptoms, returning to this
References
1. Eastell R, Rosen CJ, Black DM, Cheung AM, Murad MH, Shoback D. 9. Lewiecki EM, Boyle JF, Arora S, Bouchonville MF 2nd, Chafey DH.
Pharmacological management of osteoporosis in postmenopausal women: Telementoring: a novel approach to reducing the osteoporosis treatment gap.
an Endocrine Society clinical practice guideline. J Clin Endocrinol Metab. Osteoporos Int. 2017;28(1):407-411. PMID: 27439373
2019;104(5):1595-1622. PMID: 30907953 10. Watts NB, Adler RA, Bilezikian JP, et al; Endocrine Society. Osteoporosis in
2. Cosman F, de Beur SJ, LeBoff MS, et al; National Osteoporosis Foundation. men: an Endocrine Society clinical practice guideline. J Clin Endocrinol Metab.
Clinician’s guide to prevention and treatment of osteoporosis. Osteoporos Int. 2012;97(6):1802-1822. PMID: 22675062
2014;25(10):2359-2381. PMID: 25182228 11. Saag KG, Shane E, Boonen S, et al. Teriparatide or alendronate in
3. Camacho PM, Petak SM, Binkley N, et al. American Association of Clinical glucocorticoid-induced osteoporosis. N Engl J Med. 2007;357(20):2028-2039.
Endocrinologists and American College of Endocrinology clinical practice PMID: 18003959
guidelines for the diagnosis and treatment of postmenopausal osteoporosis 12. Rothman MS, Olenginski TP, Stanciu I, Krohn K, Lewiecki EM. Lessons
- 2016--executive summary. Endocr Pract. 2016;22(9):1111-1118. PMID: learned with Bone Health TeleECHO: making treatment decisions when
27643923 guidelines conflict. Osteoporos Int. 2019;30(12):2401-2406. PMID: 31471665
4. Qaseem A, Forciea MA, McLean RM, Denberg TD, et al; Clinical Guidelines 13. Eastell R, Rosen CJ, Black DM, Cheung AM, Murad MH, Shoback D.
Committee of the American College of Physicians. Treatment of low bone Pharmacological management of osteoporosis in postmenopausal women:
density or osteoporosis to prevent fractures in men and women: a clinical an Endocrine Society* clinical practice guideline. J Clin Endocrinol Metab.
practice guideline update from the American College of Physicians. Ann Intern 2019;104(5):1595-1622. PMID: 30907953
Med. 2017;166(11):818-839. PMID: 28492856 14. Anastasilakis AD, Polyzos SA, Makras P, Aubry-Rozier B, Kaouri S, Lamy O.
5. Dowd R, Recker RR, Heaney RP. Study subjects and ordinary patients. Clinical features of 24 patients with rebound-associated vertebral fractures
Osteoporos Int. 2000;11(6):533-536. PMID: 10982170 after denosumab discontinuation: systematic review and additional cases. J
6. Miller PD. Underdiagnosis and undertreatment of osteoporosis: the battle to Bone Miner Res. 2017;32(6):1291-1296. PMID: 28240371
be won. J Clin Endocrinol Metab. 2016;101(3):852-859. PMID: 26909798 15. Tsourdi E, Zillikens MC, Meier C, et al. Fracture risk and management of
7. Khosla S, Shane E. A crisis in the treatment of osteoporosis. J Bone Miner Res. discontinuation of denosumab therapy: a systematic review and position
2016;31(8):1485-1487. PMID: 27335158 statement by ECTS. J Clin Endocrinol Metab. 2020;dgaa756. PMID: 33103722
8. Khosla S, Cauley JA, Compston J, et al. Addressing the crisis in the treatment 16. Leder BZ, Tsai JN, Uihlein AV, et al. Denosumab and teriparatide transitions
of osteoporosis: a path forward. J Bone Miner Res. 2017;32(3):424-430. PMID: in postmenopausal osteoporosis (the DATA-Switch study): extension of
28099754 a randomised controlled trial. Lancet. 2015;386(9999):1147-1155. PMID:
26144908
88 ENDO 2022 • Endocrine Case Management
17. Greenspan SL, Emkey RD, Bone HG, Wet al. Significant differential effects of 20. Quandt SA, Thompson DE, Schneider DL, Nevitt MC, Black DM, Fracture
alendronate, estrogen, or combination therapy on the rate of bone loss after Intervention Trial Research G. Effect of alendronate on vertebral fracture
discontinuation of treatment of postmenopausal osteoporosis. A randomized, risk in women with bone mineral density T scores of -1.6 to -2.5 at the
double-blind, placebo-controlled trial. Ann Intern Med. 2002;137(11):875-883. femoral neck: the Fracture Intervention Trial. Mayo Clin Proc. 2005;80(3):343-
PMID: 12458987 349. PMID: 15757015
18. Wasnich RD, Bagger YZ, Hosking DJ, et al;Early Postmenopausal 21. Rothman MS, Olenginski TP, Stanciu I, Krohn K, Lewiecki EM. Lessons
Intervention Cohort Study Group. Changes in bone density and turnover learned with Bone Health TeleECHO: making treatment decisions when
after alendronate or estrogen withdrawal. Menopause. 2004;11(6 Pt 1):622- guidelines conflict. Osteoporos Int. 2019;30(12):2401-2406. PMID: 31471665
630. PMID: 15545790
19. Reid IR, Brown JP, Burckhardt P, et al. Intravenous zoledronic acid in
postmenopausal women with low bone mineral density. N Engl J Med.
2002;346(9):653-661. PMID: 11870242
ENDO 2022 • Bone and Mineral Metabolism 89
CARDIOVASCULAR
ENDOCRINOLOGY
Choosing Cholesterol-
Lowering Medications in
Patients With Liver Disease
Savitha Subramanian, MD. Department of Medicine, Division of Metabolism,
Endocrinology, and Nutrition, University of Seattle, Seattle, WA; E-mail:
[email protected]Learning Objectives liver disease. Lipoprotein X is an abnormal
lipoprotein particle containing free cholesterol
As a result of participating in this session, learners
often observed in acquired cholestatic liver
should be able to:
dysfunction. Skin xanthomas may arise and
• Appropriately stratify risk and manage patients marked hypercholesterolemia occurs in persons
with dyslipidemia in the setting of liver disease. with lipoprotein X. Treatment of the underlying
liver dysfunction, often requiring liver transplant,
• Initiate appropriate lipid-lowering therapy in
resolves abnormal lipoprotein X. Dyslipidemia that
patients with liver function test abnormalities.
occurs after liver transplant should be promptly
addressed with appropriate lipid-lowering therapy.
Main Conclusions
Mild transaminase elevations are a common
Significance of the
occurrence in persons with nonalcoholic/ Clinical Problem
metabolic fatty liver disease (NAFLD or recently Dyslipidemia is a common occurrence in
referred to as MAFLD), where hepatic steatosis individuals with NAFLD, and it drives
is associated with metabolic dysfunction such cardiovascular risk. NAFLD is increasing in
as insulin resistance, metabolic syndrome, prevalence and is currently estimated to affect
and type 2 diabetes mellitus. Statin therapy is nearly 25% of adults worldwide and is the leading
indicated for treatment of cardiovascular risk cause of liver disease. It is closely associated with
reduction in patients with NAFLD. Statin therapy other metabolic disorders such as obesity and type
decreases LDL-cholesterol levels and can improve 2 diabetes, and its prevalence is much higher in
transaminitis.1 Ezetimibe may also be used to persons with these conditions. An atherogenic
lower LDL cholesterol, but cardiovascular benefits dyslipidemia is often observed; NAFLD is now
in this population are still unknown. Fibrates can considered a cardiovascular risk factor, and risk
be used to treat moderate hypertriglyceridemia mitigation strategies are essential. However,
and are safe and usually well tolerated in patients individuals with NAFLD often have elevated liver
with NAFLD. However, evidence that treatment enzymes, and there is still clinician hesitancy in
of dyslipidemia with any lipid-lowering agent starting and/or continuing lipid-lowering therapy,
improves histology or liver-related outcomes in especially statins. Additionally, guidelines on lipid
patients with steatohepatitis is currently lacking. management in NAFLD are lacking.
Dyslipidemia is also described in patients The liver has a central role in cholesterol
with other liver disorders such as cholestatic metabolism, including cholesterol biosynthesis,
92 ENDO 2022 • Endocrine Case Management
storage, and excretion. Therefore, liver disease can moderate-to-advanced liver fibrosis compared
affect cholesterol levels, depending on the type of with what is observed in those without diabetes.
liver injury (eg, parenchymal, cholestatic, or mixed). Therefore, dyslipidemia of NAFLD should be
Hypercholesterolemia, with a relative increase of addressed in a timely manner by initiating statin
free cholesterol, is an early and peculiar metabolic therapy.3
alteration in cholestatic liver disease. Lipoprotein The liver has a central role in lipoprotein
composition and metabolism are altered, and an metabolism, as it is involved in the synthesis and/
anomalous lipoprotein called lipoprotein X is or clearance of all lipoproteins.4 Hepatocytes, a
present. Very high cholesterol levels (>1000 mg/dL major site of cholesterol metabolism, are a main
[>25.90 mmol/L]) levels can occur. This condition target of statins. Alanine aminotransferase elevations
is often underrecognized in clinical practice. occur independently of statins in patients with
NAFLD. Several studies have established the safety
of statins in patients with a baseline elevation in
Barriers to Optimal Practice liver enzymes. Statins decrease hepatic transaminases
in NAFLD. Risk of statin-induced hepatotoxicity
• Hesitancy using appropriate lipid-lowering
is not increased in individuals with NAFLD or
medication in the setting of transaminitis
NASH, and serious liver injury from statin therapy
associated with NAFLD and NASH.
is very rare in clinical practice. Statins should be
• Recognition of the underlying etiology leading avoided in patients with decompensated cirrhosis,
to dyslipidemia in liver disease. as there are very few safety data. Periodic monitoring
of serum alanine aminotransferase levels does not
detect or prevent serious liver injury due to statin
Strategies for Diagnosis, use and is therefore not recommended.5 Ezetimibe is
Therapy, and/or Management also associated with LDL-cholesterol reductions in
NAFLD; improvements in liver function have also
Lipid Disorders in NAFLD been observed. Effects on steatosis and NAFLD
Diagnosis activity score appear to be mixed. A post hoc analysis
Atherogenic dyslipidemia characterized by mild of the IMPROVE-IT study showed that patients
to moderate hypertriglyceridemia with low classified as being at high risk for liver fibrosis using
HDL cholesterol and increased small dense LDL the NAFLD fibrosis score showed greater reductions
particles is commonly observed. in recurrent cardiovascular disease with simvastatin
and ezetimibe compared with those classified as
Treatment being at low risk. PCSK9 inhibitors show promise
Cardiovascular morbidity and mortality are in decreasing intrahepatic fat in patients with
increased in patients with NAFLD. Metabolic NAFLD but further evidence is needed.
derangements are the primary contributor to
increased cardiovascular risk in these individuals.
NAFLD is associated with increased risk of fatal Lipid Disorders in Other
and nonfatal cardiovascular events, as well as other Liver Diseases
cardiac complications, such as cardiomyopathy, Cholestatic Liver Disease
cardiac valvular calcification, and cardiac The liver converts cholesterol to bile acids and
arrhythmias. This increased risk is independent removes free cholesterol via biliary excretion.
of traditional cardiovascular risk factors and is Cholestatic liver disease (primary biliary
proportional to the severity of liver disease.2 In cirrhosis or primary sclerosing cholangitis) due
individuals with type 2 diabetes and NALFD, to impaired bile flow results in accumulation of
atherogenic dyslipidemia is associated with bile acids and other metabolites in the liver and
ENDO 2022 • Cardiovascular Endocrinology 93
systemic circulation. In the setting of cholestatic be atherogenic. It is estimated, however, that about
liver disease, a unique, abnormal lipoprotein 10% of patients with primary biliary cirrhosis have a
with similar density to LDL but without its significant risk of cardiovascular disease and should
surface protein apolipoprotein B appears in the be treated with medications to reduce that risk.
circulation.6 These lipoproteins contain only
phospholipid and unesterified cholesterol and Dyslipidemia After Liver Transplant
are referred to as lipoprotein X. In cholestasis, Dyslipidemia is common following liver transplant,
there is impaired bile flow; it is believed that these occurring in more than 60% of transplant recipients.
lipoprotein-like lipoprotein X particles are due Posttransplant dyslipidemia is commonly observed in
to reflux of biliary lipoproteins into the systemic patients with higher BMI or type 2 diabetes before
circulation due to cholestasis. transplant; it can also develop in the absence of these
Extreme cholesterol elevations can occur comorbid conditions. Use of immunosuppressant
and result in artifactual abnormalities in medications, including calcineurin inhibitors and
electrolytes (hyponatremia) and hyperviscosity. mTOR inhibitor sirolimus, increase the risk of
Xanthomas can develop in the palms on occasion. dyslipidemia. Lipids should be regularly assessed in
Neuropathic symptoms can occur because of large liver transplant recipients.
xanthomas. Such striking cholesterol elevations Liver transplant recipients have increased
are reflective of significant liver dysfunction, and cardiovascular risk, especially those with features
hypercholesterolemia resolves only after liver of the metabolic syndrome or diabetes. Liver graft
dysfunction resolves. steatosis can occur. Statins are mostly safe to use in
Statins, ezetimibe, and fibrates have all been the posttransplant setting. Calcineurin inhibitors
shown to be safe in patients with cholestatic liver and several statins are metabolized by cytochrome
disease but are of limited benefit when serum P450 3A4, and concurrent use of higher dosages
cholesterol levels are markedly elevated. Blood may increase the risk of statin-associated myopathy.7
filtering using plasma exchange can be used as a
temporizing measure to decrease cholesterol levels;
only correction of underlying liver pathology (often
with liver transplant) results in complete reversal of
lipid abnormalities. Lipoprotein X is not believed to
Total HDL LDL Fasting plasma Hemoglobin
Time point cholesterol Triglycerides cholesterol cholesterol glucose A1c AST ALT
5 years ago 253 mg/dL 293 mg/dL 41 mg/dL 153 mg/dL 106 mg/dL 5.6% 22 U/L 61 U/L
(SI: 6.55 (SI: 3.31 (SI: 1.06 (SI: 3.96 (SI: 5.88 (38 mmol/mol) (SI: 0.37 (SI: 1.02
mmol/L) mmol/L) mmol/L) mmol/L) mmol/L) µkat/L) µkat/L)
5 years ago 274 mg/dL 275 mg/dL 40 mg/dL 179 mg/dL 130 mg/dL 6.2% 26 U/L 47 U/L
(atorvastatin, (SI: 7.10 (SI: 3.11 (SI: 1.04 (SI: 4.64 (SI: 7.22 (44 mmol/mol) (SI: 0.43 (SI: 0.78
10 mg daily, mmol/L) mmol/L) mmol/L) mmol/L) mmol/L) µkat/L) µkat/L)
started)
3 years ago 196 mg/dL 377 mg/dL 36 mg/dL 85 mg/dL 143 mg/dL 6.8% 23 U/L 64 U/L
(SI: 5.08 (SI: 4.26 (SI: 0.93 (SI: 2.20 (SI: 7.94 (51 mmol/mol) (SI: 0.38 (SI: 1.07
mmol/L) mmol/L) mmol/L) mmol/L) mmol/L) µkat/L) µkat/L)
Most recent 181 mg/dL 422 mg/dL 39 mg/dL … 150 mg/dL 7.1% 47 U/L 86 U/L
(SI: 4.69 (SI: 4.77 (SI: 1.01 (SI: 8.33 (54 mmol/mol) (SI: 0.78 (SI: 1.44
mmol/L) mmol/L) mmol/L) mmol/L) µkat/L) µkat/L)
Abbreviations: ALT, alanine aminotransferase; AST, aspartate aminotransferase.
Reference ranges: fasting glucose, 70-99 mg/dL (SI: 3.9-5.5 mmol/L); hemoglobin A1c, 4.0%-5.6% (20-38 mmol/mol); ALT, 10-40 U/L (SI: 0.17-0.67 µkat/L);
AST, 20-48 U/L (SI: 0.33-0.80 µkat/L).
94 ENDO 2022 • Endocrine Case Management
Clinical Case Vignettes levels are likely a reflection of recent-onset
diabetes corresponding to weight gain. There
Case 1 is no indication to discontinue atorvastatin
A 44-year-old woman is referred for management (Answer A) at this time. Although increasing the
of dyslipidemia and gradually increasing elevated atorvastatin dosage to 40 mg daily (Answer B) is
liver enzymes first noted about 5 years ago. At that not necessarily wrong, it is not the best option,
time, atorvastatin (10 mg daily) was initiated, and as she had good LDL-cholesterol lowering on
she has continued this regimen. the dosage of 10 mg daily. Statins are not good
Laboratory test results are shown in the Table triglyceride-lowering agents.
(previous page). Instead, maximizing metformin therapy
She followed up with her primary care (Answer C) is a better option due to rising
provider for a few years. Recently, her hemoglobin hemoglobin A1c values.
A1c level has increased, type 2 diabetes has been Additionally, intermittent scanned continuous
diagnosed, and metformin has been initiated at a glucose monitoring (Answer D), if covered by
dosage of 500 mg twice daily. She reports a weight insurance, is acceptable. This patient specifically
gain of 15 lb (6.8 kg) in the past 18 months. Her came to clinic requesting use of intermittent
weight continues to increase, and physical activity scanned continuous glucose monitoring. She
levels have dropped. Abdominal ultrasonography incorporated lifestyle efforts and was able to
obtained as part of the evaluation for elevated liver decrease her hemoglobin A1c level to 6.1%
enzymes reveals hepatic steatosis. Her most recent (43 mmol/mol). Liver transaminase levels
hemoglobin A1c level is 7.5% (58 mmol/mol). decreased to what was observed 7 years prior.
There is a family history of premature Therefore, both Answers C and D are correct.
atherosclerotic cardiovascular disease in her mother,
and many family members have type 2 diabetes. She
Case 2
does not drink more than 1 alcoholic beverage a week.
She is commercially insured through her employer. A 24-year-old man with a diagnosis of primary
She is concerned about her liver test results sclerosing cholangitis is referred from hepatology
and seeks your recommendation. for management of very high cholesterol levels
and bumps on his elbows and palms.
Which of the following is the best next step?
Laboratory test results:
A. Discontinue atorvastatin and start fenofibrate
Total cholesterol = 1382 mg/dL (<200 mg/dL
B. Increase the atorvastatin dosage to 40 mg daily [optimal]) (SI: 35.79 mmol/L [<5.18 mmol/L])
C. Increase the metformin dosage to 1000 mg Triglycerides = 141 mg/dL (<150 mg/dL
twice daily [optimal]) (SI: 1.59 mmol/L [<1.70 mmol/L])
LDL cholesterol = 1347 mg/dL (<100 mg/dL
D. Start intermittent scanned glucose monitoring [optimal]) (SI: 34.89 mmol/L [mmol/L])
HDL cholesterol = 7 mg/dL (>60 mg/dL
Answer: C and D): Increase the metformin [optimal]) (SI: 0.18 mmol/L [>1.55 mmol/L])
dosage to 1000 mg twice daily (C) and start Serum sodium = measurements range from 129 to
intermittent scanned glucose monitoring (D) 133 mEq/L (129 to 133 mmol/L)
Alanine aminotransferase = 255 U/L (10-40 U/L)
This patient has a history of mild liver (SI: 4.26 µkat/L [0.17-0.67 µkat/L])
transaminase elevations, which have recently Aspartate aminotransferase = 298 U/L (20-48 U/L)
worsened. Low-intensity atorvastatin is safe and (SI: 4.98 µkat/L [0.33-0.80 µkat/L])
Total bilirubin = 6.0 mg/dL (0.3-1.2 mg/dL)
has been effective in lowering this woman’s LDL (SI: 102.6 µmol/L [5.1-20.5 µmol/L])
cholesterol. However, recent elevation in aspartate
aminotransferase and alanine aminotransferase
ENDO 2022 • Cardiovascular Endocrinology 95
Which of the following is the best Total cholesterol = 219 mg/dL (SI: 5.67 mmol/L)
next step in this patient’s care? Triglycerides = 178 mg/dL (SI: 2.01 mmol/L)
LDL cholesterol = 155 mg/dL (SI: 4.01 mmol/L)
A. Refer for LDL apheresis
B. Refer for therapeutic plasma exchange Therefore, the best answer is none of the above
C. Start colesevelam (Answer E).
D. Start pravastatin
E. None of the above Case 3
A 62-year-old woman is referred from hepatology.
Answer: E) None of the above She underwent liver transplant for NASH-related
In cholestatic liver disease, lipoprotein X (LDL- cirrhosis. At the 1-year postliver transplant biopsy,
like particles lacking apolipoprotein B on the macrovesicular steatosis was noted, and she was
surface) can be present in the circulation, occurring subsequently referred for treatment options. Her
in up to 45% of patients. Extreme cholesterol history is notable for hypertension and type 2
elevations can occur with resultant artifactual diabetes (managed with lifestyle efforts) prior to
abnormalities in electrolytes (hyponatremia) and transplant, as well as longstanding hyperlipidemia
hyperviscosity. Xanthomas can occur in the palms. with intolerance to statins. Multiple statins were
While statins (Answer D) seem to be safe tried over the years with significant muscle
in individuals with cholestatic liver disease, it is adverse effects. There is also a history of possible
highly unlikely statins will result in significant transient ischemic attack 3 years before transplant.
LDL-cholesterol reduction in a patient with such Currently, her immunosuppression regimen
marked hypercholesteremia. includes tacrolimus. She also takes ezetimibe
Colesevelam (Answer C) is a bile acid resin without issues. Metformin was recently added due
with only modest cholesterol-lowering ability. to rising hemoglobin A1c values.
LDL apheresis (Answer A), which physically Laboratory test results are shown in the Table.
removes LDL particles (by virtue of the presence
of apolipoprotein B on LDL) from the blood, will Which of the following therapies
not clear lipoprotein X, which lacks apolipoprotein should be added at this time?
B on its surface.
A. Bempedoic acid
Plasma exchange (Answer B) can be
useful in decreasing cholesterol levels, but B. Evolocumab
it is a temporizing measure. C. Icosapent ethyl
This patient underwent orthotopic liver D. Fenofibrate
transplant, which normalized lipid levels:
Time point Total cholesterol Triglycerides HDL cholesterol LDL cholesterol Hemoglobin A1c
Pretransplant (started 190 mg/dL 82 mg/dL 59 mg/dL 115 mg/dL 6.2%
ezetimibe, 10 mg daily) (SI: 4.92 mmol/L) (SI: 0.93 mmol/L) (SI: 1.53 mmol/L) (SI: 2.98 mmol/L) (44 mmol/mol)
1 year after transplant 195 mg/dL 430 mg/dL 54 mg/dL 141 mg/dL 6.6%
(continued ezetimibe, (SI: 5.05 mmol/L) (SI: 4.86 mmol/L) (SI: 1.40 mmol/L) (SI: 3.65 mmol/L) (49 mmol/mol)
10 mg daily, added
metformin)
After transplant; most 252 mg/dL 357 mg/dL 63 mg/dL 118 mg/dL 6.7%
recent measurements (SI: 6.53 mmol/L) (SI: 4.03 mmol/L) (SI: 1.63 mmol/L) (SI: 3.06 mmol/L) (50 mmol/mol)
(continued, ezetimibe,
10 mg daily)
Reference range: hemoglobin A1c, 4.0%-5.6% (20-38 mmol/mol).
96 ENDO 2022 • Endocrine Case Management
Answer: B) Evolocumab into an active drug by a specific isozyme that is
not present in skeletal muscle and can be used
Evolocumab (Answer B) is a PCSK9 inhibitor in patients with statin intolerance due to muscle
that can be used in individuals at high risk for adverse effects. LDL-cholesterol reduction of
atherosclerotic cardiovascular disease (such as approximately 18% to 28% is observed (36% to
this woman) who have not reached their LDL- 46% when used in combination with ezetimibe).
cholesterol targets. In this patient with type 2 However, safety in liver transplant recipients
diabetes, hypertension, and history of transient is unknown.
ischemic attack, the LDL-cholesterol goal is less Fenofibrate (Answer D) is an effective
than 70 mg/dL (<1.81 mmol/L). Addition of triglyceride-lowering agent; however, there
a PCSK9 inhibitor is an appropriate next step. are very limited data on use in liver transplant
Icosapent ethyl (Answer C) is also beneficial in recipients and there is lack of proven
patients at high cardiovascular risk when it is cardiovascular benefit to justify addition at
added to statin therapy; monotherapy benefit this time.
is unclear.
Bempedoic acid (Answer A) is a new agent This patient started PCSK9 inhibitor therapy with
that acts in the same cholesterol biosynthesis excellent response:
pathway as statins. It targets ATP-citrate lyase (an
Total cholesterol = 139 mg/dL (SI: 3.60 mmol/L)
enzyme that is upstream of HMG-CoA reductase), Triglycerides = 311 mg/dL (SI: 3.51 mmol/L)
upregulates LDL receptors, and therefore lowers LDL cholesterol = 16 mg/dL (SI: 0.41 mmol/L)
LDL cholesterol. Bempedoic acid is converted HDL cholesterol = 61 mg/dL (SI: 1.58 mmol/L)
References
1. Speliotes EK, Balakrishnan M, Friedman LS, and Corey KE. Treatment 5. Bays H, Cohen DE, Chalasani N, Harrison SA, The National Lipid
of dyslipidemia in common liver diseases. Clin Gastroenterol Hepatol. Association’s Statin Safety Task Force. An assessment by the Statin Liver
2018;16(8):1189-1196. PMID: 29684459 Safety Task Force: 2014 update. J Clin Lipidol. 2014;8(Suppl 3):S47-S57.
2. Targher G, Corey KE, Byrne CD. NAFLD, and cardiovascular and cardiac PMID: 24793441
diseases: factors influencing risk, prediction and treatment. Diabetes Metab. 6. Brandt EJ, Regnier SM, Leung EK, et al. Management of lipoprotein X and
2021;47(2):101215. PMID: 33296704 its complications in a patient with primary sclerosing cholangitis. Clin Lipidol.
3. Amor AJ, and Perea V. Dyslipidemia in nonalcoholic fatty liver disease. Curr 2015;10(4):305-312. PMID: 26413163
Opin Endocrinol Diabetes Obes. 2019;26(2):103-108. PMID: 30694825 7. Kellick KA, Bottorff M, Toth PP; The National Lipid Association’s Safety
4. Nemes K, Aberg F, Gylling H, Isoniemi H. Cholesterol metabolism Task Force. A clinician’s guide to statin drug-drug interactions. J Clin Lipidol.
in cholestatic liver disease and liver transplantation: from molecular 2014;8(Suppl 3):S30-S46. PMID: 24793440
mechanisms to clinical implications. World J Hepatol. 2016;8(22):924-932.
PMID: 27574546
ENDO 2022 • Cardiovascular Endocrinology 97
Update on Genetic Causes
of Hypercholesterolemia:
What’s New in the Evaluation
and Treatment of These
Hard-to-Treat Patients
Marc-Andre Cornier, MD. Division of Endocrinology, Diabetes, and Metabolic Diseases,
Medical University of South Carolina, Charleston, SC; E-mail:
[email protected]Learning Objectives therapies offer novel mechanisms to substantially
lower LDL cholesterol in these otherwise difficult-
As a result of participating in this session, learners
to-treat patients. Finally, in some patients, LDL
should be able to:
apheresis is necessary and should be considered.
• Recognize and initiate an appropriate
evaluation of patients with
hypercholesterolemia. Significance of the
• Provide individualized goals for treatment of Clinical Problem
hypercholesterolemia.
It was not until the turn of the 20th century that
• Explain different medical treatment options cholesterol was identified as a key component of
for cholesterol lowering. atherosclerosis and that feeding cholesterol to
• Be aware of other treatment options such rodents was shown to produce atherosclerosis. It
as new pharmacologic therapies and LDL was not until the 1930s, though, that the genetic
apheresis and recognize when to refer patients link between high cholesterol and coronary heart
to a lipid disorders specialist. disease (CHD) was identified and described as FH.
This disorder was later known to be consistent with
heterozygous FH and was associated with significantly
elevated cholesterol levels and increased risk for
Main Conclusions premature ASCVD. In the 1950s, the biosynthetic
pathways of cholesterol were further understood.1
Severe hypercholesterolemia usually has a genetic
LDL cholesterol was identified, and high levels
cause such as familial hypercholesterolemia (FH)
were determined to be associated with higher
and is associated with premature atherosclerotic
risk for CHD.2 It was not until the seminal work
cardiovascular disease (ASCVD). Aggressive,
of Brown and Goldstein in the 1970s that the
early treatment is warranted and necessary to
LDL receptor was identified and that a defect in
prevent ASCVD. Combination medical therapy is
the LDL-receptor gene was discovered to be the
typically necessary to adequately reduce the total
primary defect responsible for FH.3 We now know
cholesterol burden. The new PCSK9 modulating
that there are many different pathogenic variants
98 ENDO 2022 • Endocrine Case Management
in the LDL-receptor gene, and depending on the PCSK9 is an important regulator of LDL-
site of the variant, patients with homozygous FH receptor degradation because binding of PCSK9
are unable to produce any functional receptors or to the LDL receptor results in degradation of
are able to produce low levels of normally active the receptor, thereby preventing recycling of
LDL receptors. We also know that patients with the receptor. Individuals with elevated PCSK9
heterozygous FH have about half the normal levels due to gain-of-function pathogenic variants
amount of normally functional LDL receptors. in gene encoding PCSK9 may also have severe
ASCVD remains a major cause of death in the hypercholesterolemia clinically similar to that
United States and around the world. Cholesterol observed in patients with FH. Individuals with
is at the core of atherosclerosis development loss-of-function pathogenic variants in the gene
and is one of the major modifiable risk factors. encoding PCSK9 are associated with increased
Individuals with severe hypercholesterolemia due LDL-receptor function, very low levels of LDL
to a genetic cause are at greatly increased risk for cholesterol, and very low incidence of ASCVD.
premature ASCVD because of a high cumulative This knowledge has led to new treatment options
burden of cholesterol. Current guidelines for lowering LDL cholesterol, including inhibitors
recommend that any individual with an LDL- of PCSK9 by monoclonal antibodies and inhibition
cholesterol concentration greater than 190 mg/dL of the production of PCSK9.
(>4.92 mmol/L) should be treated aggressively.4 Severe hypercholesterolemia can occur in
Severe hypercholesterolemia can occur in the setting of mixed hyperlipidemias, which is
isolation, as seen in the different forms of FH, or it also associated with premature ASCVD. Familial
can be associated with other lipid abnormalities such combined hyperlipidemia is the most common
as concomitantly elevated triglycerides and/or genetic disorder of mixed hyperlipidemia
low HDL cholesterol such as in familial combined occurring in up to 1 in 250 persons. While a
hyperlipidemia and dysbetalipoproteinemia. specific gene variant has not been identified,
These disorders are also associated with a greatly affected individuals have increased production
increased risk for ASCVD. of apolipoprotein B–containing lipoproteins.
Isolated severe hypercholesterolemia is Familial dysbetalipoproteinemia is due to a
generally consistent with FH. Approximately genetic variation in apolipoprotein E (the E2
1 in 1 million persons is homozygous and has isoform). This is associated with reduced clearance
extremely high cholesterol levels with LDL- of remnant particles (chylomicron and VLDL
cholesterol concentrations often reaching greater remnants) and leads to planar and tuberous
than 600 mg/dL (>15.54 mmol/L). If untreated, xanthomas and premature ASCVD if untreated.
these individuals develop significant tendinous “Metabolic dyslipidemia” is the most common
xanthomas and are at risk for ASCVD and death cause of mixed hyperlipidemia associated with
at a very young age (before 20 years). More than other features of the metabolic syndrome, but it is
1600 pathogenic variants in the gene encoding generally associated with more modest elevations
the LDL receptor have been documented. The in cholesterol.
prevalence of heterozygous FH is as high as 1 in
200 to 300 persons depending on the population,
rendering FH as one of the most common serious
Barriers to Optimal Practice
genetic disorders. Untreated patients with
• Lack of effectiveness of available treatment
heterozygous FH typically have LDL-cholesterol
options.
levels in the range of 200 to 300 mg/dL (5.18 to
7.77 mmol/L) and can also develop xanthomas. • Adverse effects of currently available treatment
They are at increased risk for premature ASCVD options.
events before the fourth and fifth decades of life. • Cost of therapies.
ENDO 2022 • Cardiovascular Endocrinology 99
Strategies for Diagnosis, concentration greater than the 95th
percentile for age and sex (2 points)
Therapy, and/or Management
Diagnosis b. Clinical history:
Persons with homozygous FH typically have • Patient has premature (men <55 years,
LDL-cholesterol levels greater than 600 mg/dL women <60 years) CAD (2 points) or
(>15.54 mmol/L) and have a family history that other cardiovascular disease (1 point)
suggests both parents have heterozygous FH.
Such patients must be identified at a young age c. Physical examination:
and treated aggressively. Genetic testing can be • Tendon xanthomata (6 points)
done, but not all of the more than 1600 pathogenic
• Arcus cornealis at an age younger than
variants are identified by clinically available genetic
45 years (4 points)
testing. As such, negative results on genetic testing
does not exclude the diagnosis. Patients with d. Laboratory analysis:
heterozygous FH should be suspected in the setting
of an LDL-cholesterol concentration greater than • LDL cholesterol >330 mg/dL
160 mg/dL (>4.14 mmol/L) in children or greater (>8.55 mmol/L) (8 points)
than 190 mg/dL (>4.92 mmol/L) in adults. Most • LDL cholesterol 250-329 mg/dL
affected adults have an LDL-cholesterol level (>6.48-8.52 mmol/L) (5 points)
greater than 220 mg/dL (>5.70 mmol/L). FH should
• LDL cholesterol 190-249 mg/dL
also be suspected in the setting of the following
(>4.92-6.45 mmol/L) (3 points)
physical examination findings: (1) tendon xanthomas
at any age (most commonly in the Achilles tendon • LDL cholesterol 155-189 mg/dL
and finger extensor tendons; (2) corneal arcus in (>4.01-4.89 mmol/L) (1 points)
patients younger than 45 years; and (3) xanthelasmas • HDL cholesterol and triglycerides are
in patients younger than 20 to 25 years. normal
Different criteria have been established to help • DNA analysis: functional pathogenic
make the clinical diagnosis of FH: variant present in the LDL-receptor
1. Dutch or World Health Organization gene (8 points)
diagnostic criteria. Diagnosis of heterozygous
FH is certain when the point total is greater 2. Simon Broome Register Diagnostic Criteria
than 8, probable when 6 to 8, and possible for heterozygous FH:
when 3 to 5:
a. Definite FH is defined as:
a. Family history:
• Total cholesterol >260 mg/dL
• First-degree relative with known (6.73 mmol/L) or LDL cholesterol
premature (men <55 years, women >155 mg/dL (>4.01 mmol/L) in a
<60 years) cardiovascular disease child younger than 16 years or total
(1 point) cholesterol >290 mg/dL (>7.51 mmol/L)
• First-degree relative with known LDL- or LDL cholesterol >190 mg/dL
cholesterol concentration greater than (>4.92 mmol/L) in an adult PLUS
the 95th percentile for age and sex, tendon xanthomas in the patient or in
and/or with tendon xanthomata and/ a first- or second-degree relative
or arcus cornealis, or children younger OR
than 18 years with an LDL-cholesterol
100 ENDO 2022 • Endocrine Case Management
• DNA-based evidence of an LDLR ASCVD risk factors should also be aggressively
pathogenic variant or familial defective managed, including hypertension, cigarette
apolipoprotein B100 smoking, and unhealthy lifestyle.
In patients with mixed hyperlipidemia, the
b. Possible FH is defined as: primary goal is to treat the hypercholesterolemia
• Total cholesterol >260 mg/dL to reduce the ASCVD risk as discussed above.
(>6.73 mmol/L) or LDL cholesterol Triglycerides should be targeted in patients with
>155 mg/dL (>4.01 mmol/L) in a severe elevations, (>400 or 500 mg/dL [>4.52 or
child younger than 16 years or total 5.65 mmol/L]) with a primary goal of reducing the
cholesterol >290 mg/dL (>7.51 mmol/L) risk of pancreatitis. Consideration should be given
or LDL cholesterol >190 mg/dL to prescribing icosapent ethyl to further reduce
(>4.92 mmol/L) in an adult ASCVD risk in patients with hypertriglyceridemia
and known ASCVD or in patients with diabetes
• And at least 1 of the following:
and multiple ASCVD risk factors.
ԗ Family history of myocardial
infarction at age younger than Treatment Options
50 years in second-degree relative
Medical Therapy
or younger than 60 years in first-
Medical therapy with high-intensity statins is the
degree relative
standard of care and cornerstone of treatment
ԗ Family history of elevated with a goal of reducing LDL cholesterol by 50%
cholesterol values >290 mg/dL or more.4 Patients who are homozygous and
(>7.51 mmol/L) in an adult first- most patients who are heterozygous, however,
or second-degree relative or need more LDL-cholesterol lowering than can
>260 mg/dL (>6.73 mmol/L) in be achieved by statin therapy alone. Multiple
a child or sibling younger than drugs are usually necessary and often may not be
16 years sufficient. Other medical treatment options to
consider include PCSK9 inhibitors, ezetimibe,
It is also important to also consider secondary
bempedoic acid, inclisiran, bile acid sequestrants,
causes of hypercholesterolemia in these patients,
and niacin. In patients who are homozygous,
including hypothyroidism, nephrotic syndrome,
lomitapide and evinacumab are possible options.
cholestatic liver disease, medications, and diet.
Statins are the gold standard therapy due to
the number of clinical trials that have shown
Treatment Goals benefit in patients with and without FH. Recent
The primary treatment goal is to reduce the evidence suggests that combination therapy with
risk of ASCVD-related events. Current joint ezetimibe or PCSK9 inhibitors confers further
guidelines published by the American College of risk reduction.5-7 There is less evidence in support
Cardiology and the American Heart Association of other treatment options such as combination
in 2018 have established that all individuals with therapy with bile acid sequestrants. More recently,
an LDL-cholesterol value greater than 190 mg/dL there have been 2 trials with negative outcomes
(>4.92 mmol/L) should be treated with a high- regarding the addition of niacin to statin therapy.
intensity statin (atorvastatin, 40/80 mg or Two PCSK9 inhibitors, alirocumab and
rosuvastatin, 20/40 mg) to lower LDL cholesterol evolocumab, have now been approved in the
by at least 50%.4 Secondary goals for primary United States for the treatment of heterozygous
prevention are to lower LDL-cholesterol levels FH8,9 and should be considered as an important
to less than 100 mg/dL (<2.59 mmol/L). Other treatment option for those patients who do not
ENDO 2022 • Cardiovascular Endocrinology 101
respond adequately to statins +/- ezetimibe. must be repeated on a regular basis, generally
These agents lower LDL cholesterol 60%, on every 1 to 2 weeks. Some patients are able to
average, beyond maximally tolerated statin extend the frequency of apheresis with newer
therapy. These agents have also been approved medical therapies. Because of the frequent need for
for the treatment of homozygous FH with vascular access, many, if not most, patients require
LDL-cholesterol lowering 20% to 40% beyond fistulas. Successful treatment of patients with LDL
maximally tolerated statin therapy.10 Inclisiran is apheresis requires a well-coordinated team of
small interfering RNA that inhibits translation experienced personnel.12
of PCSK9 and thus the production of PCSK9.
It was recently approved for use in Europe and Nonlipid Treatments
the United States to treat heterozygous FH. It Lifestyle modification, including cigarette smoking
is dosed at baseline, 3 months, and then every cessation, increased physical activity, and reduced
6 months thereafter as a subcutaneous injection. saturated fat intake should be recommended to
Inclisiran reduces LDL-cholesterol levels by 50% all patients with hypercholesterolemia. Secondary
to 55%, on average, beyond maximally tolerated causes of hyperlipidemia should also be treated
statin therapy.11 Bempedoic acid, an inhibitor of if present. Hypertension and diabetes should be
adenosine triphosphate citrate lyase upstream aggressively treated. Antiplatelet therapy with
from HMG-CoA reductase in the cholesterol low-dosage aspirin should be considered.
synthesis pathway, is also approved for the
treatment of heterozygous FH. It is taken orally at Clinical Case Vignettes
a dosage of 180 mg daily and results in 15% to 27%
LDL-cholesterol lowering depending on other Case 1
background therapy. A 45-year-old woman is referred for evaluation
Lomitapide and evinacumab are also approved and management of hyperlipidemia. She was
for the treatment of homozygous FH. Lomitapide otherwise healthy until she had a transient
is a microsomal triglyceride transfer protein ischemic attack and myocardial infarction with
inhibitor that is taken orally at a dosage of 5 to stent placement 2 years ago. Atorvastatin,
60 mg daily. Lomitapide has been shown to reduce 80 mg daily, was started after her myocardial
LDL cholesterol by up to 40% in those who can infarction for “high” cholesterol, but she developed
tolerate it. Fat in the diet must be restricted significant muscle aches on this treatment. Her
to prevent diarrhea. Because it inhibits VLDL cardiologist then tried rosuvastatin, 20 mg daily,
secretion, triglycerides can accumulate in the but she again developed muscle aches. She has
liver, leading to hepatotoxicity. Evinacumab is a been able to tolerate pravastatin, 20 mg daily.
monoclonal antibody against ANGPTL3, which She has not had any further ASCVD events to
is dosed as a monthly intravenous infusion. this point. She also takes clopidogrel, carvedilol,
Evinacumab has been shown to reduce LDL lisinopril, and sertraline. Her other medical
cholesterol by 45% to 50%. problems include hypertension and depression.
Her family history is notable for her father having
Apheresis a myocardial infarction in his 40s and her brother
LDL apheresis, the direct removal of cholesterol, undergoing coronary artery bypass graft surgery in
has been shown to prolong survival. Apheresis his early 50s. She does not smoke cigarettes, tries
should be considered in patients at high risk such to eat a diet low in saturated fat, and walks 2 miles
as those with known ASCVD whose condition every day.
has been refractory to medical therapy and/or Physical examination findings are unremarkable
those intolerant to medical therapy. LDL apheresis except for the presence of arcus cornealis.
only temporarily removes LDL particles and thus
102 ENDO 2022 • Endocrine Case Management
Laboratory test results (sample drawn while fasting): first be tried. This could be done by increasing
the dosage of the current treatment (pravastatin),
Total cholesterol = 247 mg/dL (SI: 6.40 mmol/L)
Triglycerides = 100 mg/dL (SI: 1.13 mmol/L) trying a different higher-potency statin such
LDL cholesterol = 175 mg/dL (SI: 4.53 mmol/L) as simvastatin or pitavastatin (Answer D), or
HDL cholesterol = 52 mg/dL (SI: 1.35 mmol/L) retrying atorvastatin or rosuvastatin at a lower
Complete metabolic panel, normal dosage (Answer C). It would also be reasonable
to try combination therapy with an agent such as
Question 1. What should this patient’s ezetimibe (Answer B), bempedoic acid, or a bile
primary LDL-cholesterol goal be? acid sequestrant in addition to maximized statin
therapy. Finally, PCSK9 inhibitors (Answer A) are
A. LDL cholesterol <100 mg/dL (<2.59 mmol/L)
also approved for the treatment of heterozygous
B. LDL cholesterol <70 mg/dL (<1.81 mmol/L) FH in patients who have not achieved control on
C. LDL-cholesterol reduction >50% maximally tolerated statin therapy. These agents
D. Lowest LDL-cholesterol level she can tolerate are the most likely to achieve good control of severe
hypercholesterolemia. Aggressive medical therapy
Answer: C) LDL-cholesterol reduction >50% may also help prevent the need for LDL apheresis.
Based on current guidelines, the primary goal would Apheresis might be a good option if this patient’s
be to treat this patient with high-intensity statin LDL cholesterol cannot be controlled with
therapy with a goal of reducing her LDL cholesterol aggressive medical therapy and/or she continues
by 50% or more (Answer C). Unfortunately, to have recurrent ASCVD events. Before the
she has been unable to tolerate high-intensity approval of PCSK9 inhibitors, apheresis was more
statin therapy. In addition, a more aggressive commonly used as a way to control the overall
LDL-cholesterol goal aiming for concentrations cholesterol burden to which these patients
lower than 70 mg/dL (<1.81 mmol/L) (Answer are exposed.
B) is a reasonable secondary goal in light of her
known clinical ASCVD. An LDL-cholesterol goal Case 2
less than 100 mg/dL (<2.59 mmol/L) (Answer A 56-year-old man with a complex medical history
A) is not aggressive enough. While potentially recently moved to the area and seeks your help. He
practical, aiming for the lowest LDL-cholesterol was diagnosed with “homozygous FH” and significant
level she can tolerate (Answer D) is not specific coronary artery disease and has been treated with
or aggressive enough and is not supported by LDL apheresis. He was first diagnosed 15 years
any guidelines. ago, although he knew that his cholesterol was
very high since age 30 years. He has had multiple
Question 2. What are the treatment options?
stents and myocardial infarctions and is status post
A. Add a PCSK9 inhibitor coronary artery bypass graft surgery before apheresis
B. Add ezetimibe was started. He has had LDL-cholesterol levels in the
C. Retry atorvastatin or rosuvastatin at a lower range of 500 to 600 mg/dL (12.95 to 15.54 mmol/L)
dosage and has xanthomas of the knuckles, as well as
D. Try a different more effective statin such as the Achilles tendons. He has a very strong family
simvastatin or pitavastatin history of heart disease in both parents and knows
of family members who have cholesterol levels
E. All of the above
greater than 300 mg/dL (>7.77 mmol/L). He has
Answer: E) All of the above been intolerant to statins because of myopathy.
Colesevelam led to significant gastrointestinal
All of these options are reasonable to consider and adverse effects, niacin led to intolerable flushing,
try (Answer E). Maximizing statin therapy should
ENDO 2022 • Cardiovascular Endocrinology 103
and ezetimibe caused muscle aches. He finally C. Add a PCSK9 inhibitor
started LDL apheresis 4 years ago, which has D. Continue LDL apheresis
helped reduce the occurrence of new ASCVD E. All of the above
events. He has had more trouble with angina,
however, since moving to a higher altitude. Answer: E) All of the above
Question 1. What should this patient’s All of these options are reasonable to consider
LDL-cholesterol goal be? and try (Answer E). At this time, LDL apheresis
(Answer D) should be continued, as he has had
A. LDL cholesterol <150 mg/dL (<3.9 mmol/L)
persistent disease and apheresis has helped reduce
B. LDL cholesterol <100 mg/dL (cholesterol event rates. It would also be reasonable, though,
2.6 mmol/L) to add medical therapy to see if the frequency of
C. LDL-cholesterol reduction >50% apheresis could be reduced. All 3 listed medical
D. Lowest LDL-cholesterol level he can tolerate treatment options are approved for patients with
E. Treatment with statin therapy is primary goal homozygous FH. While all of these agents are
associated with a high cost, they are much less
Answer: D) Lowest LDL-cholesterol level he can tolerate expensive than frequent apheresis.
PCSK9 inhibitors (Answer C) are approved
Based on current guidelines, the primary goal
for the treatment of homozygous FH with LDL-
would be to treat this patient with high-intensity
cholesterol lowering of 20% to 30%.
statin therapy to reduce his LDL cholesterol by
Lomitapide (Answer B) has been shown
50% or more. However, he has persistent ASCVD.
to lower LDL cholesterol up to 40% at higher
As such, his LDL cholesterol should be treated as
dosages, but it can be difficult to tolerate.
aggressively as he can tolerate (thus, Answer D is
Evinacumab (Answer A) lowers LDL cholesterol
correct and Answers A, B, C, and E are incorrect).
45% to 50% as a monthly intravenous infusion.
Question 2. What are the treatment options? Combinations of these newer agents have not
been studied.
A. Add evinacumab
B. Add lomitapide
References
1. Bloch K. The biological synthesis of cholesterol. Science. 1965;150(3692):19- 7. Schwartz GG, Step PG, Szarek M, et al; ODYSSEY OUTCOMES Committees
28. PMID: 5319508 and Investigators. Alirocumab and cardiovasular outcomes after acute
2. Gofman JW, Glazier F, Tamplin A, Strisower B, De Lalla O. Lipoproteins, coronary syndrome. N Engl J Med. 2018;379(22):2097-2107. PMID: 30403574
coronary heart disease, and atherosclerosis. Physiol Rev. 1954;34(3):589-607. 8. Raal FJ, Stein EA, Dufour R, et al; RUTHERFORD-2 Investigators.
PMID: 13185756 PCSK9 inhibition with evolocumab (AMG 145) in heterozygous familial
3. Goldstein JL, Brown MS. Familial hypercholesterolemia: identification of a hypercholesterolaemia (RUTHERFORD-2): a randomised, double-blind,
defect in the regulation of 3-hydroxy-3-methylglutaryl coenzyme A reductase placebo-controlled trial. Lancet. 2015;385(9965):331-340. PMID: 25282519
activity associated with overproduction of cholesterol. Proc Natl Acad Sci U S 9. Kastelein JJP, Ginsberg HN, Langslet G, et al. ODYSSEY FH I and FH II:
A. 1973;70(10):2804-2808. PMID: 4355366 78 week results with alirocumab treatment in 735 patients with heterozygous
4. Grundy SM, Stone NJ, Bailey AL, et al. 2018 AHA/ACC/AACVPR/ familial hypercholesterolaemia. Eur Heart J. 2015;36(43):2996-3003. PMID:
AAPA/ABC/ACPM/ADA/AGS/APhA/ASPC/NLA/PCNA guideline on 26330422
the management of blood cholesterol: a report of the American College of 10. Raal FJ, Honarpour N, Blom DJ, et al; TESLA Investigators. Inhibition of
Cardiology/American Heart Association Task Force on Clinical Practice PCSK9 with evolocumab in homozygous familial hypercholesterolaemia
Guidelines. J Am Coll Cardiol. 2019;73(24):e285-e350. PMID: 30423393 (TESLA Part B): a randomised, double-blind, placebo-controlled trial. Lancet.
5. Cannon CP, Blazing MA, Giugliano RP, et al. Ezetimibe added to statin 2015;385(9965):341-350. PMID: 25282520
therapy after acute coronary syndromes. N Engl J Med. 2015;372(25):2387- 11. Ray KK, Landmesser U, Leiter LA, et al. Inclisiran in patients at
2397. PMID: 26039521 high cardiovasular risk with elevated LDL cholesterol. N Engl J Med.
6. Sabatine MS, Giugliano RP, Keech AC, et al; FOURIER Steering Committee 2017;376(15):1430-1440. PMID: 28306389
and Investigators. Evolocumab and clinical outcomes in patients with